Ulcerative colitis and an abnormal cholangiogram

Article Type
Changed
Fri, 02/16/2018 - 11:07
Display Headline
Ulcerative colitis and an abnormal cholangiogram

A 49-year-old man has had ulcerative colitis for more than 30 years. It is well controlled with sulfasalazine (Azulfidine). Now, he has come to see his primary care physician because for the past 3 months he has had mild, intermittent pain in his right upper abdominal quadrant.

His physical examination is normal. Routine laboratory testing shows the following:

  • Hemoglobin 14.2 g/dL (reference range 13.5–17.5)
  • White blood cell count 6.7 × 109/L (3.5–10.5)
  • Platelet count 279 × 109/L (150–450)
  • Alkaline phosphatase 387 U/L (45–115)
  • Total bilirubin 0.9 mg/dL (0.1–1.0)
  • Aspartate aminotransferase (AST) 35 U/L (35–48)
  • Alanine aminotransferase (ALT) 30 U/L (7–55).

Figure 1. Intraoperative cholangiography demonstrates annular, multifocal stricturing and beading of the extrahepatic biliary system (arrow).
His physician is concerned about his elevated alkaline phosphatase level, which can be a sign of cholestatic liver disease (ie, involving blockage of the flow of bile). He sends him for ultrasonography, which reveals mild thickening of the gallbladder wall. The patient is referred to a general surgeon, who decides to remove the gallbladder. The procedure goes well, but when contrast dye is injected into the biliary system during cholangiography, the image is markedly abnormal (Figure 1). The patient is referred to Mayo Clinic for further evaluation.

WHAT IS THE DIAGNOSIS?

1. Based on this information, which of the following is the most likely diagnosis?

  • Autoimmune hepatitis
  • Primary sclerosing cholangitis
  • Primary biliary cirrhosis
  • Idiopathic adulthood ductopenia

Primary sclerosing cholangitis

The most likely diagnosis is primary sclerosing cholangitis, a chronic cholestatic liver disease characterized by diffuse inflammatory destruction of intrahepatic and extrahepatic bile ducts, resulting in fibrosis, cirrhosis, and liver failure. Its cause is unknown, but it is likely the result of acquired exposures interacting with predisposing host factors. Current diagnostic criteria include:

  • Characteristic cholangiographic abnormalities of the biliary tree
  • Compatible clinical and biochemical findings (typically cholestasis with elevated alkaline phosphatase levels for at least 6 months)
  • Exclusion of causes of secondary sclerosing cholangitis: secondary sclerosing cholangitis is characterized by a similar multifocal biliary stricturing process, but with an identifiable cause such as long-term biliary obstruction, surgical biliary trauma, or recurrent pancreatitis.1

At presentation, the most common liver enzyme abnormality is an elevated alkaline phosphatase level, often three or four times the normal level.2 In contrast, aminotransferase levels are only modestly elevated, less than three times the upper limit of normal.3 At the time of diagnosis, serum bilirubin levels are normal in 60% of patients.4

Two large epidemiologic studies (one from Olmsted County, MN,5 the other from Swansea, Wales, UK6) estimated the age-adjusted incidence of primary sclerosing cholangitis to be 0.9 per 100,000 individuals. The median age of the patients at onset was in the 30s or 40s, and most were men. At 10 years, an estimated 65% were still alive and had not undergone liver transplantation—a significantly lower percentage than in age- and sex-matched populations.

It is estimated that more than 70% of patients with primary sclerosing cholangitis also have inflammatory bowel disease.5 In fact, the most common presentation of primary sclerosing cholangitis is asymptomatic inflammatory bowel disease and persistently elevated alkaline phosphatase—usually first noted on routine biochemical screening, as in our patient.

Imaging of the biliary tree is essential for the diagnosis of primary sclerosing cholangitis. Typical findings on cholangiography include multifocal stricturing and beading, usually involving both the intrahepatic and the extrahepatic biliary systems, as in our patient (Figure 1). Endoscopic retrograde cholangiopancreatography (ERCP) is considered the gold standard imaging test, but recent studies have shown that magnetic resonance cholangiopancreatography (MRCP) is an acceptable noninvasive substitute,7 and it may cost less per diagnosis.8

Liver biopsy alone is generally nondiagnostic because the histologic changes are quite variable in different segments of the same liver. The classic “onion-skin fibrosis” of primary sclerosing cholangitis is seen in fewer than 10% of biopsy specimens.9

Autoimmune hepatitis

Autoimmune hepatitis is chronic and is characterized by circulating autoantibodies and high serum globulin concentrations.10 Its presentation is heterogeneous, varying from no symptoms to nonspecific symptoms of malaise, fatigue, abdominal pain, itching, and arthralgia. Generally, elevations in aminotransferases are much more prominent than abnormalities in bilirubin and alkaline phosphatase levels10—unlike the pattern in our patient.

Primary biliary cirrhosis

Primary biliary cirrhosis is diagnosed if the patient has at least two of these three clinical criteria:

  • Biochemical evidence of cholestasis, with elevation of alkaline phosphatase for at least 6 months
  • Antimitochondrial antibody
  • Histologic evidence of nonsuppurative cholangitis and destruction of small or medium-sized bile ducts.11

In patients who lack antimitochondrial antibody, liver biopsy is necessary to establish the diagnosis. Given that primary biliary cirrhosis involves only small and medium-sized bile ducts, cholangiography is usually normal unless the patient has advanced cirrhosis.

Idiopathic adulthood ductopenia

Idiopathic adulthood ductopenia is a rare condition of unknown cause that involves the progressive destruction of segments of the small bile ducts inside the liver (“small-duct” biliary disease).12 Laboratory findings reveal a cholestatic pattern of liver injury, but biopsy samples show no features diagnostic or suggestive of another biliary disease; cholangiography is typically normal.12,13

 

 

ASSOCIATION WITH INFLAMMATORY BOWEL DISEASE

2. Which statement best characterizes inflammatory bowel disease associated with primary sclerosing cholangitis?

  • Crohn disease of the small bowel is the most common form
  • Liver disease often precedes the bowel disease
  • Treating the underlying bowel disease improves the long-term prognosis for the liver condition
  • Patients with primary sclerosing cholangitis and chronic ulcerative colitis are at higher risk of colonic dysplasia than patients with chronic ulcerative colitis alone

From 70% to 80% of patients with primary sclerosing cholangitis also have inflammatory bowel disease, usually chronic ulcerative colitis.14,15 Conversely, 2.4% to 4% of patients with ulcerative colitis and 1.4% to 3.4% of patients with Crohn disease have primary sclerosing cholangitis.1

Typically, the diagnosis of inflammatory bowel disease is made 8 to 10 years before the diagnosis of liver disease, although cases have also been reported to occur years after the diagnosis of cholangitis.15,16

No association between the severity of bowel disease and liver disease has been reported, and treating the inflammatory bowel disease does not alter the natural history of primary sclerosing cholangitis. Particularly, proctocolectomy, the most aggressive treatment for chronic ulcerative colitis, appears to have no effect on the course of the cholangitis.17

In patients with both primary sclerosing cholangitis and chronic ulcerative colitis, the risk of colonic dysplasia is higher than in patients with chronic ulcerative colitis alone.18 Recent studies have predicted that the risk of colorectal carcinoma in patients with primary sclerosing cholangitis and inflammatory bowel disease is as high as 25% after 10 years.19,20 Therefore, annual colonoscopy with surveillance biopsy is recommended in patients with both primary sclerosing cholangitis and chronic ulcerative colitis, since screening and early detection improve survival rates.15

TREATMENT AND PROGNOSIS

After being diagnosed with primary sclerosing cholangitis, the patient inquires about ongoing medical therapy and long-term prognosis.

3. Which is the only life-prolonging therapy for primary sclerosing cholangitis?

  • Methotrexate (Trexall)
  • Ursodeoxycholic acid (UDCA) (Actigall) at a standard dosage (13–15 mg/kg/day)
  • UDCA at a high dosage (20–30 mg/kg/day)
  • Liver transplantation

Drug therapy has not been shown to improve the prognosis of primary sclerosing cholangitis.

In randomized placebo-controlled trials, penicillamine (Depen), colchicine (Colcrys), methotrexate, and UDCA (13–15 mg/kg per day) failed to show efficacy.21–23

In pilot studies, high-dose UDCA (20 to 30 mg/kg/day) initially appeared to bring an improvement in survival probability, with trends toward histologic improvement,24,25 but larger randomized placebo-controlled trials found no improvement in symptoms, quality of life, survival rates, or risk of cholangiocarcinoma with high-dose UDCA.26,27 In fact, in 5 years of follow-up, patients on high-dose UDCA had a risk of death or transplantation two times higher than with placebo.27 One study indicated UDCA may decrease the incidence of colonic dysplasia in patients with primary sclerosing cholangitis and chronic ulcerative colitis.28 However, more prospective studies are required to better define the routine use of UDCA as a prophylactic agent.

Liver transplantation remains the most effective treatment for primary sclerosing cholangitis, and it improves the rate of survival.29 Nevertheless, about 20% of patients who undergo transplantation have a recurrence of cholangitis, and it may recur earlier after living-donor liver transplantation, particularly when the graft is from a biologically related donor.30 Proposed risk factors for recurrence include inflammatory bowel disease, prolonged ischemia time, the number of cellular rejection events, prior biliary surgery, cytomegalovirus infection, and lymphocytotoxic cross-match.31

4. In addition to cirrhosis and cholangitis, which of the following is a potential long-term complication of primary sclerosing cholangitis?

  • Colon cancer
  • Cholangiocarcinoma
  • Osteoporosis
  • Fat-soluble vitamin deficiency
  • All of the above

All are potential long-term complications.

Colon cancer. Concomitant chronic ulcerative colitis puts the patient at a higher risk of colonic dysplasia compared with patients with chronic ulcerative colitis alone.18 According to recent studies of patients with primary sclerosing cholangitis and inflammatory bowel disease, 19,20 the risk of colorectal carcinoma after 10 years of disease is as high as 25%.

Cholangiocarcinoma. Primary sclerosing cholangitis is considered a risk factor for cholangiocarcinoma, with an estimated 10-year cumulative incidence of 7% to 9%.1,20 In a retrospective study of 30 patients,32 the median survival was 5 months from the time of diagnosis of cholangiocarcinoma; at the time of diagnosis approximately 19 patients (63%) had metastatic disease.

At present, early detection of cholangiocarcinoma is hampered by the low sensitivity and specificity of standard diagnostic approaches. Carbohydrate antigen 19-9 has been used as a marker, but it has questionable accuracy, since elevations of this antigen can also be a result of pancreatic malignancy and bacterial cholangitis. However, cholangiocarcinoma should be suspected when patients present with progressive jaundice, weight loss, abdominal discomfort, and a sudden rise in carbohydrate antigen 19-9.

Conventional ultrasonography and computed tomography (CT) have poor sensitivity for detecting this malignancy. ERCP with biliary brushings should be considered when evaluating for biliary malignancy. New diagnostic methods such as digitized image analysis and fluorescence in situ hybridization on biliary brushings offer promise to evaluate bile duct lesions for cellular aneuploidy and chromosomal aberrations, which may improve the detection of cholangiocarcinoma.33 A recent large-scale study of nearly 500 patients showed that fluorescence in situ hybridization had a higher sensitivity (42.9%) than routine cytology (20.1%) with identical specificity (99.6%) for malignancy.34

Metabolic bone disease, usually osteoporosis rather than osteomalacia, is relatively common and is an important complication of primary sclerosing cholangitis.35 Patients with osteoporosis should be treated with vitamin D and calcium supplementation. Bisphosphonates have been used with varying results in primary biliary cirrhosis36 and can be considered in patients with advanced osteoporosis.

Fat-soluble vitamin deficiency is relatively common in primary sclerosing cholangitis, particularly as it progresses to advanced liver disease. Up to 40% of patients have vitamin A deficiency, 14% have vitamin D deficiency, and 2% have vitamin E deficiency.37 Patients can undergo simple oral replacement therapy.

 

 

A stone is removed, fever develops

Three years after the diagnosis of primary sclerosing cholangitis, the patient develops mild hyperbilirubinemia and undergoes ERCP at his local hospital. A stone is found obstructing the common bile duct and is successfully extracted.

Twenty-four hours after this procedure, he develops severe right-upper-quadrant pain and fever. He is seen at his local emergency department and blood cultures are drawn. He is started on antibiotics and is transferred to Mayo Clinic for further management.

5. In addition to continuing a broad-spectrum antibiotic, which would be the next best step for this patient?

  • ERCP
  • MRCP
  • Abdominal ultrasonography
  • Abdominal CT

The patient’s clinical presentation is consistent with acute bacterial cholangitis. The classic Charcot triad of fever, right-upper-quadrant pain, and jaundice occurs in only 50% to 75% of patients with acute cholangitis.38 In addition to receiving a broad-spectrum antibiotic, patients with bacterial cholangitis require emergency endoscopic evaluation—ERCP—to find and remove stones from the bile ducts and, if necessary, to dilate the biliary strictures to allow adequate drainage.

In our experience, more than 10% of patients with primary sclerosing cholangitis who undergo ERCP develop complications requiring hospitalization.39 The procedure generally takes longer to perform and the incidence of cholangitis is higher, despite routine antibiotic prophylaxis, in patients with primary sclerosing cholangitis than in those without it. However, the overall risk of pancreatitis, perforation, and bleeding was similar in patients with or without sclerosing cholangitis.39

MRCP is a promising noninvasive substitute for ERCP in establishing the diagnosis of primary sclerosing cholangitis.7,8 Unfortunately, as with other noninvasive imaging studies such as abdominal ultrasonography and CT, MRCP does not allow for therapeutic biliary decompression.

The patient undergoes ERCP with stenting

The patient’s acute cholangitis is thought to be a complication of his recent ERCP procedure. He undergoes emergency ERCP with balloon dilation and placement of a temporary left hepatic stent. His fever improves and he is discharged 48 hours later. He completes a 14-day course of antibiotics for Enterococcus faecalis bacteremia. Six weeks later, he undergoes ERCP yet again to remove the stent and tolerates the procedure well without complications.

TAKE-HOME POINTS

  • Primary sclerosing cholangitis is a progressive cholestatic liver disease of unknown etiology that primarily affects men during the fourth decade of life.
  • This condition is strongly associated with inflammatory bowel disease, particularly with ulcerative colitis.
  • Cholangiocarcinoma and colon cancer are dreaded complications.
  • Liver transplantation is the only life-extending therapy for primary sclerosing cholangitis; however, the condition can recur in the allograft.
References
  1. Chapman R, Fevery J, Kalloo A, et al; American Association for the Study of Liver Diseases. Diagnosis and management of primary sclerosing cholangitis. Hepatology 2010; 51:660678.
  2. Silveira MG, Lindor KD. Clinical features and management of primary sclerosing cholangitis. World J Gastroenterol 2008; 14:33383349.
  3. Lee YM, Kaplan MM. Primary sclerosing cholangitis. N Engl J Med 1995; 332:924933.
  4. Talwalkar JA, Lindor KD. Primary sclerosing cholangitis. Inflamm Bowel Dis 2005; 11:6272.
  5. Bambha K, Kim WR, Talwalkar J, et al. Incidence, clinical spectrum, and outcomes of primary sclerosing cholangitis in a United States community. Gastroenterology 2003; 125:13641369.
  6. Kingham JG, Kochar N, Gravenor MB. Incidence, clinical patterns, and outcomes of primary sclerosing cholangitis in South Wales, United Kingdom. Gastroenterology 2004; 126:19291930.
  7. Berstad AE, Aabakken L, Smith HJ, Aasen S, Boberg KM, Schrumpf E. Diagnostic accuracy of magnetic resonance and endoscopic retrograde cholangiography in primary sclerosing cholangitis. Clin Gastroenterol Hepatol 2006; 4:514520.
  8. Talwalkar JA, Angulo P, Johnson CD, Petersen BT, Lindor KD. Cost-minimization analysis of MRC versus ERCP for the diagnosis of primary sclerosing cholangitis. Hepatology 2004; 40:3945.
  9. Ludwig J, Barham SS, LaRusso NF, Elveback LR, Wiesner RH, McCall JT. Morphologic features of chronic hepatitis associated with primary sclerosing cholangitis and chronic ulcerative colitis. Hepatology 1981; 1:632640.
  10. Krawitt EL. Autoimmune hepatitis. N Engl J Med 2006; 354:5466.
  11. Lindor KD, Gershwin ME, Poupon R, Kaplan M, Bergasa NV, Heathcote EJ; American Association for Study of Liver Diseases. Primary biliary cirrhosis. Hepatology 2009; 50:291308.
  12. Ludwig J, Wiesner RH, LaRusso NF. Idiopathic adulthood ductopenia. A cause of chronic cholestatic liver disease and biliary cirrhosis. J Hepatol 1988; 7:193199.
  13. Ludwig J. Idiopathic adulthood ductopenia: an update. Mayo Clin Proc 1998; 73:285291.
  14. Fausa O, Schrumpf E, Elgjo K. Relationship of inflammatory bowel disease and primary sclerosing cholangitis. Semin Liver Dis 1991; 11:3139.
  15. Loftus EV, Aguilar HI, Sandborn WJ, et al. Risk of colorectal neoplasia in patients with primary sclerosing cholangitis and ulcerative colitis following orthotopic liver transplantation. Hepatology 1998; 27:685690.
  16. Loftus EV, Sandborn WJ, Tremaine WJ, et al. Risk of colorectal neoplasia in patients with primary sclerosing cholangitis. Gastroenterology 1996; 110:432440.
  17. Cangemi JR, Wiesner RH, Beaver SJ, et al. Effect of proctocolectomy for chronic ulcerative colitis on the natural history of primary sclerosing cholangitis. Gastroenterology 1989; 96:790794.
  18. Broomé U, Löfberg R, Veress B, Eriksson LS. Primary sclerosing cholangitis and ulcerative colitis: evidence for increased neoplastic potential. Hepatology 1995; 22:14041408.
  19. Kornfeld D, Ekbom A, Ihre T. Is there an excess risk for colorectal cancer in patients with ulcerative colitis and concomitant primary sclerosing cholangitis? A population based study. Gut 1997; 41:522525.
  20. Claessen MM, Vleggaar FP, Tytgat KM, Siersema PD, van Buuren HR. High lifetime risk of cancer in primary sclerosing cholangitis. J Hepatol 2009; 50:158164.
  21. Lindor KD. Ursodiol for primary sclerosing cholangitis. Mayo Primary Sclerosing Cholangitis-Ursodeoxycholic Acid Study Group. N Engl J Med 1997; 336:691695.
  22. Olsson R, Broomé U, Danielsson A, et al. Colchicine treatment of primary sclerosing cholangitis. Gastroenterology 1995; 108:11991203.
  23. LaRusso NF, Wiesner RH, Ludwig J, MacCarty RL, Beaver SJ, Zinsmeister AR. Prospective trial of penicillamine in primary sclerosing cholangitis. Gastroenterology 1988; 95:10361042.
  24. Mitchell SA, Bansi DS, Hunt N, Von Bergmann K, Fleming KA, Chapman RW. A preliminary trial of high-dose ursodeoxycholic acid in primary sclerosing cholangitis. Gastroenterology 2001; 121:900907.
  25. Cullen SN, Rust C, Fleming K, Edwards C, Beuers U, Chapman RW. High dose ursodeoxycholic acid for the treatment of primary sclerosing cholangitis is safe and effective. J Hepatol 2008; 48:792800.
  26. Olsson R, Boberg KM, de Muckadell OS, et al. High-dose ursodeoxycholic acid in primary sclerosing cholangitis: a 5-year multicenter, randomized, controlled study. Gastroenterology 2005; 129:14641472.
  27. Lindor KD, Kowdley KV, Luketic VA, et al. High-dose ursodeoxycholic acid for the treatment of primary sclerosing cholangitis. Hepatology 2009; 50:808814.
  28. Tung BY, Emond MJ, Haggitt RC, et al. Ursodiol use is associated with lower prevalence of colonic neoplasia in patients with ulcerative colitis and primary sclerosing cholangitis. Ann Intern Med 2001; 134:8995.
  29. Wiesner RH, Porayko MK, Hay JE, et al. Liver transplantation for primary sclerosing cholangitis: impact of risk factors on outcome. Liver Transpl Surg 1996; 2(suppl 1):99108..
  30. Tamura S, Sugawara Y, Kaneko J, Matsui Y, Togashi J, Makuuchi M. Recurrence of primary sclerosing cholangitis after living donor liver transplantation. Liver Int 2007; 27:8694.
  31. Gautam M, Cheruvattath R, Balan V. Recurrence of autoimmune liver disease after liver transplantation: a systematic review. Liver Transpl 2006; 12:18131824.
  32. Rosen CB, Nagorney DM, Wiesner RH, Coffey RJ, LaRusso NF. Cholangiocarcinoma complicating primary sclerosing cholangitis. Ann Surg 1991; 213:2125.
  33. Lazaridis KN, Gores GJ. Cholangiocarcinoma. Gastroenterology 2005; 128:16551667.
  34. Fritcher EG, Kipp BR, Halling KC, et al. A multivariable model using advanced cytologic methods for the evaluation of indeterminate pancreatobiliary strictures. Gastroenterology 2009; 136:21802186.
  35. Hay JE, Lindor KD, Wiesner RH, Dickson ER, Krom RA, LaRusso NF. The metabolic bone disease of primary sclerosing cholangitis. Hepatology 1991; 14:257261.
  36. Guañabens N, Parés A, Ros I, et al. Alendronate is more effective than etidronate for increasing bone mass in osteopenic patients with primary biliary cirrhosis. Am J Gastroenterol 2003; 98:22682274.
  37. Jorgensen RA, Lindor KD, Sartin JS, LaRusso NF, Wiesner RH. Serum lipid and fat-soluble vitamin levels in primary sclerosing cholangitis. J Clin Gastroenterol 1995; 20:215219.
  38. Saik RP, Greenburg AG, Farris JM, Peskin GW. Spectrum of cholangitis. Am J Surg 1975; 130:143150.
  39. Bangarulingam SY, Gossard AA, Petersen BT, Ott BJ, Lindor KD. Complications of endoscopic retrograde cholangiopancreatography in primary sclerosing cholangitis. Am J Gastroenterol 2009; 104:855860.
Article PDF
Author and Disclosure Information

Douglas L. Nguyen, MD
Resident Physician, Department of Internal Medicine, Mayo Clinic College of Medicine, Rochester, MN

Konstantinos N. Lazaridis, MD
Associate Professor of Medicine, Center for Basic Research in Digestive Diseases, Division of Gastroenterology and Hepatology, Mayo Clinic College of Medicine, Rochester, MN

Address: Konstantinos N. Lazaridis, MD, Division of Gastroenterology and Hepatology, Mayo Clinic College of Medicine, 200 First Street SW, Rochester, MN 55905; e-mail [email protected]

Issue
Cleveland Clinic Journal of Medicine - 78(5)
Publications
Topics
Page Number
306-311
Sections
Author and Disclosure Information

Douglas L. Nguyen, MD
Resident Physician, Department of Internal Medicine, Mayo Clinic College of Medicine, Rochester, MN

Konstantinos N. Lazaridis, MD
Associate Professor of Medicine, Center for Basic Research in Digestive Diseases, Division of Gastroenterology and Hepatology, Mayo Clinic College of Medicine, Rochester, MN

Address: Konstantinos N. Lazaridis, MD, Division of Gastroenterology and Hepatology, Mayo Clinic College of Medicine, 200 First Street SW, Rochester, MN 55905; e-mail [email protected]

Author and Disclosure Information

Douglas L. Nguyen, MD
Resident Physician, Department of Internal Medicine, Mayo Clinic College of Medicine, Rochester, MN

Konstantinos N. Lazaridis, MD
Associate Professor of Medicine, Center for Basic Research in Digestive Diseases, Division of Gastroenterology and Hepatology, Mayo Clinic College of Medicine, Rochester, MN

Address: Konstantinos N. Lazaridis, MD, Division of Gastroenterology and Hepatology, Mayo Clinic College of Medicine, 200 First Street SW, Rochester, MN 55905; e-mail [email protected]

Article PDF
Article PDF

A 49-year-old man has had ulcerative colitis for more than 30 years. It is well controlled with sulfasalazine (Azulfidine). Now, he has come to see his primary care physician because for the past 3 months he has had mild, intermittent pain in his right upper abdominal quadrant.

His physical examination is normal. Routine laboratory testing shows the following:

  • Hemoglobin 14.2 g/dL (reference range 13.5–17.5)
  • White blood cell count 6.7 × 109/L (3.5–10.5)
  • Platelet count 279 × 109/L (150–450)
  • Alkaline phosphatase 387 U/L (45–115)
  • Total bilirubin 0.9 mg/dL (0.1–1.0)
  • Aspartate aminotransferase (AST) 35 U/L (35–48)
  • Alanine aminotransferase (ALT) 30 U/L (7–55).

Figure 1. Intraoperative cholangiography demonstrates annular, multifocal stricturing and beading of the extrahepatic biliary system (arrow).
His physician is concerned about his elevated alkaline phosphatase level, which can be a sign of cholestatic liver disease (ie, involving blockage of the flow of bile). He sends him for ultrasonography, which reveals mild thickening of the gallbladder wall. The patient is referred to a general surgeon, who decides to remove the gallbladder. The procedure goes well, but when contrast dye is injected into the biliary system during cholangiography, the image is markedly abnormal (Figure 1). The patient is referred to Mayo Clinic for further evaluation.

WHAT IS THE DIAGNOSIS?

1. Based on this information, which of the following is the most likely diagnosis?

  • Autoimmune hepatitis
  • Primary sclerosing cholangitis
  • Primary biliary cirrhosis
  • Idiopathic adulthood ductopenia

Primary sclerosing cholangitis

The most likely diagnosis is primary sclerosing cholangitis, a chronic cholestatic liver disease characterized by diffuse inflammatory destruction of intrahepatic and extrahepatic bile ducts, resulting in fibrosis, cirrhosis, and liver failure. Its cause is unknown, but it is likely the result of acquired exposures interacting with predisposing host factors. Current diagnostic criteria include:

  • Characteristic cholangiographic abnormalities of the biliary tree
  • Compatible clinical and biochemical findings (typically cholestasis with elevated alkaline phosphatase levels for at least 6 months)
  • Exclusion of causes of secondary sclerosing cholangitis: secondary sclerosing cholangitis is characterized by a similar multifocal biliary stricturing process, but with an identifiable cause such as long-term biliary obstruction, surgical biliary trauma, or recurrent pancreatitis.1

At presentation, the most common liver enzyme abnormality is an elevated alkaline phosphatase level, often three or four times the normal level.2 In contrast, aminotransferase levels are only modestly elevated, less than three times the upper limit of normal.3 At the time of diagnosis, serum bilirubin levels are normal in 60% of patients.4

Two large epidemiologic studies (one from Olmsted County, MN,5 the other from Swansea, Wales, UK6) estimated the age-adjusted incidence of primary sclerosing cholangitis to be 0.9 per 100,000 individuals. The median age of the patients at onset was in the 30s or 40s, and most were men. At 10 years, an estimated 65% were still alive and had not undergone liver transplantation—a significantly lower percentage than in age- and sex-matched populations.

It is estimated that more than 70% of patients with primary sclerosing cholangitis also have inflammatory bowel disease.5 In fact, the most common presentation of primary sclerosing cholangitis is asymptomatic inflammatory bowel disease and persistently elevated alkaline phosphatase—usually first noted on routine biochemical screening, as in our patient.

Imaging of the biliary tree is essential for the diagnosis of primary sclerosing cholangitis. Typical findings on cholangiography include multifocal stricturing and beading, usually involving both the intrahepatic and the extrahepatic biliary systems, as in our patient (Figure 1). Endoscopic retrograde cholangiopancreatography (ERCP) is considered the gold standard imaging test, but recent studies have shown that magnetic resonance cholangiopancreatography (MRCP) is an acceptable noninvasive substitute,7 and it may cost less per diagnosis.8

Liver biopsy alone is generally nondiagnostic because the histologic changes are quite variable in different segments of the same liver. The classic “onion-skin fibrosis” of primary sclerosing cholangitis is seen in fewer than 10% of biopsy specimens.9

Autoimmune hepatitis

Autoimmune hepatitis is chronic and is characterized by circulating autoantibodies and high serum globulin concentrations.10 Its presentation is heterogeneous, varying from no symptoms to nonspecific symptoms of malaise, fatigue, abdominal pain, itching, and arthralgia. Generally, elevations in aminotransferases are much more prominent than abnormalities in bilirubin and alkaline phosphatase levels10—unlike the pattern in our patient.

Primary biliary cirrhosis

Primary biliary cirrhosis is diagnosed if the patient has at least two of these three clinical criteria:

  • Biochemical evidence of cholestasis, with elevation of alkaline phosphatase for at least 6 months
  • Antimitochondrial antibody
  • Histologic evidence of nonsuppurative cholangitis and destruction of small or medium-sized bile ducts.11

In patients who lack antimitochondrial antibody, liver biopsy is necessary to establish the diagnosis. Given that primary biliary cirrhosis involves only small and medium-sized bile ducts, cholangiography is usually normal unless the patient has advanced cirrhosis.

Idiopathic adulthood ductopenia

Idiopathic adulthood ductopenia is a rare condition of unknown cause that involves the progressive destruction of segments of the small bile ducts inside the liver (“small-duct” biliary disease).12 Laboratory findings reveal a cholestatic pattern of liver injury, but biopsy samples show no features diagnostic or suggestive of another biliary disease; cholangiography is typically normal.12,13

 

 

ASSOCIATION WITH INFLAMMATORY BOWEL DISEASE

2. Which statement best characterizes inflammatory bowel disease associated with primary sclerosing cholangitis?

  • Crohn disease of the small bowel is the most common form
  • Liver disease often precedes the bowel disease
  • Treating the underlying bowel disease improves the long-term prognosis for the liver condition
  • Patients with primary sclerosing cholangitis and chronic ulcerative colitis are at higher risk of colonic dysplasia than patients with chronic ulcerative colitis alone

From 70% to 80% of patients with primary sclerosing cholangitis also have inflammatory bowel disease, usually chronic ulcerative colitis.14,15 Conversely, 2.4% to 4% of patients with ulcerative colitis and 1.4% to 3.4% of patients with Crohn disease have primary sclerosing cholangitis.1

Typically, the diagnosis of inflammatory bowel disease is made 8 to 10 years before the diagnosis of liver disease, although cases have also been reported to occur years after the diagnosis of cholangitis.15,16

No association between the severity of bowel disease and liver disease has been reported, and treating the inflammatory bowel disease does not alter the natural history of primary sclerosing cholangitis. Particularly, proctocolectomy, the most aggressive treatment for chronic ulcerative colitis, appears to have no effect on the course of the cholangitis.17

In patients with both primary sclerosing cholangitis and chronic ulcerative colitis, the risk of colonic dysplasia is higher than in patients with chronic ulcerative colitis alone.18 Recent studies have predicted that the risk of colorectal carcinoma in patients with primary sclerosing cholangitis and inflammatory bowel disease is as high as 25% after 10 years.19,20 Therefore, annual colonoscopy with surveillance biopsy is recommended in patients with both primary sclerosing cholangitis and chronic ulcerative colitis, since screening and early detection improve survival rates.15

TREATMENT AND PROGNOSIS

After being diagnosed with primary sclerosing cholangitis, the patient inquires about ongoing medical therapy and long-term prognosis.

3. Which is the only life-prolonging therapy for primary sclerosing cholangitis?

  • Methotrexate (Trexall)
  • Ursodeoxycholic acid (UDCA) (Actigall) at a standard dosage (13–15 mg/kg/day)
  • UDCA at a high dosage (20–30 mg/kg/day)
  • Liver transplantation

Drug therapy has not been shown to improve the prognosis of primary sclerosing cholangitis.

In randomized placebo-controlled trials, penicillamine (Depen), colchicine (Colcrys), methotrexate, and UDCA (13–15 mg/kg per day) failed to show efficacy.21–23

In pilot studies, high-dose UDCA (20 to 30 mg/kg/day) initially appeared to bring an improvement in survival probability, with trends toward histologic improvement,24,25 but larger randomized placebo-controlled trials found no improvement in symptoms, quality of life, survival rates, or risk of cholangiocarcinoma with high-dose UDCA.26,27 In fact, in 5 years of follow-up, patients on high-dose UDCA had a risk of death or transplantation two times higher than with placebo.27 One study indicated UDCA may decrease the incidence of colonic dysplasia in patients with primary sclerosing cholangitis and chronic ulcerative colitis.28 However, more prospective studies are required to better define the routine use of UDCA as a prophylactic agent.

Liver transplantation remains the most effective treatment for primary sclerosing cholangitis, and it improves the rate of survival.29 Nevertheless, about 20% of patients who undergo transplantation have a recurrence of cholangitis, and it may recur earlier after living-donor liver transplantation, particularly when the graft is from a biologically related donor.30 Proposed risk factors for recurrence include inflammatory bowel disease, prolonged ischemia time, the number of cellular rejection events, prior biliary surgery, cytomegalovirus infection, and lymphocytotoxic cross-match.31

4. In addition to cirrhosis and cholangitis, which of the following is a potential long-term complication of primary sclerosing cholangitis?

  • Colon cancer
  • Cholangiocarcinoma
  • Osteoporosis
  • Fat-soluble vitamin deficiency
  • All of the above

All are potential long-term complications.

Colon cancer. Concomitant chronic ulcerative colitis puts the patient at a higher risk of colonic dysplasia compared with patients with chronic ulcerative colitis alone.18 According to recent studies of patients with primary sclerosing cholangitis and inflammatory bowel disease, 19,20 the risk of colorectal carcinoma after 10 years of disease is as high as 25%.

Cholangiocarcinoma. Primary sclerosing cholangitis is considered a risk factor for cholangiocarcinoma, with an estimated 10-year cumulative incidence of 7% to 9%.1,20 In a retrospective study of 30 patients,32 the median survival was 5 months from the time of diagnosis of cholangiocarcinoma; at the time of diagnosis approximately 19 patients (63%) had metastatic disease.

At present, early detection of cholangiocarcinoma is hampered by the low sensitivity and specificity of standard diagnostic approaches. Carbohydrate antigen 19-9 has been used as a marker, but it has questionable accuracy, since elevations of this antigen can also be a result of pancreatic malignancy and bacterial cholangitis. However, cholangiocarcinoma should be suspected when patients present with progressive jaundice, weight loss, abdominal discomfort, and a sudden rise in carbohydrate antigen 19-9.

Conventional ultrasonography and computed tomography (CT) have poor sensitivity for detecting this malignancy. ERCP with biliary brushings should be considered when evaluating for biliary malignancy. New diagnostic methods such as digitized image analysis and fluorescence in situ hybridization on biliary brushings offer promise to evaluate bile duct lesions for cellular aneuploidy and chromosomal aberrations, which may improve the detection of cholangiocarcinoma.33 A recent large-scale study of nearly 500 patients showed that fluorescence in situ hybridization had a higher sensitivity (42.9%) than routine cytology (20.1%) with identical specificity (99.6%) for malignancy.34

Metabolic bone disease, usually osteoporosis rather than osteomalacia, is relatively common and is an important complication of primary sclerosing cholangitis.35 Patients with osteoporosis should be treated with vitamin D and calcium supplementation. Bisphosphonates have been used with varying results in primary biliary cirrhosis36 and can be considered in patients with advanced osteoporosis.

Fat-soluble vitamin deficiency is relatively common in primary sclerosing cholangitis, particularly as it progresses to advanced liver disease. Up to 40% of patients have vitamin A deficiency, 14% have vitamin D deficiency, and 2% have vitamin E deficiency.37 Patients can undergo simple oral replacement therapy.

 

 

A stone is removed, fever develops

Three years after the diagnosis of primary sclerosing cholangitis, the patient develops mild hyperbilirubinemia and undergoes ERCP at his local hospital. A stone is found obstructing the common bile duct and is successfully extracted.

Twenty-four hours after this procedure, he develops severe right-upper-quadrant pain and fever. He is seen at his local emergency department and blood cultures are drawn. He is started on antibiotics and is transferred to Mayo Clinic for further management.

5. In addition to continuing a broad-spectrum antibiotic, which would be the next best step for this patient?

  • ERCP
  • MRCP
  • Abdominal ultrasonography
  • Abdominal CT

The patient’s clinical presentation is consistent with acute bacterial cholangitis. The classic Charcot triad of fever, right-upper-quadrant pain, and jaundice occurs in only 50% to 75% of patients with acute cholangitis.38 In addition to receiving a broad-spectrum antibiotic, patients with bacterial cholangitis require emergency endoscopic evaluation—ERCP—to find and remove stones from the bile ducts and, if necessary, to dilate the biliary strictures to allow adequate drainage.

In our experience, more than 10% of patients with primary sclerosing cholangitis who undergo ERCP develop complications requiring hospitalization.39 The procedure generally takes longer to perform and the incidence of cholangitis is higher, despite routine antibiotic prophylaxis, in patients with primary sclerosing cholangitis than in those without it. However, the overall risk of pancreatitis, perforation, and bleeding was similar in patients with or without sclerosing cholangitis.39

MRCP is a promising noninvasive substitute for ERCP in establishing the diagnosis of primary sclerosing cholangitis.7,8 Unfortunately, as with other noninvasive imaging studies such as abdominal ultrasonography and CT, MRCP does not allow for therapeutic biliary decompression.

The patient undergoes ERCP with stenting

The patient’s acute cholangitis is thought to be a complication of his recent ERCP procedure. He undergoes emergency ERCP with balloon dilation and placement of a temporary left hepatic stent. His fever improves and he is discharged 48 hours later. He completes a 14-day course of antibiotics for Enterococcus faecalis bacteremia. Six weeks later, he undergoes ERCP yet again to remove the stent and tolerates the procedure well without complications.

TAKE-HOME POINTS

  • Primary sclerosing cholangitis is a progressive cholestatic liver disease of unknown etiology that primarily affects men during the fourth decade of life.
  • This condition is strongly associated with inflammatory bowel disease, particularly with ulcerative colitis.
  • Cholangiocarcinoma and colon cancer are dreaded complications.
  • Liver transplantation is the only life-extending therapy for primary sclerosing cholangitis; however, the condition can recur in the allograft.

A 49-year-old man has had ulcerative colitis for more than 30 years. It is well controlled with sulfasalazine (Azulfidine). Now, he has come to see his primary care physician because for the past 3 months he has had mild, intermittent pain in his right upper abdominal quadrant.

His physical examination is normal. Routine laboratory testing shows the following:

  • Hemoglobin 14.2 g/dL (reference range 13.5–17.5)
  • White blood cell count 6.7 × 109/L (3.5–10.5)
  • Platelet count 279 × 109/L (150–450)
  • Alkaline phosphatase 387 U/L (45–115)
  • Total bilirubin 0.9 mg/dL (0.1–1.0)
  • Aspartate aminotransferase (AST) 35 U/L (35–48)
  • Alanine aminotransferase (ALT) 30 U/L (7–55).

Figure 1. Intraoperative cholangiography demonstrates annular, multifocal stricturing and beading of the extrahepatic biliary system (arrow).
His physician is concerned about his elevated alkaline phosphatase level, which can be a sign of cholestatic liver disease (ie, involving blockage of the flow of bile). He sends him for ultrasonography, which reveals mild thickening of the gallbladder wall. The patient is referred to a general surgeon, who decides to remove the gallbladder. The procedure goes well, but when contrast dye is injected into the biliary system during cholangiography, the image is markedly abnormal (Figure 1). The patient is referred to Mayo Clinic for further evaluation.

WHAT IS THE DIAGNOSIS?

1. Based on this information, which of the following is the most likely diagnosis?

  • Autoimmune hepatitis
  • Primary sclerosing cholangitis
  • Primary biliary cirrhosis
  • Idiopathic adulthood ductopenia

Primary sclerosing cholangitis

The most likely diagnosis is primary sclerosing cholangitis, a chronic cholestatic liver disease characterized by diffuse inflammatory destruction of intrahepatic and extrahepatic bile ducts, resulting in fibrosis, cirrhosis, and liver failure. Its cause is unknown, but it is likely the result of acquired exposures interacting with predisposing host factors. Current diagnostic criteria include:

  • Characteristic cholangiographic abnormalities of the biliary tree
  • Compatible clinical and biochemical findings (typically cholestasis with elevated alkaline phosphatase levels for at least 6 months)
  • Exclusion of causes of secondary sclerosing cholangitis: secondary sclerosing cholangitis is characterized by a similar multifocal biliary stricturing process, but with an identifiable cause such as long-term biliary obstruction, surgical biliary trauma, or recurrent pancreatitis.1

At presentation, the most common liver enzyme abnormality is an elevated alkaline phosphatase level, often three or four times the normal level.2 In contrast, aminotransferase levels are only modestly elevated, less than three times the upper limit of normal.3 At the time of diagnosis, serum bilirubin levels are normal in 60% of patients.4

Two large epidemiologic studies (one from Olmsted County, MN,5 the other from Swansea, Wales, UK6) estimated the age-adjusted incidence of primary sclerosing cholangitis to be 0.9 per 100,000 individuals. The median age of the patients at onset was in the 30s or 40s, and most were men. At 10 years, an estimated 65% were still alive and had not undergone liver transplantation—a significantly lower percentage than in age- and sex-matched populations.

It is estimated that more than 70% of patients with primary sclerosing cholangitis also have inflammatory bowel disease.5 In fact, the most common presentation of primary sclerosing cholangitis is asymptomatic inflammatory bowel disease and persistently elevated alkaline phosphatase—usually first noted on routine biochemical screening, as in our patient.

Imaging of the biliary tree is essential for the diagnosis of primary sclerosing cholangitis. Typical findings on cholangiography include multifocal stricturing and beading, usually involving both the intrahepatic and the extrahepatic biliary systems, as in our patient (Figure 1). Endoscopic retrograde cholangiopancreatography (ERCP) is considered the gold standard imaging test, but recent studies have shown that magnetic resonance cholangiopancreatography (MRCP) is an acceptable noninvasive substitute,7 and it may cost less per diagnosis.8

Liver biopsy alone is generally nondiagnostic because the histologic changes are quite variable in different segments of the same liver. The classic “onion-skin fibrosis” of primary sclerosing cholangitis is seen in fewer than 10% of biopsy specimens.9

Autoimmune hepatitis

Autoimmune hepatitis is chronic and is characterized by circulating autoantibodies and high serum globulin concentrations.10 Its presentation is heterogeneous, varying from no symptoms to nonspecific symptoms of malaise, fatigue, abdominal pain, itching, and arthralgia. Generally, elevations in aminotransferases are much more prominent than abnormalities in bilirubin and alkaline phosphatase levels10—unlike the pattern in our patient.

Primary biliary cirrhosis

Primary biliary cirrhosis is diagnosed if the patient has at least two of these three clinical criteria:

  • Biochemical evidence of cholestasis, with elevation of alkaline phosphatase for at least 6 months
  • Antimitochondrial antibody
  • Histologic evidence of nonsuppurative cholangitis and destruction of small or medium-sized bile ducts.11

In patients who lack antimitochondrial antibody, liver biopsy is necessary to establish the diagnosis. Given that primary biliary cirrhosis involves only small and medium-sized bile ducts, cholangiography is usually normal unless the patient has advanced cirrhosis.

Idiopathic adulthood ductopenia

Idiopathic adulthood ductopenia is a rare condition of unknown cause that involves the progressive destruction of segments of the small bile ducts inside the liver (“small-duct” biliary disease).12 Laboratory findings reveal a cholestatic pattern of liver injury, but biopsy samples show no features diagnostic or suggestive of another biliary disease; cholangiography is typically normal.12,13

 

 

ASSOCIATION WITH INFLAMMATORY BOWEL DISEASE

2. Which statement best characterizes inflammatory bowel disease associated with primary sclerosing cholangitis?

  • Crohn disease of the small bowel is the most common form
  • Liver disease often precedes the bowel disease
  • Treating the underlying bowel disease improves the long-term prognosis for the liver condition
  • Patients with primary sclerosing cholangitis and chronic ulcerative colitis are at higher risk of colonic dysplasia than patients with chronic ulcerative colitis alone

From 70% to 80% of patients with primary sclerosing cholangitis also have inflammatory bowel disease, usually chronic ulcerative colitis.14,15 Conversely, 2.4% to 4% of patients with ulcerative colitis and 1.4% to 3.4% of patients with Crohn disease have primary sclerosing cholangitis.1

Typically, the diagnosis of inflammatory bowel disease is made 8 to 10 years before the diagnosis of liver disease, although cases have also been reported to occur years after the diagnosis of cholangitis.15,16

No association between the severity of bowel disease and liver disease has been reported, and treating the inflammatory bowel disease does not alter the natural history of primary sclerosing cholangitis. Particularly, proctocolectomy, the most aggressive treatment for chronic ulcerative colitis, appears to have no effect on the course of the cholangitis.17

In patients with both primary sclerosing cholangitis and chronic ulcerative colitis, the risk of colonic dysplasia is higher than in patients with chronic ulcerative colitis alone.18 Recent studies have predicted that the risk of colorectal carcinoma in patients with primary sclerosing cholangitis and inflammatory bowel disease is as high as 25% after 10 years.19,20 Therefore, annual colonoscopy with surveillance biopsy is recommended in patients with both primary sclerosing cholangitis and chronic ulcerative colitis, since screening and early detection improve survival rates.15

TREATMENT AND PROGNOSIS

After being diagnosed with primary sclerosing cholangitis, the patient inquires about ongoing medical therapy and long-term prognosis.

3. Which is the only life-prolonging therapy for primary sclerosing cholangitis?

  • Methotrexate (Trexall)
  • Ursodeoxycholic acid (UDCA) (Actigall) at a standard dosage (13–15 mg/kg/day)
  • UDCA at a high dosage (20–30 mg/kg/day)
  • Liver transplantation

Drug therapy has not been shown to improve the prognosis of primary sclerosing cholangitis.

In randomized placebo-controlled trials, penicillamine (Depen), colchicine (Colcrys), methotrexate, and UDCA (13–15 mg/kg per day) failed to show efficacy.21–23

In pilot studies, high-dose UDCA (20 to 30 mg/kg/day) initially appeared to bring an improvement in survival probability, with trends toward histologic improvement,24,25 but larger randomized placebo-controlled trials found no improvement in symptoms, quality of life, survival rates, or risk of cholangiocarcinoma with high-dose UDCA.26,27 In fact, in 5 years of follow-up, patients on high-dose UDCA had a risk of death or transplantation two times higher than with placebo.27 One study indicated UDCA may decrease the incidence of colonic dysplasia in patients with primary sclerosing cholangitis and chronic ulcerative colitis.28 However, more prospective studies are required to better define the routine use of UDCA as a prophylactic agent.

Liver transplantation remains the most effective treatment for primary sclerosing cholangitis, and it improves the rate of survival.29 Nevertheless, about 20% of patients who undergo transplantation have a recurrence of cholangitis, and it may recur earlier after living-donor liver transplantation, particularly when the graft is from a biologically related donor.30 Proposed risk factors for recurrence include inflammatory bowel disease, prolonged ischemia time, the number of cellular rejection events, prior biliary surgery, cytomegalovirus infection, and lymphocytotoxic cross-match.31

4. In addition to cirrhosis and cholangitis, which of the following is a potential long-term complication of primary sclerosing cholangitis?

  • Colon cancer
  • Cholangiocarcinoma
  • Osteoporosis
  • Fat-soluble vitamin deficiency
  • All of the above

All are potential long-term complications.

Colon cancer. Concomitant chronic ulcerative colitis puts the patient at a higher risk of colonic dysplasia compared with patients with chronic ulcerative colitis alone.18 According to recent studies of patients with primary sclerosing cholangitis and inflammatory bowel disease, 19,20 the risk of colorectal carcinoma after 10 years of disease is as high as 25%.

Cholangiocarcinoma. Primary sclerosing cholangitis is considered a risk factor for cholangiocarcinoma, with an estimated 10-year cumulative incidence of 7% to 9%.1,20 In a retrospective study of 30 patients,32 the median survival was 5 months from the time of diagnosis of cholangiocarcinoma; at the time of diagnosis approximately 19 patients (63%) had metastatic disease.

At present, early detection of cholangiocarcinoma is hampered by the low sensitivity and specificity of standard diagnostic approaches. Carbohydrate antigen 19-9 has been used as a marker, but it has questionable accuracy, since elevations of this antigen can also be a result of pancreatic malignancy and bacterial cholangitis. However, cholangiocarcinoma should be suspected when patients present with progressive jaundice, weight loss, abdominal discomfort, and a sudden rise in carbohydrate antigen 19-9.

Conventional ultrasonography and computed tomography (CT) have poor sensitivity for detecting this malignancy. ERCP with biliary brushings should be considered when evaluating for biliary malignancy. New diagnostic methods such as digitized image analysis and fluorescence in situ hybridization on biliary brushings offer promise to evaluate bile duct lesions for cellular aneuploidy and chromosomal aberrations, which may improve the detection of cholangiocarcinoma.33 A recent large-scale study of nearly 500 patients showed that fluorescence in situ hybridization had a higher sensitivity (42.9%) than routine cytology (20.1%) with identical specificity (99.6%) for malignancy.34

Metabolic bone disease, usually osteoporosis rather than osteomalacia, is relatively common and is an important complication of primary sclerosing cholangitis.35 Patients with osteoporosis should be treated with vitamin D and calcium supplementation. Bisphosphonates have been used with varying results in primary biliary cirrhosis36 and can be considered in patients with advanced osteoporosis.

Fat-soluble vitamin deficiency is relatively common in primary sclerosing cholangitis, particularly as it progresses to advanced liver disease. Up to 40% of patients have vitamin A deficiency, 14% have vitamin D deficiency, and 2% have vitamin E deficiency.37 Patients can undergo simple oral replacement therapy.

 

 

A stone is removed, fever develops

Three years after the diagnosis of primary sclerosing cholangitis, the patient develops mild hyperbilirubinemia and undergoes ERCP at his local hospital. A stone is found obstructing the common bile duct and is successfully extracted.

Twenty-four hours after this procedure, he develops severe right-upper-quadrant pain and fever. He is seen at his local emergency department and blood cultures are drawn. He is started on antibiotics and is transferred to Mayo Clinic for further management.

5. In addition to continuing a broad-spectrum antibiotic, which would be the next best step for this patient?

  • ERCP
  • MRCP
  • Abdominal ultrasonography
  • Abdominal CT

The patient’s clinical presentation is consistent with acute bacterial cholangitis. The classic Charcot triad of fever, right-upper-quadrant pain, and jaundice occurs in only 50% to 75% of patients with acute cholangitis.38 In addition to receiving a broad-spectrum antibiotic, patients with bacterial cholangitis require emergency endoscopic evaluation—ERCP—to find and remove stones from the bile ducts and, if necessary, to dilate the biliary strictures to allow adequate drainage.

In our experience, more than 10% of patients with primary sclerosing cholangitis who undergo ERCP develop complications requiring hospitalization.39 The procedure generally takes longer to perform and the incidence of cholangitis is higher, despite routine antibiotic prophylaxis, in patients with primary sclerosing cholangitis than in those without it. However, the overall risk of pancreatitis, perforation, and bleeding was similar in patients with or without sclerosing cholangitis.39

MRCP is a promising noninvasive substitute for ERCP in establishing the diagnosis of primary sclerosing cholangitis.7,8 Unfortunately, as with other noninvasive imaging studies such as abdominal ultrasonography and CT, MRCP does not allow for therapeutic biliary decompression.

The patient undergoes ERCP with stenting

The patient’s acute cholangitis is thought to be a complication of his recent ERCP procedure. He undergoes emergency ERCP with balloon dilation and placement of a temporary left hepatic stent. His fever improves and he is discharged 48 hours later. He completes a 14-day course of antibiotics for Enterococcus faecalis bacteremia. Six weeks later, he undergoes ERCP yet again to remove the stent and tolerates the procedure well without complications.

TAKE-HOME POINTS

  • Primary sclerosing cholangitis is a progressive cholestatic liver disease of unknown etiology that primarily affects men during the fourth decade of life.
  • This condition is strongly associated with inflammatory bowel disease, particularly with ulcerative colitis.
  • Cholangiocarcinoma and colon cancer are dreaded complications.
  • Liver transplantation is the only life-extending therapy for primary sclerosing cholangitis; however, the condition can recur in the allograft.
References
  1. Chapman R, Fevery J, Kalloo A, et al; American Association for the Study of Liver Diseases. Diagnosis and management of primary sclerosing cholangitis. Hepatology 2010; 51:660678.
  2. Silveira MG, Lindor KD. Clinical features and management of primary sclerosing cholangitis. World J Gastroenterol 2008; 14:33383349.
  3. Lee YM, Kaplan MM. Primary sclerosing cholangitis. N Engl J Med 1995; 332:924933.
  4. Talwalkar JA, Lindor KD. Primary sclerosing cholangitis. Inflamm Bowel Dis 2005; 11:6272.
  5. Bambha K, Kim WR, Talwalkar J, et al. Incidence, clinical spectrum, and outcomes of primary sclerosing cholangitis in a United States community. Gastroenterology 2003; 125:13641369.
  6. Kingham JG, Kochar N, Gravenor MB. Incidence, clinical patterns, and outcomes of primary sclerosing cholangitis in South Wales, United Kingdom. Gastroenterology 2004; 126:19291930.
  7. Berstad AE, Aabakken L, Smith HJ, Aasen S, Boberg KM, Schrumpf E. Diagnostic accuracy of magnetic resonance and endoscopic retrograde cholangiography in primary sclerosing cholangitis. Clin Gastroenterol Hepatol 2006; 4:514520.
  8. Talwalkar JA, Angulo P, Johnson CD, Petersen BT, Lindor KD. Cost-minimization analysis of MRC versus ERCP for the diagnosis of primary sclerosing cholangitis. Hepatology 2004; 40:3945.
  9. Ludwig J, Barham SS, LaRusso NF, Elveback LR, Wiesner RH, McCall JT. Morphologic features of chronic hepatitis associated with primary sclerosing cholangitis and chronic ulcerative colitis. Hepatology 1981; 1:632640.
  10. Krawitt EL. Autoimmune hepatitis. N Engl J Med 2006; 354:5466.
  11. Lindor KD, Gershwin ME, Poupon R, Kaplan M, Bergasa NV, Heathcote EJ; American Association for Study of Liver Diseases. Primary biliary cirrhosis. Hepatology 2009; 50:291308.
  12. Ludwig J, Wiesner RH, LaRusso NF. Idiopathic adulthood ductopenia. A cause of chronic cholestatic liver disease and biliary cirrhosis. J Hepatol 1988; 7:193199.
  13. Ludwig J. Idiopathic adulthood ductopenia: an update. Mayo Clin Proc 1998; 73:285291.
  14. Fausa O, Schrumpf E, Elgjo K. Relationship of inflammatory bowel disease and primary sclerosing cholangitis. Semin Liver Dis 1991; 11:3139.
  15. Loftus EV, Aguilar HI, Sandborn WJ, et al. Risk of colorectal neoplasia in patients with primary sclerosing cholangitis and ulcerative colitis following orthotopic liver transplantation. Hepatology 1998; 27:685690.
  16. Loftus EV, Sandborn WJ, Tremaine WJ, et al. Risk of colorectal neoplasia in patients with primary sclerosing cholangitis. Gastroenterology 1996; 110:432440.
  17. Cangemi JR, Wiesner RH, Beaver SJ, et al. Effect of proctocolectomy for chronic ulcerative colitis on the natural history of primary sclerosing cholangitis. Gastroenterology 1989; 96:790794.
  18. Broomé U, Löfberg R, Veress B, Eriksson LS. Primary sclerosing cholangitis and ulcerative colitis: evidence for increased neoplastic potential. Hepatology 1995; 22:14041408.
  19. Kornfeld D, Ekbom A, Ihre T. Is there an excess risk for colorectal cancer in patients with ulcerative colitis and concomitant primary sclerosing cholangitis? A population based study. Gut 1997; 41:522525.
  20. Claessen MM, Vleggaar FP, Tytgat KM, Siersema PD, van Buuren HR. High lifetime risk of cancer in primary sclerosing cholangitis. J Hepatol 2009; 50:158164.
  21. Lindor KD. Ursodiol for primary sclerosing cholangitis. Mayo Primary Sclerosing Cholangitis-Ursodeoxycholic Acid Study Group. N Engl J Med 1997; 336:691695.
  22. Olsson R, Broomé U, Danielsson A, et al. Colchicine treatment of primary sclerosing cholangitis. Gastroenterology 1995; 108:11991203.
  23. LaRusso NF, Wiesner RH, Ludwig J, MacCarty RL, Beaver SJ, Zinsmeister AR. Prospective trial of penicillamine in primary sclerosing cholangitis. Gastroenterology 1988; 95:10361042.
  24. Mitchell SA, Bansi DS, Hunt N, Von Bergmann K, Fleming KA, Chapman RW. A preliminary trial of high-dose ursodeoxycholic acid in primary sclerosing cholangitis. Gastroenterology 2001; 121:900907.
  25. Cullen SN, Rust C, Fleming K, Edwards C, Beuers U, Chapman RW. High dose ursodeoxycholic acid for the treatment of primary sclerosing cholangitis is safe and effective. J Hepatol 2008; 48:792800.
  26. Olsson R, Boberg KM, de Muckadell OS, et al. High-dose ursodeoxycholic acid in primary sclerosing cholangitis: a 5-year multicenter, randomized, controlled study. Gastroenterology 2005; 129:14641472.
  27. Lindor KD, Kowdley KV, Luketic VA, et al. High-dose ursodeoxycholic acid for the treatment of primary sclerosing cholangitis. Hepatology 2009; 50:808814.
  28. Tung BY, Emond MJ, Haggitt RC, et al. Ursodiol use is associated with lower prevalence of colonic neoplasia in patients with ulcerative colitis and primary sclerosing cholangitis. Ann Intern Med 2001; 134:8995.
  29. Wiesner RH, Porayko MK, Hay JE, et al. Liver transplantation for primary sclerosing cholangitis: impact of risk factors on outcome. Liver Transpl Surg 1996; 2(suppl 1):99108..
  30. Tamura S, Sugawara Y, Kaneko J, Matsui Y, Togashi J, Makuuchi M. Recurrence of primary sclerosing cholangitis after living donor liver transplantation. Liver Int 2007; 27:8694.
  31. Gautam M, Cheruvattath R, Balan V. Recurrence of autoimmune liver disease after liver transplantation: a systematic review. Liver Transpl 2006; 12:18131824.
  32. Rosen CB, Nagorney DM, Wiesner RH, Coffey RJ, LaRusso NF. Cholangiocarcinoma complicating primary sclerosing cholangitis. Ann Surg 1991; 213:2125.
  33. Lazaridis KN, Gores GJ. Cholangiocarcinoma. Gastroenterology 2005; 128:16551667.
  34. Fritcher EG, Kipp BR, Halling KC, et al. A multivariable model using advanced cytologic methods for the evaluation of indeterminate pancreatobiliary strictures. Gastroenterology 2009; 136:21802186.
  35. Hay JE, Lindor KD, Wiesner RH, Dickson ER, Krom RA, LaRusso NF. The metabolic bone disease of primary sclerosing cholangitis. Hepatology 1991; 14:257261.
  36. Guañabens N, Parés A, Ros I, et al. Alendronate is more effective than etidronate for increasing bone mass in osteopenic patients with primary biliary cirrhosis. Am J Gastroenterol 2003; 98:22682274.
  37. Jorgensen RA, Lindor KD, Sartin JS, LaRusso NF, Wiesner RH. Serum lipid and fat-soluble vitamin levels in primary sclerosing cholangitis. J Clin Gastroenterol 1995; 20:215219.
  38. Saik RP, Greenburg AG, Farris JM, Peskin GW. Spectrum of cholangitis. Am J Surg 1975; 130:143150.
  39. Bangarulingam SY, Gossard AA, Petersen BT, Ott BJ, Lindor KD. Complications of endoscopic retrograde cholangiopancreatography in primary sclerosing cholangitis. Am J Gastroenterol 2009; 104:855860.
References
  1. Chapman R, Fevery J, Kalloo A, et al; American Association for the Study of Liver Diseases. Diagnosis and management of primary sclerosing cholangitis. Hepatology 2010; 51:660678.
  2. Silveira MG, Lindor KD. Clinical features and management of primary sclerosing cholangitis. World J Gastroenterol 2008; 14:33383349.
  3. Lee YM, Kaplan MM. Primary sclerosing cholangitis. N Engl J Med 1995; 332:924933.
  4. Talwalkar JA, Lindor KD. Primary sclerosing cholangitis. Inflamm Bowel Dis 2005; 11:6272.
  5. Bambha K, Kim WR, Talwalkar J, et al. Incidence, clinical spectrum, and outcomes of primary sclerosing cholangitis in a United States community. Gastroenterology 2003; 125:13641369.
  6. Kingham JG, Kochar N, Gravenor MB. Incidence, clinical patterns, and outcomes of primary sclerosing cholangitis in South Wales, United Kingdom. Gastroenterology 2004; 126:19291930.
  7. Berstad AE, Aabakken L, Smith HJ, Aasen S, Boberg KM, Schrumpf E. Diagnostic accuracy of magnetic resonance and endoscopic retrograde cholangiography in primary sclerosing cholangitis. Clin Gastroenterol Hepatol 2006; 4:514520.
  8. Talwalkar JA, Angulo P, Johnson CD, Petersen BT, Lindor KD. Cost-minimization analysis of MRC versus ERCP for the diagnosis of primary sclerosing cholangitis. Hepatology 2004; 40:3945.
  9. Ludwig J, Barham SS, LaRusso NF, Elveback LR, Wiesner RH, McCall JT. Morphologic features of chronic hepatitis associated with primary sclerosing cholangitis and chronic ulcerative colitis. Hepatology 1981; 1:632640.
  10. Krawitt EL. Autoimmune hepatitis. N Engl J Med 2006; 354:5466.
  11. Lindor KD, Gershwin ME, Poupon R, Kaplan M, Bergasa NV, Heathcote EJ; American Association for Study of Liver Diseases. Primary biliary cirrhosis. Hepatology 2009; 50:291308.
  12. Ludwig J, Wiesner RH, LaRusso NF. Idiopathic adulthood ductopenia. A cause of chronic cholestatic liver disease and biliary cirrhosis. J Hepatol 1988; 7:193199.
  13. Ludwig J. Idiopathic adulthood ductopenia: an update. Mayo Clin Proc 1998; 73:285291.
  14. Fausa O, Schrumpf E, Elgjo K. Relationship of inflammatory bowel disease and primary sclerosing cholangitis. Semin Liver Dis 1991; 11:3139.
  15. Loftus EV, Aguilar HI, Sandborn WJ, et al. Risk of colorectal neoplasia in patients with primary sclerosing cholangitis and ulcerative colitis following orthotopic liver transplantation. Hepatology 1998; 27:685690.
  16. Loftus EV, Sandborn WJ, Tremaine WJ, et al. Risk of colorectal neoplasia in patients with primary sclerosing cholangitis. Gastroenterology 1996; 110:432440.
  17. Cangemi JR, Wiesner RH, Beaver SJ, et al. Effect of proctocolectomy for chronic ulcerative colitis on the natural history of primary sclerosing cholangitis. Gastroenterology 1989; 96:790794.
  18. Broomé U, Löfberg R, Veress B, Eriksson LS. Primary sclerosing cholangitis and ulcerative colitis: evidence for increased neoplastic potential. Hepatology 1995; 22:14041408.
  19. Kornfeld D, Ekbom A, Ihre T. Is there an excess risk for colorectal cancer in patients with ulcerative colitis and concomitant primary sclerosing cholangitis? A population based study. Gut 1997; 41:522525.
  20. Claessen MM, Vleggaar FP, Tytgat KM, Siersema PD, van Buuren HR. High lifetime risk of cancer in primary sclerosing cholangitis. J Hepatol 2009; 50:158164.
  21. Lindor KD. Ursodiol for primary sclerosing cholangitis. Mayo Primary Sclerosing Cholangitis-Ursodeoxycholic Acid Study Group. N Engl J Med 1997; 336:691695.
  22. Olsson R, Broomé U, Danielsson A, et al. Colchicine treatment of primary sclerosing cholangitis. Gastroenterology 1995; 108:11991203.
  23. LaRusso NF, Wiesner RH, Ludwig J, MacCarty RL, Beaver SJ, Zinsmeister AR. Prospective trial of penicillamine in primary sclerosing cholangitis. Gastroenterology 1988; 95:10361042.
  24. Mitchell SA, Bansi DS, Hunt N, Von Bergmann K, Fleming KA, Chapman RW. A preliminary trial of high-dose ursodeoxycholic acid in primary sclerosing cholangitis. Gastroenterology 2001; 121:900907.
  25. Cullen SN, Rust C, Fleming K, Edwards C, Beuers U, Chapman RW. High dose ursodeoxycholic acid for the treatment of primary sclerosing cholangitis is safe and effective. J Hepatol 2008; 48:792800.
  26. Olsson R, Boberg KM, de Muckadell OS, et al. High-dose ursodeoxycholic acid in primary sclerosing cholangitis: a 5-year multicenter, randomized, controlled study. Gastroenterology 2005; 129:14641472.
  27. Lindor KD, Kowdley KV, Luketic VA, et al. High-dose ursodeoxycholic acid for the treatment of primary sclerosing cholangitis. Hepatology 2009; 50:808814.
  28. Tung BY, Emond MJ, Haggitt RC, et al. Ursodiol use is associated with lower prevalence of colonic neoplasia in patients with ulcerative colitis and primary sclerosing cholangitis. Ann Intern Med 2001; 134:8995.
  29. Wiesner RH, Porayko MK, Hay JE, et al. Liver transplantation for primary sclerosing cholangitis: impact of risk factors on outcome. Liver Transpl Surg 1996; 2(suppl 1):99108..
  30. Tamura S, Sugawara Y, Kaneko J, Matsui Y, Togashi J, Makuuchi M. Recurrence of primary sclerosing cholangitis after living donor liver transplantation. Liver Int 2007; 27:8694.
  31. Gautam M, Cheruvattath R, Balan V. Recurrence of autoimmune liver disease after liver transplantation: a systematic review. Liver Transpl 2006; 12:18131824.
  32. Rosen CB, Nagorney DM, Wiesner RH, Coffey RJ, LaRusso NF. Cholangiocarcinoma complicating primary sclerosing cholangitis. Ann Surg 1991; 213:2125.
  33. Lazaridis KN, Gores GJ. Cholangiocarcinoma. Gastroenterology 2005; 128:16551667.
  34. Fritcher EG, Kipp BR, Halling KC, et al. A multivariable model using advanced cytologic methods for the evaluation of indeterminate pancreatobiliary strictures. Gastroenterology 2009; 136:21802186.
  35. Hay JE, Lindor KD, Wiesner RH, Dickson ER, Krom RA, LaRusso NF. The metabolic bone disease of primary sclerosing cholangitis. Hepatology 1991; 14:257261.
  36. Guañabens N, Parés A, Ros I, et al. Alendronate is more effective than etidronate for increasing bone mass in osteopenic patients with primary biliary cirrhosis. Am J Gastroenterol 2003; 98:22682274.
  37. Jorgensen RA, Lindor KD, Sartin JS, LaRusso NF, Wiesner RH. Serum lipid and fat-soluble vitamin levels in primary sclerosing cholangitis. J Clin Gastroenterol 1995; 20:215219.
  38. Saik RP, Greenburg AG, Farris JM, Peskin GW. Spectrum of cholangitis. Am J Surg 1975; 130:143150.
  39. Bangarulingam SY, Gossard AA, Petersen BT, Ott BJ, Lindor KD. Complications of endoscopic retrograde cholangiopancreatography in primary sclerosing cholangitis. Am J Gastroenterol 2009; 104:855860.
Issue
Cleveland Clinic Journal of Medicine - 78(5)
Issue
Cleveland Clinic Journal of Medicine - 78(5)
Page Number
306-311
Page Number
306-311
Publications
Publications
Topics
Article Type
Display Headline
Ulcerative colitis and an abnormal cholangiogram
Display Headline
Ulcerative colitis and an abnormal cholangiogram
Sections
Disallow All Ads
Alternative CME
Article PDF Media

Facial swelling and ulceration with nasal destruction

Article Type
Changed
Thu, 10/19/2017 - 15:47
Display Headline
Facial swelling and ulceration with nasal destruction

A 12-year-old boy presents with painless swelling and ulceration on and around his nose that has progressed gradually over the last 6 months. The lesion has increased in size despite treatment with topical neomycin and oral erythromycin. He has no systemic symptoms.

Figure 1. Ulcerated plaque with destruction of the right nasal wing.
On examination (Figure 1), we note an indurated, nontender plaque with scarring at places on his right cheek, nose, and the vermilion border of the lip. In addition, there are two purulent ulcerations on the nose partly destroying the right nasal wing. The upper lip is also infiltrated, studded with a solitary ulceration. There is no regional lymphadenopathy. An examination of systems is normal.

Q: What is the diagnosis?

  • Lupus vulgaris (tuberculosis of the skin)
  • Wegener granulomatosis
  • Midline lethal granuloma (natural killer T-cell lymphoma)
  • Hansen disease (leprosy)
  • Lupoid form of cutaneous leishmaniasis

A: Lupus vulgaris is the correct diagnosis.

Cutaneous tuberculosis occurs in many forms, and lupus vulgaris is one of the most common.1 Lupus vulgaris usually arises as a result of hematogenous spread from an endogenous source. It may also arise from exogenous inoculation or as a complication of vaccination with bacille Calmette-Guérin.2

Several morphologic variants have been described.1,2 One form is characterized by plaques, often studded with psoriasiform scales. Large plaques may show irregular areas of scarring with islands of active lupus tissue and a thickened and hyperkeratotic margin. Ulcerative and mutilating variants of lupus vulgaris are characterized by scarring, ulceration, crusts over areas of necrosis, and destruction of the deep tissues and cartilage, resulting in deformities. The vegetative form produces marked infiltration, ulceration, and necrosis, with minimal scarring. Mucous membranes and cartilages are often destroyed. Tumor-like hypertrophic lesions and multiple papular and nodular lesions may also be seen. Nasal lesions may start as nodules, which may bleed and then ulcerate, sometimes resulting in cartilage destruction.

CLINICAL FEATURES AND LABORATORY WORKUP CLINCHED THE DIAGNOSIS

A number of factors helped to confirm the diagnosis in this patient:

  • A strongly positive Mantoux test (22-mm induration at 48 hours)
  • Acid-fast bacilli on Ziehl-Neelsen staining of the smear taken from the purulent ulceration
  • Isolation of Mycobacterium tuberculosis from the purulent exudates via culture in Lowenstein-Jensen medium
  • Figure 2. Epithelioid cell granuloma and giant cells (hematoxylin and eosin, × 100).
    A suggestive histopathologic picture (Figure 2)
  • The features on presentation
  • A significant clinical improvement within 2 months of starting antituberculosis therapy.

DIFFERENTIAL DIAGNOSIS

The differential diagnosis includes all the conditions in the question above. However, the absence of respiratory and renal involvement helps rule out Wegener granulomatosis; the absence of impaired sensation and nerve thickening helps rule out Hansen disease; and the absence of a nasal septal defect helps rule out Wegener granulomatosis, midline lethal granuloma, and Hansen disease.

On the other hand, the lupoid form of cutaneous leishmaniasis usually presents as an erythematous, infiltrated plaque that often closely resembles lupus vulgaris, but these lesions are usually less destructive than lupus vulgaris. However, the laboratory workup including the microbiological and histopathologic examination clearly excluded the other potential diagnoses in this patient.

TREATMENT

Lupus vulgaris is treated with standard antituberculosis therapy.3 The first phase of a fourdrug regimen is given for 2 months—isoniazid, rifampin (Rifadin), pyrazinamide, and ethambutol (Myambutol). The second phase consists of isoniazid and rifampin for 4 months.3

Early recognition and confirmation of the diagnosis followed by treatment are of immense importance for preventing permanent disfigurement.

References
  1. Freitag DS, Chin R. Facial granulomas with nasal destruction. Chest 1988; 93:422423.
  2. Yates VM. Mycobacterial infections. In:Burns T, Breathnach S, Cox N, Griffiths C, editors. Rook’s Textbook of Dermatology, Volume 2. 8th ed. Oxford: Wiley-Blackwell; 2010: 31.131.41.
  3. Takwale A, Berth–Jones J. Tuberculosis and tuberculids. In:Lebwohl MG, Berth–Jones J., Heymann WR, Coulson I, editors. Treatment of Skin Disease: Comprehensive Therapeutic. 3rd ed. Philadelphia: Saunders-Elsevier; 2010:758759.
Article PDF
Author and Disclosure Information

Sudip Kumar Ghosh, MD, DNB
Assistant Professor, Department of Dermatology, Venereology, and Leprosy, R. G. Kar Medical College, Kolkata, India

Debarbrata Bandyopadhyay, MD
Professor, Department of Dermatology, Venereology, and Leprosy, R. G. Kar Medical College, Kolkata, India

Loknath Ghoshal, MD
Resident Medical Officer-cum-Clinical Tutor, Department of Dermatology, Venereology, and Leprosy, R. G. Kar Medical College, Kolkata, India

Address: Sudip Kumar Ghosh, MD, DNB, Department of Dermatology, Venereology, and Leprosy, R. G. Kar Medical College, 1, Khudiram Bose Sarani, 700004 Kolkata, India; e-mail [email protected]

Issue
Cleveland Clinic Journal of Medicine - 78(5)
Publications
Topics
Page Number
289-290
Sections
Author and Disclosure Information

Sudip Kumar Ghosh, MD, DNB
Assistant Professor, Department of Dermatology, Venereology, and Leprosy, R. G. Kar Medical College, Kolkata, India

Debarbrata Bandyopadhyay, MD
Professor, Department of Dermatology, Venereology, and Leprosy, R. G. Kar Medical College, Kolkata, India

Loknath Ghoshal, MD
Resident Medical Officer-cum-Clinical Tutor, Department of Dermatology, Venereology, and Leprosy, R. G. Kar Medical College, Kolkata, India

Address: Sudip Kumar Ghosh, MD, DNB, Department of Dermatology, Venereology, and Leprosy, R. G. Kar Medical College, 1, Khudiram Bose Sarani, 700004 Kolkata, India; e-mail [email protected]

Author and Disclosure Information

Sudip Kumar Ghosh, MD, DNB
Assistant Professor, Department of Dermatology, Venereology, and Leprosy, R. G. Kar Medical College, Kolkata, India

Debarbrata Bandyopadhyay, MD
Professor, Department of Dermatology, Venereology, and Leprosy, R. G. Kar Medical College, Kolkata, India

Loknath Ghoshal, MD
Resident Medical Officer-cum-Clinical Tutor, Department of Dermatology, Venereology, and Leprosy, R. G. Kar Medical College, Kolkata, India

Address: Sudip Kumar Ghosh, MD, DNB, Department of Dermatology, Venereology, and Leprosy, R. G. Kar Medical College, 1, Khudiram Bose Sarani, 700004 Kolkata, India; e-mail [email protected]

Article PDF
Article PDF

A 12-year-old boy presents with painless swelling and ulceration on and around his nose that has progressed gradually over the last 6 months. The lesion has increased in size despite treatment with topical neomycin and oral erythromycin. He has no systemic symptoms.

Figure 1. Ulcerated plaque with destruction of the right nasal wing.
On examination (Figure 1), we note an indurated, nontender plaque with scarring at places on his right cheek, nose, and the vermilion border of the lip. In addition, there are two purulent ulcerations on the nose partly destroying the right nasal wing. The upper lip is also infiltrated, studded with a solitary ulceration. There is no regional lymphadenopathy. An examination of systems is normal.

Q: What is the diagnosis?

  • Lupus vulgaris (tuberculosis of the skin)
  • Wegener granulomatosis
  • Midline lethal granuloma (natural killer T-cell lymphoma)
  • Hansen disease (leprosy)
  • Lupoid form of cutaneous leishmaniasis

A: Lupus vulgaris is the correct diagnosis.

Cutaneous tuberculosis occurs in many forms, and lupus vulgaris is one of the most common.1 Lupus vulgaris usually arises as a result of hematogenous spread from an endogenous source. It may also arise from exogenous inoculation or as a complication of vaccination with bacille Calmette-Guérin.2

Several morphologic variants have been described.1,2 One form is characterized by plaques, often studded with psoriasiform scales. Large plaques may show irregular areas of scarring with islands of active lupus tissue and a thickened and hyperkeratotic margin. Ulcerative and mutilating variants of lupus vulgaris are characterized by scarring, ulceration, crusts over areas of necrosis, and destruction of the deep tissues and cartilage, resulting in deformities. The vegetative form produces marked infiltration, ulceration, and necrosis, with minimal scarring. Mucous membranes and cartilages are often destroyed. Tumor-like hypertrophic lesions and multiple papular and nodular lesions may also be seen. Nasal lesions may start as nodules, which may bleed and then ulcerate, sometimes resulting in cartilage destruction.

CLINICAL FEATURES AND LABORATORY WORKUP CLINCHED THE DIAGNOSIS

A number of factors helped to confirm the diagnosis in this patient:

  • A strongly positive Mantoux test (22-mm induration at 48 hours)
  • Acid-fast bacilli on Ziehl-Neelsen staining of the smear taken from the purulent ulceration
  • Isolation of Mycobacterium tuberculosis from the purulent exudates via culture in Lowenstein-Jensen medium
  • Figure 2. Epithelioid cell granuloma and giant cells (hematoxylin and eosin, × 100).
    A suggestive histopathologic picture (Figure 2)
  • The features on presentation
  • A significant clinical improvement within 2 months of starting antituberculosis therapy.

DIFFERENTIAL DIAGNOSIS

The differential diagnosis includes all the conditions in the question above. However, the absence of respiratory and renal involvement helps rule out Wegener granulomatosis; the absence of impaired sensation and nerve thickening helps rule out Hansen disease; and the absence of a nasal septal defect helps rule out Wegener granulomatosis, midline lethal granuloma, and Hansen disease.

On the other hand, the lupoid form of cutaneous leishmaniasis usually presents as an erythematous, infiltrated plaque that often closely resembles lupus vulgaris, but these lesions are usually less destructive than lupus vulgaris. However, the laboratory workup including the microbiological and histopathologic examination clearly excluded the other potential diagnoses in this patient.

TREATMENT

Lupus vulgaris is treated with standard antituberculosis therapy.3 The first phase of a fourdrug regimen is given for 2 months—isoniazid, rifampin (Rifadin), pyrazinamide, and ethambutol (Myambutol). The second phase consists of isoniazid and rifampin for 4 months.3

Early recognition and confirmation of the diagnosis followed by treatment are of immense importance for preventing permanent disfigurement.

A 12-year-old boy presents with painless swelling and ulceration on and around his nose that has progressed gradually over the last 6 months. The lesion has increased in size despite treatment with topical neomycin and oral erythromycin. He has no systemic symptoms.

Figure 1. Ulcerated plaque with destruction of the right nasal wing.
On examination (Figure 1), we note an indurated, nontender plaque with scarring at places on his right cheek, nose, and the vermilion border of the lip. In addition, there are two purulent ulcerations on the nose partly destroying the right nasal wing. The upper lip is also infiltrated, studded with a solitary ulceration. There is no regional lymphadenopathy. An examination of systems is normal.

Q: What is the diagnosis?

  • Lupus vulgaris (tuberculosis of the skin)
  • Wegener granulomatosis
  • Midline lethal granuloma (natural killer T-cell lymphoma)
  • Hansen disease (leprosy)
  • Lupoid form of cutaneous leishmaniasis

A: Lupus vulgaris is the correct diagnosis.

Cutaneous tuberculosis occurs in many forms, and lupus vulgaris is one of the most common.1 Lupus vulgaris usually arises as a result of hematogenous spread from an endogenous source. It may also arise from exogenous inoculation or as a complication of vaccination with bacille Calmette-Guérin.2

Several morphologic variants have been described.1,2 One form is characterized by plaques, often studded with psoriasiform scales. Large plaques may show irregular areas of scarring with islands of active lupus tissue and a thickened and hyperkeratotic margin. Ulcerative and mutilating variants of lupus vulgaris are characterized by scarring, ulceration, crusts over areas of necrosis, and destruction of the deep tissues and cartilage, resulting in deformities. The vegetative form produces marked infiltration, ulceration, and necrosis, with minimal scarring. Mucous membranes and cartilages are often destroyed. Tumor-like hypertrophic lesions and multiple papular and nodular lesions may also be seen. Nasal lesions may start as nodules, which may bleed and then ulcerate, sometimes resulting in cartilage destruction.

CLINICAL FEATURES AND LABORATORY WORKUP CLINCHED THE DIAGNOSIS

A number of factors helped to confirm the diagnosis in this patient:

  • A strongly positive Mantoux test (22-mm induration at 48 hours)
  • Acid-fast bacilli on Ziehl-Neelsen staining of the smear taken from the purulent ulceration
  • Isolation of Mycobacterium tuberculosis from the purulent exudates via culture in Lowenstein-Jensen medium
  • Figure 2. Epithelioid cell granuloma and giant cells (hematoxylin and eosin, × 100).
    A suggestive histopathologic picture (Figure 2)
  • The features on presentation
  • A significant clinical improvement within 2 months of starting antituberculosis therapy.

DIFFERENTIAL DIAGNOSIS

The differential diagnosis includes all the conditions in the question above. However, the absence of respiratory and renal involvement helps rule out Wegener granulomatosis; the absence of impaired sensation and nerve thickening helps rule out Hansen disease; and the absence of a nasal septal defect helps rule out Wegener granulomatosis, midline lethal granuloma, and Hansen disease.

On the other hand, the lupoid form of cutaneous leishmaniasis usually presents as an erythematous, infiltrated plaque that often closely resembles lupus vulgaris, but these lesions are usually less destructive than lupus vulgaris. However, the laboratory workup including the microbiological and histopathologic examination clearly excluded the other potential diagnoses in this patient.

TREATMENT

Lupus vulgaris is treated with standard antituberculosis therapy.3 The first phase of a fourdrug regimen is given for 2 months—isoniazid, rifampin (Rifadin), pyrazinamide, and ethambutol (Myambutol). The second phase consists of isoniazid and rifampin for 4 months.3

Early recognition and confirmation of the diagnosis followed by treatment are of immense importance for preventing permanent disfigurement.

References
  1. Freitag DS, Chin R. Facial granulomas with nasal destruction. Chest 1988; 93:422423.
  2. Yates VM. Mycobacterial infections. In:Burns T, Breathnach S, Cox N, Griffiths C, editors. Rook’s Textbook of Dermatology, Volume 2. 8th ed. Oxford: Wiley-Blackwell; 2010: 31.131.41.
  3. Takwale A, Berth–Jones J. Tuberculosis and tuberculids. In:Lebwohl MG, Berth–Jones J., Heymann WR, Coulson I, editors. Treatment of Skin Disease: Comprehensive Therapeutic. 3rd ed. Philadelphia: Saunders-Elsevier; 2010:758759.
References
  1. Freitag DS, Chin R. Facial granulomas with nasal destruction. Chest 1988; 93:422423.
  2. Yates VM. Mycobacterial infections. In:Burns T, Breathnach S, Cox N, Griffiths C, editors. Rook’s Textbook of Dermatology, Volume 2. 8th ed. Oxford: Wiley-Blackwell; 2010: 31.131.41.
  3. Takwale A, Berth–Jones J. Tuberculosis and tuberculids. In:Lebwohl MG, Berth–Jones J., Heymann WR, Coulson I, editors. Treatment of Skin Disease: Comprehensive Therapeutic. 3rd ed. Philadelphia: Saunders-Elsevier; 2010:758759.
Issue
Cleveland Clinic Journal of Medicine - 78(5)
Issue
Cleveland Clinic Journal of Medicine - 78(5)
Page Number
289-290
Page Number
289-290
Publications
Publications
Topics
Article Type
Display Headline
Facial swelling and ulceration with nasal destruction
Display Headline
Facial swelling and ulceration with nasal destruction
Sections
Disallow All Ads
Alternative CME
Article PDF Media

A 46-year-old man with fever, ST-segment elevation

Article Type
Changed
Tue, 06/12/2018 - 08:08
Display Headline
A 46-year-old man with fever, ST-segment elevation

An otherwise healthy 46-year-old man presents with fever, chills, and rigors that began 1 day ago. He reports shortness of breath, nausea, neck pain, sore throat, and right-sided jaw pain, but no chest pain, headache, or photophobia.

His vital signs are within normal limits, except for a temperature of 38.5°C (101.3°F). His jugular venous pressure and heart sounds are normal, and no focal deficit or nuchal rigidity is elicited. Laboratory tests (hemography, biochemistry panel, and cardiac biomarkers) are normal. An enzyme immunoassay for influenza A and B is negative, as is a rapid antigen detection test for streptococcal infection.

Figure 1. The electrocardiogram at admission shows ST-segment elevation in leads V1 and V2 (arrows) at a body temperature of 38.5°C.
Chest radiography and computed tomography of the head are normal. Standard 12-lead electrocardiography (ECG) shows 3-mm ST-segment elevation in leads V1 and V2 (Figure 1).

Figure 2. After cardiac catheterization, the electrocardiogram shows the type 1 Brugada pattern at a body temperature of 39.3°C.
He receives sublingual nitroglycerin, aspirin, clopidogrel (Plavix) 600 mg, and morphine. Emergency cardiac catheterization shows no obstructive atherosclerotic coronary disease. ECG after catheterization shows “coved” ST-segment elevations in the right precordial leads V1 and V2 (Figure 2). At this point, his temperature is 39.3°C (102°F).

Q: What is the most likely diagnosis?

  • Acute myocardial infarction
  • Coronary vasospasm
  • Brugada syndrome
  • Acute pericarditis
  • Acute meningitis

A: ST elevation commonly represents acute myocardial infarction, but it is associated with other conditions, including Prinzmetal angina, hyperkalemia, hypercalcemia, early repolarization, Brugada syndrome, and acute pericarditis.1 These conditions should be considered before an invasive intervention. The ECG findings (ST elevation in the right precordial leads) in this patient were consistent with those of Brugada syndrome.

WHAT IS BRUGADA SYNDROME?

Brugada syndrome is an arrhythmogenic disease characterized by ST-segment elevation in the right precordial leads, right bundle branch block, and a high incidence of sudden cardiac death in younger people.2 It accounts for 4% of all sudden deaths.3

Three different types of changes on ECG have been associated with Brugada syndrome. Type 1 is a coved ST-segment elevation of at least 2 mm, followed by a negative T wave, with little or no isoelectric separation, and present in more than one right precordial lead (from V1 to V3). Type 2 and type 3 patterns on ECG show the same 2-mm or greater J-point elevation, but a positive T wave gives the “saddleback” appearance to the ST-T portion.

Brugada syndrome is confirmed when a type 1 pattern is observed in conjunction with one of the following:

  • Documented ventricular fibrillation
  • Polymorphic ventricular tachycardia
  • A family history of sudden cardiac death at 45 years of age or younger
  • Type 1 pattern on ECG in family members
  • Ventricular tachycardia that can be induced with programmed electrical stimulation
  • Syncope
  • Nocturnal agonal respiration.3

This patient had type 1 changes on ECG but none of the above findings.

Brugada syndrome is inherited as an autosomal dominant trait, and mutations in gene SCN5A account for 18% to 30% of cases.3 These mutations impair the function of the sodium channel current, leading to an unopposed outward shift of net transmembrane current at the end of phase 1 of the right ventricular epicardial action potential. Interestingly, changes on ECG that are associated with Brugada syndrome are often dynamic or concealed and are unmasked by sodium channel blockers, fever, vagotonic agents, adrenergic agonists or antagonists, and various electrolyte abnormalities.3

At temperatures above the physiologic range, the inward sodium current is reduced, either because of failure of expression of sodium channels or because of premature closing of the sodium channels in genetically susceptible individuals.4 Therefore, fever can unmask Brugada syndrome, as it did in our patient.

For patients with symptomatic Brugada syndrome, the only current treatment is implantation of a cardioverter-defibrillator.5 Patients without symptoms may benefit from an electrophysiologic study for risk stratification, and an implantable cardioverter-defibrillator is recommended for those in whom ventricular fibrillation can be induced.3,5

CASE CONTINUED

The patient remained febrile after catheterization and received vancomycin (Vancocin) and ceftriaxone (Rocephin) empirically for presumed meningitis. Multiple peripheral blood cultures grew gram-positive cocci in pairs and chains, which were identified as Streptococcus pneumoniae. His fever abated soon after the antibiotic therapy was started.

Lumbar puncture was not done. Transesophageal echocardiography revealed no vegetations, with preserved ejection fraction. The patient has no family history of sudden death and no personal history of syncope or presyncope.

On hospital day 3, although his fever was gone, ECG still showed a Brugada pattern. He was discharged home on a 3-week regimen of intravenous penicillin, with plans for appropriate follow-up, and he was counseled that his family should be screened. An electrophysiologic study was not done, and he had no symptoms 1 year later.

As seen in this patient, Brugada syndrome is important to consider in the differential diagnosis in young patients who present with fever and ST elevations.

References
  1. Wang K, Asinger RW, Marriott HJ. ST-segment elevation in conditions other than acute myocardial infarction. N Engl J Med 2003; 349:21282135.
  2. Brugada P, Brugada J. Right bundle branch block, persistent ST segment elevation and sudden cardiac death: a distinct clinical and electrocardiographic syndrome. A multicenter report. J Am Coll Cardiol 1992; 20:13911396.
  3. Antzelevitch C, Brugada P, Borggrefe M, et al. Brugada syndrome: report of the second consensus conference: endorsed by the Heart Rhythm Society and the European Heart Rhythm Association. Circulation 2005; 111:659670.
  4. Dumaine R, Towbin JA, Brugada P, et al. Ionic mechanisms responsible for the electrocardiographic phenotype of the Brugada syndrome are temperature dependent. Circ Res 1999; 85:803809.
  5. Antzelevitch C, Nof E. Brugada syndrome: recent advances and controversies. Curr Cardiol Rep 2008; 10:376383.
Article PDF
Author and Disclosure Information

Atul Singla, MD
Section of Vascular Medicine, Department of Cardiovascular Medicine, Cleveland Clinic

Walter Coats, MD
Division of Cardiovascular Medicine, University of Missouri School of Medicine, Columbia, MO

Gregory C. Flaker, MD
Division of Cardiovascular Medicine, University of Missouri School of Medicine, Columbia, MO

Address: Atul Singla, MD, Section of Vascular Medicine, Department of Cardiovascular Medicine, J3-5, Cleveland Clinic, 9500 Euclid Avenue, Cleveland, OH 44195; e-mail [email protected]

Issue
Cleveland Clinic Journal of Medicine - 78(5)
Publications
Topics
Page Number
286-288
Sections
Author and Disclosure Information

Atul Singla, MD
Section of Vascular Medicine, Department of Cardiovascular Medicine, Cleveland Clinic

Walter Coats, MD
Division of Cardiovascular Medicine, University of Missouri School of Medicine, Columbia, MO

Gregory C. Flaker, MD
Division of Cardiovascular Medicine, University of Missouri School of Medicine, Columbia, MO

Address: Atul Singla, MD, Section of Vascular Medicine, Department of Cardiovascular Medicine, J3-5, Cleveland Clinic, 9500 Euclid Avenue, Cleveland, OH 44195; e-mail [email protected]

Author and Disclosure Information

Atul Singla, MD
Section of Vascular Medicine, Department of Cardiovascular Medicine, Cleveland Clinic

Walter Coats, MD
Division of Cardiovascular Medicine, University of Missouri School of Medicine, Columbia, MO

Gregory C. Flaker, MD
Division of Cardiovascular Medicine, University of Missouri School of Medicine, Columbia, MO

Address: Atul Singla, MD, Section of Vascular Medicine, Department of Cardiovascular Medicine, J3-5, Cleveland Clinic, 9500 Euclid Avenue, Cleveland, OH 44195; e-mail [email protected]

Article PDF
Article PDF

An otherwise healthy 46-year-old man presents with fever, chills, and rigors that began 1 day ago. He reports shortness of breath, nausea, neck pain, sore throat, and right-sided jaw pain, but no chest pain, headache, or photophobia.

His vital signs are within normal limits, except for a temperature of 38.5°C (101.3°F). His jugular venous pressure and heart sounds are normal, and no focal deficit or nuchal rigidity is elicited. Laboratory tests (hemography, biochemistry panel, and cardiac biomarkers) are normal. An enzyme immunoassay for influenza A and B is negative, as is a rapid antigen detection test for streptococcal infection.

Figure 1. The electrocardiogram at admission shows ST-segment elevation in leads V1 and V2 (arrows) at a body temperature of 38.5°C.
Chest radiography and computed tomography of the head are normal. Standard 12-lead electrocardiography (ECG) shows 3-mm ST-segment elevation in leads V1 and V2 (Figure 1).

Figure 2. After cardiac catheterization, the electrocardiogram shows the type 1 Brugada pattern at a body temperature of 39.3°C.
He receives sublingual nitroglycerin, aspirin, clopidogrel (Plavix) 600 mg, and morphine. Emergency cardiac catheterization shows no obstructive atherosclerotic coronary disease. ECG after catheterization shows “coved” ST-segment elevations in the right precordial leads V1 and V2 (Figure 2). At this point, his temperature is 39.3°C (102°F).

Q: What is the most likely diagnosis?

  • Acute myocardial infarction
  • Coronary vasospasm
  • Brugada syndrome
  • Acute pericarditis
  • Acute meningitis

A: ST elevation commonly represents acute myocardial infarction, but it is associated with other conditions, including Prinzmetal angina, hyperkalemia, hypercalcemia, early repolarization, Brugada syndrome, and acute pericarditis.1 These conditions should be considered before an invasive intervention. The ECG findings (ST elevation in the right precordial leads) in this patient were consistent with those of Brugada syndrome.

WHAT IS BRUGADA SYNDROME?

Brugada syndrome is an arrhythmogenic disease characterized by ST-segment elevation in the right precordial leads, right bundle branch block, and a high incidence of sudden cardiac death in younger people.2 It accounts for 4% of all sudden deaths.3

Three different types of changes on ECG have been associated with Brugada syndrome. Type 1 is a coved ST-segment elevation of at least 2 mm, followed by a negative T wave, with little or no isoelectric separation, and present in more than one right precordial lead (from V1 to V3). Type 2 and type 3 patterns on ECG show the same 2-mm or greater J-point elevation, but a positive T wave gives the “saddleback” appearance to the ST-T portion.

Brugada syndrome is confirmed when a type 1 pattern is observed in conjunction with one of the following:

  • Documented ventricular fibrillation
  • Polymorphic ventricular tachycardia
  • A family history of sudden cardiac death at 45 years of age or younger
  • Type 1 pattern on ECG in family members
  • Ventricular tachycardia that can be induced with programmed electrical stimulation
  • Syncope
  • Nocturnal agonal respiration.3

This patient had type 1 changes on ECG but none of the above findings.

Brugada syndrome is inherited as an autosomal dominant trait, and mutations in gene SCN5A account for 18% to 30% of cases.3 These mutations impair the function of the sodium channel current, leading to an unopposed outward shift of net transmembrane current at the end of phase 1 of the right ventricular epicardial action potential. Interestingly, changes on ECG that are associated with Brugada syndrome are often dynamic or concealed and are unmasked by sodium channel blockers, fever, vagotonic agents, adrenergic agonists or antagonists, and various electrolyte abnormalities.3

At temperatures above the physiologic range, the inward sodium current is reduced, either because of failure of expression of sodium channels or because of premature closing of the sodium channels in genetically susceptible individuals.4 Therefore, fever can unmask Brugada syndrome, as it did in our patient.

For patients with symptomatic Brugada syndrome, the only current treatment is implantation of a cardioverter-defibrillator.5 Patients without symptoms may benefit from an electrophysiologic study for risk stratification, and an implantable cardioverter-defibrillator is recommended for those in whom ventricular fibrillation can be induced.3,5

CASE CONTINUED

The patient remained febrile after catheterization and received vancomycin (Vancocin) and ceftriaxone (Rocephin) empirically for presumed meningitis. Multiple peripheral blood cultures grew gram-positive cocci in pairs and chains, which were identified as Streptococcus pneumoniae. His fever abated soon after the antibiotic therapy was started.

Lumbar puncture was not done. Transesophageal echocardiography revealed no vegetations, with preserved ejection fraction. The patient has no family history of sudden death and no personal history of syncope or presyncope.

On hospital day 3, although his fever was gone, ECG still showed a Brugada pattern. He was discharged home on a 3-week regimen of intravenous penicillin, with plans for appropriate follow-up, and he was counseled that his family should be screened. An electrophysiologic study was not done, and he had no symptoms 1 year later.

As seen in this patient, Brugada syndrome is important to consider in the differential diagnosis in young patients who present with fever and ST elevations.

An otherwise healthy 46-year-old man presents with fever, chills, and rigors that began 1 day ago. He reports shortness of breath, nausea, neck pain, sore throat, and right-sided jaw pain, but no chest pain, headache, or photophobia.

His vital signs are within normal limits, except for a temperature of 38.5°C (101.3°F). His jugular venous pressure and heart sounds are normal, and no focal deficit or nuchal rigidity is elicited. Laboratory tests (hemography, biochemistry panel, and cardiac biomarkers) are normal. An enzyme immunoassay for influenza A and B is negative, as is a rapid antigen detection test for streptococcal infection.

Figure 1. The electrocardiogram at admission shows ST-segment elevation in leads V1 and V2 (arrows) at a body temperature of 38.5°C.
Chest radiography and computed tomography of the head are normal. Standard 12-lead electrocardiography (ECG) shows 3-mm ST-segment elevation in leads V1 and V2 (Figure 1).

Figure 2. After cardiac catheterization, the electrocardiogram shows the type 1 Brugada pattern at a body temperature of 39.3°C.
He receives sublingual nitroglycerin, aspirin, clopidogrel (Plavix) 600 mg, and morphine. Emergency cardiac catheterization shows no obstructive atherosclerotic coronary disease. ECG after catheterization shows “coved” ST-segment elevations in the right precordial leads V1 and V2 (Figure 2). At this point, his temperature is 39.3°C (102°F).

Q: What is the most likely diagnosis?

  • Acute myocardial infarction
  • Coronary vasospasm
  • Brugada syndrome
  • Acute pericarditis
  • Acute meningitis

A: ST elevation commonly represents acute myocardial infarction, but it is associated with other conditions, including Prinzmetal angina, hyperkalemia, hypercalcemia, early repolarization, Brugada syndrome, and acute pericarditis.1 These conditions should be considered before an invasive intervention. The ECG findings (ST elevation in the right precordial leads) in this patient were consistent with those of Brugada syndrome.

WHAT IS BRUGADA SYNDROME?

Brugada syndrome is an arrhythmogenic disease characterized by ST-segment elevation in the right precordial leads, right bundle branch block, and a high incidence of sudden cardiac death in younger people.2 It accounts for 4% of all sudden deaths.3

Three different types of changes on ECG have been associated with Brugada syndrome. Type 1 is a coved ST-segment elevation of at least 2 mm, followed by a negative T wave, with little or no isoelectric separation, and present in more than one right precordial lead (from V1 to V3). Type 2 and type 3 patterns on ECG show the same 2-mm or greater J-point elevation, but a positive T wave gives the “saddleback” appearance to the ST-T portion.

Brugada syndrome is confirmed when a type 1 pattern is observed in conjunction with one of the following:

  • Documented ventricular fibrillation
  • Polymorphic ventricular tachycardia
  • A family history of sudden cardiac death at 45 years of age or younger
  • Type 1 pattern on ECG in family members
  • Ventricular tachycardia that can be induced with programmed electrical stimulation
  • Syncope
  • Nocturnal agonal respiration.3

This patient had type 1 changes on ECG but none of the above findings.

Brugada syndrome is inherited as an autosomal dominant trait, and mutations in gene SCN5A account for 18% to 30% of cases.3 These mutations impair the function of the sodium channel current, leading to an unopposed outward shift of net transmembrane current at the end of phase 1 of the right ventricular epicardial action potential. Interestingly, changes on ECG that are associated with Brugada syndrome are often dynamic or concealed and are unmasked by sodium channel blockers, fever, vagotonic agents, adrenergic agonists or antagonists, and various electrolyte abnormalities.3

At temperatures above the physiologic range, the inward sodium current is reduced, either because of failure of expression of sodium channels or because of premature closing of the sodium channels in genetically susceptible individuals.4 Therefore, fever can unmask Brugada syndrome, as it did in our patient.

For patients with symptomatic Brugada syndrome, the only current treatment is implantation of a cardioverter-defibrillator.5 Patients without symptoms may benefit from an electrophysiologic study for risk stratification, and an implantable cardioverter-defibrillator is recommended for those in whom ventricular fibrillation can be induced.3,5

CASE CONTINUED

The patient remained febrile after catheterization and received vancomycin (Vancocin) and ceftriaxone (Rocephin) empirically for presumed meningitis. Multiple peripheral blood cultures grew gram-positive cocci in pairs and chains, which were identified as Streptococcus pneumoniae. His fever abated soon after the antibiotic therapy was started.

Lumbar puncture was not done. Transesophageal echocardiography revealed no vegetations, with preserved ejection fraction. The patient has no family history of sudden death and no personal history of syncope or presyncope.

On hospital day 3, although his fever was gone, ECG still showed a Brugada pattern. He was discharged home on a 3-week regimen of intravenous penicillin, with plans for appropriate follow-up, and he was counseled that his family should be screened. An electrophysiologic study was not done, and he had no symptoms 1 year later.

As seen in this patient, Brugada syndrome is important to consider in the differential diagnosis in young patients who present with fever and ST elevations.

References
  1. Wang K, Asinger RW, Marriott HJ. ST-segment elevation in conditions other than acute myocardial infarction. N Engl J Med 2003; 349:21282135.
  2. Brugada P, Brugada J. Right bundle branch block, persistent ST segment elevation and sudden cardiac death: a distinct clinical and electrocardiographic syndrome. A multicenter report. J Am Coll Cardiol 1992; 20:13911396.
  3. Antzelevitch C, Brugada P, Borggrefe M, et al. Brugada syndrome: report of the second consensus conference: endorsed by the Heart Rhythm Society and the European Heart Rhythm Association. Circulation 2005; 111:659670.
  4. Dumaine R, Towbin JA, Brugada P, et al. Ionic mechanisms responsible for the electrocardiographic phenotype of the Brugada syndrome are temperature dependent. Circ Res 1999; 85:803809.
  5. Antzelevitch C, Nof E. Brugada syndrome: recent advances and controversies. Curr Cardiol Rep 2008; 10:376383.
References
  1. Wang K, Asinger RW, Marriott HJ. ST-segment elevation in conditions other than acute myocardial infarction. N Engl J Med 2003; 349:21282135.
  2. Brugada P, Brugada J. Right bundle branch block, persistent ST segment elevation and sudden cardiac death: a distinct clinical and electrocardiographic syndrome. A multicenter report. J Am Coll Cardiol 1992; 20:13911396.
  3. Antzelevitch C, Brugada P, Borggrefe M, et al. Brugada syndrome: report of the second consensus conference: endorsed by the Heart Rhythm Society and the European Heart Rhythm Association. Circulation 2005; 111:659670.
  4. Dumaine R, Towbin JA, Brugada P, et al. Ionic mechanisms responsible for the electrocardiographic phenotype of the Brugada syndrome are temperature dependent. Circ Res 1999; 85:803809.
  5. Antzelevitch C, Nof E. Brugada syndrome: recent advances and controversies. Curr Cardiol Rep 2008; 10:376383.
Issue
Cleveland Clinic Journal of Medicine - 78(5)
Issue
Cleveland Clinic Journal of Medicine - 78(5)
Page Number
286-288
Page Number
286-288
Publications
Publications
Topics
Article Type
Display Headline
A 46-year-old man with fever, ST-segment elevation
Display Headline
A 46-year-old man with fever, ST-segment elevation
Sections
Disallow All Ads
Alternative CME
Use ProPublica
Article PDF Media

Recognizing the unusual: The diagnostic epiphany

Article Type
Changed
Thu, 10/19/2017 - 15:32
Display Headline
Recognizing the unusual: The diagnostic epiphany

The discussion of angiotensin-converting enzyme (ACE) inhibitor therapy and visceral angioedema by Korniyenko et al in this issue of the Journal prompted me to consider the diagnostic epiphany.

This group had a patient with unexplained abdominal pain who ultimately underwent laparotomy, which did not reveal the diagnosis. I can reconstruct the thought processes that led to the decision for surgery, but far more intriguing is what provoked the “aha” moment when the true diagnosis—ACE inhibitor-associated angioedema—finally occurred to someone.

This is a rare complication of a common therapy, perhaps read about but not reasonable to expect all physicians to recall. If that is true, why can’t we incorporate technology into our care system to intelligently supplement the individual physician’s memory? What would have been the result if a “smart” electronic record had flagged the combination of ACE inhibitor therapy and recurrent abdominal pain and provided a citation on visceral angioedema?

We have all experienced a diagnostic epiphany, the sudden recognition of an arcane or unexpected diagnosis—as on the TV show House, but without the sneer or commercials. Some epiphanies result from suddenly seeing theretofore disconnected dots as a recognizable pattern. Some result from sudden recall of “I saw something like this once.” The superb diagnosticians seem to have these experiences more than the rest of us. Their powers of clinical reasoning are not always transparent. Some are based on the gestalt born of perception and experience, others are the result of incredibly compulsive structured analysis. Both require experience, contextual knowledge, and accurate historical information. These components will need to be incorporated into any diagnostic assistive software. But is this possible?

Those who have read my previous commentaries know that I value highly the clinical skills of history-taking and examination. I believe that these fundamental processes should be used to direct laboratory and imaging studies. I also optimistically expect that electronic medical records will evolve to become far more useful than most currently are, ultimately acting as true auxiliary brains, able to remind us of facts that we can’t recall (eg, that visceral angioedema is associated with ACE inhibitors). But there will never be a substitute for the artful and compulsive interview that establishes whether our patient is actually taking his or her medication, and whether there is a relationship between when a medication is ingested and when symptoms appear. The quality of the data entered into our electronic medical record (or other auxiliary brains), to then be associated with various informational databases, will always depend on the skill of the listening and examining clinician.

Article PDF
Author and Disclosure Information

Brian F. Mandell, MD, PhD
Editor in Chief

Issue
Cleveland Clinic Journal of Medicine - 78(5)
Publications
Topics
Page Number
277
Sections
Author and Disclosure Information

Brian F. Mandell, MD, PhD
Editor in Chief

Author and Disclosure Information

Brian F. Mandell, MD, PhD
Editor in Chief

Article PDF
Article PDF
Related Articles

The discussion of angiotensin-converting enzyme (ACE) inhibitor therapy and visceral angioedema by Korniyenko et al in this issue of the Journal prompted me to consider the diagnostic epiphany.

This group had a patient with unexplained abdominal pain who ultimately underwent laparotomy, which did not reveal the diagnosis. I can reconstruct the thought processes that led to the decision for surgery, but far more intriguing is what provoked the “aha” moment when the true diagnosis—ACE inhibitor-associated angioedema—finally occurred to someone.

This is a rare complication of a common therapy, perhaps read about but not reasonable to expect all physicians to recall. If that is true, why can’t we incorporate technology into our care system to intelligently supplement the individual physician’s memory? What would have been the result if a “smart” electronic record had flagged the combination of ACE inhibitor therapy and recurrent abdominal pain and provided a citation on visceral angioedema?

We have all experienced a diagnostic epiphany, the sudden recognition of an arcane or unexpected diagnosis—as on the TV show House, but without the sneer or commercials. Some epiphanies result from suddenly seeing theretofore disconnected dots as a recognizable pattern. Some result from sudden recall of “I saw something like this once.” The superb diagnosticians seem to have these experiences more than the rest of us. Their powers of clinical reasoning are not always transparent. Some are based on the gestalt born of perception and experience, others are the result of incredibly compulsive structured analysis. Both require experience, contextual knowledge, and accurate historical information. These components will need to be incorporated into any diagnostic assistive software. But is this possible?

Those who have read my previous commentaries know that I value highly the clinical skills of history-taking and examination. I believe that these fundamental processes should be used to direct laboratory and imaging studies. I also optimistically expect that electronic medical records will evolve to become far more useful than most currently are, ultimately acting as true auxiliary brains, able to remind us of facts that we can’t recall (eg, that visceral angioedema is associated with ACE inhibitors). But there will never be a substitute for the artful and compulsive interview that establishes whether our patient is actually taking his or her medication, and whether there is a relationship between when a medication is ingested and when symptoms appear. The quality of the data entered into our electronic medical record (or other auxiliary brains), to then be associated with various informational databases, will always depend on the skill of the listening and examining clinician.

The discussion of angiotensin-converting enzyme (ACE) inhibitor therapy and visceral angioedema by Korniyenko et al in this issue of the Journal prompted me to consider the diagnostic epiphany.

This group had a patient with unexplained abdominal pain who ultimately underwent laparotomy, which did not reveal the diagnosis. I can reconstruct the thought processes that led to the decision for surgery, but far more intriguing is what provoked the “aha” moment when the true diagnosis—ACE inhibitor-associated angioedema—finally occurred to someone.

This is a rare complication of a common therapy, perhaps read about but not reasonable to expect all physicians to recall. If that is true, why can’t we incorporate technology into our care system to intelligently supplement the individual physician’s memory? What would have been the result if a “smart” electronic record had flagged the combination of ACE inhibitor therapy and recurrent abdominal pain and provided a citation on visceral angioedema?

We have all experienced a diagnostic epiphany, the sudden recognition of an arcane or unexpected diagnosis—as on the TV show House, but without the sneer or commercials. Some epiphanies result from suddenly seeing theretofore disconnected dots as a recognizable pattern. Some result from sudden recall of “I saw something like this once.” The superb diagnosticians seem to have these experiences more than the rest of us. Their powers of clinical reasoning are not always transparent. Some are based on the gestalt born of perception and experience, others are the result of incredibly compulsive structured analysis. Both require experience, contextual knowledge, and accurate historical information. These components will need to be incorporated into any diagnostic assistive software. But is this possible?

Those who have read my previous commentaries know that I value highly the clinical skills of history-taking and examination. I believe that these fundamental processes should be used to direct laboratory and imaging studies. I also optimistically expect that electronic medical records will evolve to become far more useful than most currently are, ultimately acting as true auxiliary brains, able to remind us of facts that we can’t recall (eg, that visceral angioedema is associated with ACE inhibitors). But there will never be a substitute for the artful and compulsive interview that establishes whether our patient is actually taking his or her medication, and whether there is a relationship between when a medication is ingested and when symptoms appear. The quality of the data entered into our electronic medical record (or other auxiliary brains), to then be associated with various informational databases, will always depend on the skill of the listening and examining clinician.

Issue
Cleveland Clinic Journal of Medicine - 78(5)
Issue
Cleveland Clinic Journal of Medicine - 78(5)
Page Number
277
Page Number
277
Publications
Publications
Topics
Article Type
Display Headline
Recognizing the unusual: The diagnostic epiphany
Display Headline
Recognizing the unusual: The diagnostic epiphany
Sections
Disallow All Ads
Alternative CME
Article PDF Media

Visceral angioedema due to angiotensin-converting enzyme inhibitor therapy

Article Type
Changed
Fri, 10/20/2017 - 08:12
Display Headline
Visceral angioedema due to angiotensin-converting enzyme inhibitor therapy

A 57-year-old black woman presented to the emergency department with severe, dull abdominal pain associated with nonbilious vomiting and nausea. She had diabetes mellitus and hypertension, for which she had been taking metformin (Glucophage) 500 mg twice a day and lisinopril (available as Prinivil and Zestril) 20 mg daily for the last 4 years.

Multiple admissions in the past 4 years

The patient started taking lisinopril 10 mg daily in 2005, and she presented to her medical provider 2 weeks later with abdominal discomfort. Colonoscopy was performed, which revealed a benign polyp. She continued taking her medications, including lisinopril.

She continued to occasionally have abdominal pain of variable severity, but it was tolerable until 6 months later, when she presented to the emergency department with severe recurrent abdominal pain.

In view of the clinical picture, her physicians decided to treat her for small bowel obstruction, and an exploratory laparotomy was performed. The surgeons noted that she had moderate ascites, adhesions on the omentum, and a thickened high loop of the small bowel that was unequivocally viable and hyperemic, with thickening of the mesentery. Ascitic fluid was evacuated, adhesions were lysed, and the abdomen was closed. She was discharged with the same medications, including lisinopril; the dose was subsequently increased for better control of her hypertension.

The woman was admitted three more times within the same year for the same symptoms and underwent multiple workups for pancreatitis, gastritis, small-bowel obstruction, and other common gastrointestinal diseases.

Present admission

On review of systems, she denied any dry cough, weight loss or gain, food allergies, new medications, or hematochezia.

On physical examination, she had hypoactive bowel sounds and diffuse tenderness with guarding around the epigastric area.

Laboratory tests did not reveal any abnormalities; in particular, her C1 esterase concentration was normal. Stool studies were negative for infectious diseases.

Plain radiography of the abdomen showed a nonobstructive bowel-gas pattern.

Figure 1. Abdominal computed tomography with intravenous and oral contrast shows oral contrast in the middle of the lumen, giving it a “target-sign” appearance (image A). On the last two images (C and D), the contrast did not reach the lumen, but thickening of the intestinal wall is visible (arrows). Image A shows impressive thickening of the duodenal wall (arrows). The submucosa can be differentiated from the serosa in all intestinal images because of the thickening of the mucosal wall with multiple dilated loops. Image B shows some ascites and thickening of the stomach wall (arrows).
Computed tomography (CT) of the abdomen and pelvis showed diffuse thickening of the duodenal wall, jejunum, and areas of the stomach. The duodenal wall was almost four times as thick as normal, with differentiation of the mucosa and submucosa due to the degree of edema (Figure 1). There was a trace of ascites around the liver and small intestine. Previous CT of the abdomen showed perihepatic fluid collection but no evidence of small-bowel obstruction or pancreatic pathology. However, that study did show multiple areas of wall-thickening with resolution at the jejunum and distal antrum-pylorus of the stomach.

She was diagnosed with gastrointestinal angioedema secondary to angiotensin-converting enzyme (ACE) inhibitor therapy. Her lisinopril was discontinued, and the symptoms resolved completely in 24 hours. On follow-up 8 weeks and 16 months later, her symptoms had not returned.

A RARE COMPLICATION OF ACE-INHIBITOR THERAPY

Angioedema occurs in 0.1% to 0.7% of patients taking ACE inhibitors, and it can affect about 1 of 2,500 patients during the first week of exposure.1–3 It usually manifests as swelling of the face, tongue, and lips, and in rare cases, the gastrointestinal wall. Thus, visceral angioedema is a rare complication of ACE-inhibitor therapy.

Because angioedema is less obvious when it involves abdominal organs, it presents a diagnostic challenge. It is placed lower in the differential diagnosis, as other, more common, and occasionally more high-risk medical conditions are generally considered first. Most of the time, the diagnosis is missed. Some physicians may not be aware of this problem, since only a few case reports have been published. Nevertheless, this potential complication needs to be considered when any patient receiving ACE inhibitors for treatment of hypertension, myocardial infarction, heart failure, or diabetic nephropathy presents with diffuse abdominal pain, diarrhea, or edema of the upper airways.4–8

If a high level of suspicion is applied along with good clinical judgment, then hospitalizations, unnecessary procedures, patient discomfort, and unnecessary health care costs can be prevented.

A MEDLINE SEARCH

To investigate the characteristics associated with this unusual presentation, including the time of symptom onset, the types of symptoms, and the diagnostic studies performed on the patients with visceral angioedema, we performed a MEDLINE search to identify case reports and case series published in English from 1980 to 2010 on the topic of abdominal or visceral angioedema. The search terms used were “visceral,” “intestinal angioedema,” “ACE-inhibitor side effects,” and the names of various ACE inhibitors.

Pertinent articles were identified, and clinical characteristics were collected, including demographics, onset of symptoms, the drug’s name, and others. In our summary below, data are presented as the mean and standard deviation for continuous variables and percentages for categorical variables.

 

 

SUMMARY OF REPORTED CASES

Our search revealed 27 reported cases of visceral angioedema associated with ACE inhibitors (a table summarizing our findings is available).9–34 The drug most often involved was lisinopril (11 cases), followed by enalapril (Vasotec) (8 cases).

Twenty-three (82%) of the cases were in women. The mean age of the patients was 49.5 ± 12.2 years (range 29–77 years); the mean age was 46.7 ± 11.7 years in women and 57 ± 13 years in men. Unfortunately, the race and ethnicity of the patients was documented in only some cases.

In 15 (54%) of the cases, the patient presented to a physician or emergency department within 72 hours (41.1 ± 17.4) of starting therapy, and in 8 cases the patient presented between 2 weeks and 18 months.

In 10 cases (including the case we are reporting here), the patients were kept on ACE inhibitors from 2 to 9 years after the initial presentation, as the diagnosis was missed.9,12,14,18,20,31,32 In 2 cases, the dose of the ACE inhibitor had been increased after the patient presented with the abdominal pain.

All of the patients were hospitalized for further diagnostic workup.

As for the presenting symptoms, all the patients had abdominal pain, 24 (86%) had emesis, 14 (50%) had diarrhea, and 20 (71%) had ascites. Laboratory results were mostly nonspecific. Twelve (44%) of the patients had leukocytosis. The C1 esterase inhibitor concentration was measured in 18 patients, and the results were normal in all of them.

Twenty-four (86%) of the patients underwent abdominal and pelvic CT or ultrasonography as part of the initial diagnostic evaluation, and intestinal wall-thickening was found in 21 (87.5%) of them.

Either surgery or gastrointestinal biopsy was performed in 16 (57%) of the patients; the surgical procedures included 2 cholecystectomies and 1 bone marrow biopsy. Only 1 case was diagnosed on the basis of clinical suspicion and abdominal radiographs alone.

The combination of intestinal and stomach angioedema was found in only 2 cases.

Two patients were kept on an ACE inhibitor in spite of symptoms and intestinal wall edema that showed a migratory pattern on imaging after chronic exposure.

The thickening involved the jejunum in 14 patients (50%), the ileum in 8 (29%), the duodenum in 5 (18%), the stomach in 2, and the sigmoid colon in 1.

In 12 cases (43%), visceral angioedema and its symptoms resolved within 48 hours of stopping the ACE inhibitor.

A DIAGNOSIS TO KEEP IN MIND

As we have seen, the diagnosis of visceral angioedema needs to be kept in mind when a patient—especially a middle-aged woman—taking an ACE inhibitor presents with abdominal pain, vomiting, diarrhea, leukocytosis, ascites, and wall-thickening of the small bowel on imaging studies.9,35,36

The diagnosis is hard to establish, and in the interim the patient may undergo invasive and unnecessary procedures, which can be avoided by a heightened awareness of this complication. In all of the reported cases, the patients required hospitalization because of the severity of symptoms and attempts to exclude other possible diseases.36

POSSIBLY DUE TO BRADYKININ

Several theories have been proposed to explain how visceral angioedema is induced by ACE inhibitors. The possible mechanisms that have been described include the following:

  • The accumulation of bradykinin and substance P secondary to the effect of the ACE inhibitor, which may lead to the inflammatory response, therefore increasing permeability of the vascular compartment
  • Deficiency of complement and the enzymes carboxypeptidase N and alpha-1 antitrypsin
  • An antibody-antigen reaction37
  • Hormones such as estrogen and progesterone (suggested by the greater number of women represented38)
  • Contrast media used for imaging39
  • Genetic predisposition
  • Inflammation due to acute-phase proteins
  • C1-inhibitor deficiency or dysfunction (however, the levels of C1/C4 and the C1-esterase inhibitor functional activity usually are normal2,10,40).

Many other theories are being explored.11,12,38,41–53

The most plausible mechanism is an increase in the levels of bradykinin and its metabolites.45 The absence of ACE can lead to breakdown of bradykinin to des-Arg bradykinin via the minor pathway, which can lead to more pronounced vasodilation and vascular permeability.54,55 During an acute attack of angioedema secondary to ACE inhibition, the bradykinin concentration can increase to more than 10 times the normal level.56

Moreover, C-reactive protein levels were higher (mean 4.42 mg/dL ± 0.15 mg/dL) in patients with ACE-inhibitor-induced angioedema than in those with other causes of angioedema (P < .0001).52 The patients taking ACE inhibitors without any previous angioedema had normal C-reactive protein levels (0.39 mg/dL ± 0.1 mg/dL).52

INCIDENCE RATES

In our review of the literature, all of the patients were taking an ACE inhibitor, and some were taking both an ACE inhibitor and an angiotensin-receptor blocker (ARB).

Initially, the incidence rate of angioedema was thought to be 0.1% to 0.2%, but recently the Omapatrilat Cardiovascular Treatment Assessment vs Enalapril (OCTAVE) trial had more than 12,000 patients on enalapril and reported the incidence of angioedema to be 0.68%,57 with a higher risk in women than in men (0.84% vs 0.54%)58 and a relative risk of 3.03 for blacks compared with whites.59

Even though ARBs seem to be safer, angioedema can recur in up to one-third of patients who switch from an ACE inhibitor to an ARB.60–63

Moreover, one study in the United States found that the frequency of hospital admission of patients with angioedema increased from 8,839 per year in 1998 to 11,925 in 2005, and the cost was estimated to be close to $123 million in 2005.64

Interestingly, when angioedema involved the face, it developed within the first week in 60% of cases,65 whereas when visceral angioedema developed, it did so within the first week in 59% of cases. Therefore, the timing of the onset is similar regardless of the body area involved.

Smokers who developed ACE-inhibitor-induced cough had a higher risk of ACE-inhibitor-induced angioedema in a retrospective cohort study by Morimoto,66 but no relationship to the area of involvement was made.

 

 

ON IMAGING, A THICKENED BOWEL WALL

Computed tomography can reveal bowel edema and ascites more reliably than plain radiography or barium studies. Edema thickens the bowel wall, with increased contrast enhancement that makes mesenteric vessels show up on the study. In some instances edema is so significant that edematous submucosa can be differentiated from the serosa due to impressive thickening of the mucosal wall.15,16 Oral contrast can be seen in the middle of the lumen, giving it a target-sign appearance. Edema of the small bowel and ascites can lead to fluid sequestration in the abdomen, resulting in a presentation with shock.67

Magnetic resonance imaging can be even more useful in identifying gastrointestinal angioedema, but it would not be cost-effective, and based on our study, CT and ultrasonography of the abdomen were diagnostic in most cases.

AVOIDING UNNECESSARY TESTING

Hemodynamic instability and abdominal pain usually trigger a surgical consult and a more extensive workup, but with a good clinical approach, unnecessary testing and invasive diagnostic procedures can be avoided under the right circumstances.

Numerous surgical procedures have been reported in patients presenting with visceral angioedema secondary to ACE inhibitors.67 Although a thorough history and physical examination can give us a clue in the diagnosis of drug-induced gastrointestinal angioedema, CT is extremely helpful, as it shows dilated loops, thickened mucosal folds, perihepatic fluid, ascites, mesenteric edema, and a “doughnut” or “stacked coin” appearance.17,68

So far, there have been only two reports of angioedema of the stomach (the case reported by Shahzad et al10 and the current report). Angioedema can affect any visceral organ, but we usually see involvement of the jejunum followed by the ileum and duodenum.40

FINDINGS ON ENDOSCOPY

Usually, endoscopic examination of the upper and lower gastrointestinal tract does not reveal any specific pathology, but endoscopy and biopsy can rule out other causes of abdominal pain, such as Crohn disease, ulcerative colitis, infection, malignancy, granuloma, and vasculitis. Also, hereditary or acquired C1-esterase deficiency and other autoimmune disorders should be considered in the workup.18,69 In the reported cases, endoscopy revealed petechial bleeding with generalized edema.19

Biopsy often demonstrates an expanded edematous submucosal layer with inflammatory cell infiltration and protrusion of the proper muscular layer into the submucosal layer.15 A proper muscular layer and an edematous submucosal layer can produce edema so severe as to obstruct the intestine.15

Ultrasonography or CT provides essential information as to location, structure, and size, and it rules out other diagnoses. Therefore, consideration should be given to noninvasive imaging studies and laboratory testing (C1-esterase inhibitor, complement, antinuclear antibody, complete metabolic panel, complete blood cell count) before resorting to endoscopy or exploratory laparotomy.20,70 In three case reports,29,30,32 abdominal ultrasonography did not show any thickening of the small-bowel wall. Several cases have been diagnosed with the help of endoscopy.

Symptoms usually resolve when the ACE inhibitor is stopped

There is no standard treatment for ACE-inhibitor-induced visceral angioedema. In most patients, stopping the drug, giving nothing by mouth, and giving intravenous fluids to prevent dehydration are sufficient. Symptoms usually resolve within 48 hours.

In several case reports, fresh-frozen plasma was used to increase the levels of kininase II, which can degrade high levels of bradykinin.51,71,72 However, no randomized controlled trial of fresh-frozen plasma for ACE-inhibitor-induced angioedema has been published.

Drugs for hereditary angioedema—eg, recombinant C1-INH, the kallikrein inhibitor ecallantide (Kalbitor), and the BKR-2-antagonist icatibant (Firazyr)73—have not been prospectively studied in gastrointestinal angioedema associated with ACE inhibitors. Icatibant has been shown to be effective in the treatment of hereditary angioedema and could be promising in treating angioedema secondary to ACE inhibitors.8 Rosenberg et al21 described a patient who was on prednisone when she developed intestinal angioedema, thus calling into question the efficacy of steroids in the treatment of visceral angioedema.

RAISING AWARENESS

Identifying the red flags (Table 1) can promote early recognition of ACE inhibitor-induced visceral angioedema. In previous reports, a median of 10 months elapsed between the onset of angioedema and withdrawal of the ACE inhibitor (range 1 day to 10 years).74 In our review of the literature, we found several patients with visceral angioedema who took ACE inhibitors for years before the cause of their symptoms was recognized.9,12,14,18,20,31,32

More than 40 million patients are currently taking ACE inhibitors or ARBs.9 Therefore, we suggest that patients with a known history of angioedema in response to these drugs should wear an identification bracelet to increase awareness and to prevent recurrence of angioedema.

References
  1. Brown NJ, Snowden M, Griffin MR. Recurrent angiotensin-converting enzyme inhibitor–associated angioedema. JAMA 1997; 278:232233.
  2. Israili ZH, Hall WD. Cough and angioneurotic edema associated with angiotensin-converting enzyme inhibitor therapy. A review of the literature and pathophysiology. Ann Intern Med 1992; 117:234242.
  3. Messerli FH, Nussberger J. Vasopeptidase inhibition and angiooedema. Lancet 2000; 356:608609.
  4. Jessup M, Brozena S. Heart failure. N Engl J Med 2003; 348:20072018.
  5. Jessup M. The less familiar face of heart failure. J Am Coll Cardiol 2003; 41:224226.
  6. Chobanian AV. Clinical practice. Isolated systolic hypertension in the elderly. N Engl J Med 2007; 357:789796.
  7. Casas JP, Chua W, Loukogeorgakis S, et al. Effect of inhibitors of the renin-angiotensin system and other antihypertensive drugs on renal outcomes: systematic review and meta-analysis. Lancet 2005; 366:20262033.
  8. Weber MA, Messerli FH. Angiotensin-converting enzyme inhibitors and angioedema: estimating the risk. Hypertension 2008; 51:14651467.
  9. Oudit G, Girgrah N, Allard J. ACE inhibitor-induced angioedema of the intestine: Case report, incidence, pathophysiology, diagnosis and management. Can J Gastroenterol 2001; 15:827832.
  10. Shahzad G, Korsten MA, Blatt C, Motwani P. Angiotensin-converting enzyme (ACE) inhibitor-associated angioedema of the stomach and small intestine: a case report. Mt Sinai J Med 2006; 73:11231125.
  11. Chase MP, Fiarman GS, Scholz FJ, MacDermott RP. Angioedema of the small bowel due to an angiotensin-converting enzyme inhibitor. J Clin Gastroenterol 2000; 31:254257.
  12. Mullins RJ, Shanahan TM, Dobson RT. Visceral angioedema related to treatment with an ACE inhibitor. Med J Aust 1996; 165:319321.
  13. Schmidt TD, McGrath KM. Angiotensin-converting enzyme inhibitor angioedema of the intestine: a case report and review of the literature. Am J Med Sci 2002; 324:106108.
  14. Smoger SH, Sayed MA. Simultaneous mucosal and small bowel angioedema due to captopril. South Med J 1998; 91:10601063.
  15. Tojo A, Onozato ML, Fujita T. Repeated subileus due to angioedema during renin-angiotensin system blockade. Am J Med Sci 2006; 332:3638.
  16. De Backer AI, De Schepper AM, Vandevenne JE, Schoeters P, Michielsen P, Stevens WJ. CT of angioedema of the small bowel. AJR Am J Roentgenol 2001; 176:649652.
  17. Marmery H, Mirvis SE. Angiotensin-converting enzyme inhibitor-induced visceral angioedema. Clin Radiol 2006; 61:979982.
  18. Orr KK, Myers JR. Intermittent visceral edema induced by long-term enalapril administration. Ann Pharmacother 2004; 38:825827.
  19. Spahn TW, Grosse-Thie W, Mueller MK. Endoscopic visualization of angiotensin-converting enzyme inhibitor-induced small bowel angioedema as a cause of relapsing abdominal pain using double-balloon enteroscopy. Dig Dis Sci 2008; 53:12571260.
  20. Byrne TJ, Douglas DD, Landis ME, Heppell JP. Isolated visceral angioedema: an underdiagnosed complication of ACE inhibitors? Mayo Clin Proc 2000; 75:12011204.
  21. Rosenberg EI, Mishra G, Abdelmalek MF. Angiotensin-converting enzyme inhibitor-induced isolated visceral angioedema in a liver transplant recipient. Transplantation 2003; 75:730732.
  22. Salloum H, Locher C, Chenard A, et al. [Small bowel angioedema due to perindopril]. Gastroenterol Clin Biol 2005; 29:11801181.
  23. Arakawa M, Murata Y, Rikimaru Y, Sasaki Y. Drug-induced isolated visceral angioneurotic edema. Intern Med 2005; 44:975978.
  24. Abdelmalek MF, Douglas DD. Lisinopril-induced isolated visceral angioedema: review of ACE-inhibitor-induced small bowel angioedema. Dig Dis Sci 1997; 42:847850.
  25. Gregory KW, Davis RC. Images in clinical medicine. Angioedema of the intestine. N Engl J Med 1996; 334:1641.
  26. Farraye FA, Peppercorn MA, Steer ML, Joffe N, Rees M. Acute small-bowel mucosal edema following enalapril use. JAMA 1988; 259:3131.
  27. Jacobs RL, Hoberman LJ, Goldstein HM. Angioedema of the small bowel caused by an angiotensin-converting enzyme inhibitor. Am J Gastroenterol 1994; 89:127128.
  28. Herman L, Jocums SB, Coleman MD. A 29-year-old woman with crampy abdominal pain. Tenn Med 1999; 92:272273.
  29. Guy C, Cathébras P, Rousset H. Suspected angioedema of abdominal viscera. Ann Intern Med 1994; 121:900.
  30. Dupasquier E. [A rare clinical form of angioneurotic edema caused by enalapril: acute abdomen]. Arch Mal Coeur Vaiss 1994; 87:13711374.
  31. Jardine DL, Anderson JC, McClintock AD. Delayed diagnosis of recurrent visceral angio-oedema secondary to ACE inhibitor therapy. Aust N Z J Med 1999; 29:377378.
  32. Matsumura M, Haruki K, Kajinami K, Takada T. Angioedema likely related to angiotensin converting enzyme inhibitors. Intern Med 1993; 32:424426.
  33. Khan MU, Baig MA, Javed RA, et al. Benazepril induced isolated visceral angioedema: a rare and under diagnosed adverse effect of angiotensin converting enzyme inhibitors. Int J Cardiol 2007; 118:e68e69.
  34. Adhikari SP, Schneider JI. An unusual cause of abdominal pain and hypotension: angioedema of the bowel. J Emerg Med 2009; 36:2325.
  35. Gibbs CR, Lip GY, Beevers DG. Angioedema due to ACE inhibitors: increased risk in patients of African origin. Br J Clin Pharmacol 1999; 48:861865.
  36. Johnsen SP, Jacobsen J, Monster TB, Friis S, McLaughlin JK, Sørensen HT. Risk of first-time hospitalization for angioedema among users of ACE inhibitors and angiotensin receptor antagonists. Am J Med 2005; 118:14281329.
  37. Bi CK, Soltani K, Sloan JB, Weber RR, Elliott WJ, Murphy MB. Tissue-specific autoantibodies induced by captopril. Clin Res 1987; 35:922A.
  38. Bork K, Dewald G. Hereditary angioedema type III, angioedema associated with angiotensin II receptor antagonists, and female sex. Am J Med 2004; 116:644645.
  39. Witten DM, Hirsch FD, Hartman GW. Acute reactions to urographic contrast medium: incidence, clinical characteristics and relationship to history of hypersensitivity states. Am J Roentgenol Radium Ther Nucl Med 1973; 119:832840.
  40. Eck SL, Morse JH, Janssen DA, Emerson SG, Markovitz DM. Angioedema presenting as chronic gastrointestinal symptoms. Am J Gastroenterol 1993; 88:436439.
  41. Coleman JW, Yeung JH, Roberts DH, Breckenridge AM, Park BK. Drug-specific antibodies in patients receiving captopril. Br J Clin Pharmacol 1986; 22:161165.
  42. Kallenberg CG. Autoantibodies during captopril treatment. Arthritis Rheum 1985; 28:597598.
  43. Inman WH, Rawson NS, Wilton LV, Pearce GL, Speirs CJ. Postmarketing surveillance of enalapril. I: Results of prescription-event monitoring. BMJ 1988; 297:826829.
  44. Lefebvre J, Murphey LJ, Hartert TV, Jiao Shan R, Simmons WH, Brown NJ. Dipeptidyl peptidase IV activity in patients with ACE-inhibitor-associated angioedema. Hypertension 2002; 39:460464.
  45. Molinaro G, Cugno M, Perez M, et al. Angiotensin-converting enzyme inhibitor-associated angioedema is characterized by a slower degradation of des-arginine(9)-bradykinin. J Pharmacol Exp Ther 2002; 303:232237.
  46. Adam A, Cugno M, Molinaro G, Perez M, Lepage Y, Agostoni A. Aminopeptidase P in individuals with a history of angiooedema on ACE inhibitors. Lancet 2002; 359:20882089.
  47. Binkley KE, Davis A. Clinical, biochemical, and genetic characterization of a novel estrogen-dependent inherited form of angioedema. J Allergy Clin Immunol 2000; 106:546550.
  48. Yeung JH, Coleman JW, Park BK. Drug-protein conjugates—IX. Immunogenicity of captopril-protein conjugates. Biochem Pharmacol 1985; 34:40054012.
  49. Abbosh J, Anderson JA, Levine AB, Kupin WL. Angiotensin converting enzyme inhibitor-induced angioedema more prevalent in transplant patients. Ann Allergy Asthma Immunol 1999; 82:473476.
  50. Pichler WJ, Lehner R, Späth PJ. Recurrent angioedema associated with hypogonadism or anti-androgen therapy. Ann Allergy 1989; 63:301305.
  51. Bass G, Honan D. Octaplas is not equivalent to fresh frozen plasma in the treatment of acute angioedema. Eur J Anaesthesiol 2007; 24:10621063.
  52. Bas M, Hoffmann TK, Bier H, Kojda G. Increased C-reactive protein in ACE-inhibitor-induced angioedema. Br J Clin Pharmacol 2005; 59:233238.
  53. Herman AG. Differences in structure of angiotensin-converting enzyme inhibitors might predict differences in action. Am J Cardiol 1992; 70:102C108C.
  54. Cunnion KM, Lee JC, Frank MM. Capsule production and growth phase influence binding of complement to Staphylococcus aureus. Infect Immunol 2001; 69:67966803.
  55. Cunnion KM, Wagner E, Frank MM. Complement and kinins. In:Parlow TG, Stites DP, Imboden JB, editors. Medical Immunology. 10th ed. New York, NY: Lange Medical Books; 2001:186188.
  56. Pellacani A, Brunner HR, Nussberger J. Plasma kinins increase after angiotensin-converting enzyme inhibition in human subjects. Clin Sci (Lond) 1994; 87:567574.
  57. Bristol-Myers Squibb Pharmaceutical Research Institute. FDA Advisory Committee Briefing Book for OMAPATRILAT Tablets NDA 21-188. www.fda.gov/ohrms/dockets/ac/02/briefing/3877B2_01_BristolMeyersSquibb.pdf. Accessed 2/4/2011.
  58. Kostis JB, Kim HJ, Rusnak J, et al. Incidence and characteristics of angioedema associated with enalapril. Arch Intern Med 2005; 165:16371642.
  59. Mahoney EJ, Devaiah AK. Angioedema and angiotensin-converting enzyme inhibitors: are demographics a risk? Otolaryngol Head Neck Surg 2008; 139:105108.
  60. Warner KK, Visconti JA, Tschampel MM. Angiotensin II receptor blockers in patients with ACE inhibitor-induced angioedema. Ann Pharmacother 2000; 34:526528.
  61. Kyrmizakis DE, Papadakis CE, Liolios AD, et al. Angiotensin-converting enzyme inhibitors and angiotensin II receptor antagonists. Arch Otolaryngol Head Neck Surg 2004; 130:14161419.
  62. MacLean JA, Hannaway PJ. Angioedema and AT1 receptor blockers: proceed with caution. Arch Intern Med 2003; 163:14881489,
  63. Abdi R, Dong VM, Lee CJ, Ntoso KA. Angiotensin II receptor blocker-associated angioedema: on the heels of ACE inhibitor angioedema. Pharmacotherapy 2002; 22:11731175.
  64. Lin RY, Shah SN. Increasing hospitalizations due to angioedema in the United States. Ann Allergy Asthma Immunol 2008; 101:185192.
  65. Slater EE, Merrill DD, Guess HA, et al. Clinical profile of angioedema associated with angiotensin converting-enzyme inhibition. JAMA 1988; 260:967970.
  66. Morimoto T, Gandhi TK, Fiskio JM, et al. An evaluation of risk factors for adverse drug events associated with angiotensin-converting enzyme inhibitors. J Eval Clin Pract 2004; 10:499509.
  67. Cohen N, Sharon A, Golik A, Zaidenstein R, Modai D. Hereditary angioneurotic edema with severe hypovolemic shock. J Clin Gastroenterol 1993; 16:237239.
  68. Ciaccia D, Brazer SR, Baker ME. Acquired C1 esterase inhibitor deficiency causing intestinal angioedema: CT appearance. AJR Am J Roentgenol 1993; 161:12151216.
  69. Malcolm A, Prather CM. Intestinal angioedema mimicking Crohn’s disease. Med J Aust 1999; 171:418420.
  70. Schmidt TD, McGrath KM. Angiotensin-converting enzyme inhibitor angioedema of the intestine: a case report and review of the literature. Am J Med Sci 2002; 324:106108.
  71. Karim MY, Masood A. Fresh-frozen plasma as a treatment for life-threatening ACE-inhibitor angioedema. J Allergy Clin Immunol 2002; 109:370371.
  72. Warrier MR, Copilevitz CA, Dykewicz MS, Slavin RG. Fresh frozen plasma in the treatment of resistant angiotensin-converting enzyme inhibitor angioedema. Ann Allergy Asthma Immunol 2004; 92:573575.
  73. Bas M, Adams V, Suvorava T, Niehues T, Hoffmann TK, Kojda G. Nonallergic angioedema: role of bradykinin. Allergy 2007; 62:842856.
  74. Agostoni A, Cicardi M, Cugno M, Zingale LC, Gioffré D, Nussberger J. Angioedema due to angiotensin-converting enzyme inhibitors. Immunopharmacology 1999; 44:2125.
Article PDF
Author and Disclosure Information

Aleksandr Korniyenko, MD
St. Luke’s-Roosevelt Hospital Center, Columbia University College of Physicians and Surgeons, New York, NY

Carlos L. Alviar, MD
St. Luke’s-Roosevelt Hospital Center, Columbia University College of Physicians and Surgeons, New York, NY

Juan P. Cordova, MD
St. Luke’s-Roosevelt Hospital Center, Columbia University College of Physicians and Surgeons, New York, NY

Franz H. Messerli, MD, FACC, FACP
Director, Hypertension Program, Professor of Clinical Medicine, Columbia University College of Physicians and Surgeons; Division of Cardiology, St. Luke’s-Roosevelt Hospital Center, New York, NY

Address: Aleksandr Korniyenko, MD, Department of Medicine, St. Luke’s-Roosevelt Hospital Center, Columbia University College of Physicians & Surgeons, 1000 10th Avenue, New York, NY 10019; e-mail [email protected]

Issue
Cleveland Clinic Journal of Medicine - 78(5)
Publications
Topics
Page Number
297-304
Sections
Author and Disclosure Information

Aleksandr Korniyenko, MD
St. Luke’s-Roosevelt Hospital Center, Columbia University College of Physicians and Surgeons, New York, NY

Carlos L. Alviar, MD
St. Luke’s-Roosevelt Hospital Center, Columbia University College of Physicians and Surgeons, New York, NY

Juan P. Cordova, MD
St. Luke’s-Roosevelt Hospital Center, Columbia University College of Physicians and Surgeons, New York, NY

Franz H. Messerli, MD, FACC, FACP
Director, Hypertension Program, Professor of Clinical Medicine, Columbia University College of Physicians and Surgeons; Division of Cardiology, St. Luke’s-Roosevelt Hospital Center, New York, NY

Address: Aleksandr Korniyenko, MD, Department of Medicine, St. Luke’s-Roosevelt Hospital Center, Columbia University College of Physicians & Surgeons, 1000 10th Avenue, New York, NY 10019; e-mail [email protected]

Author and Disclosure Information

Aleksandr Korniyenko, MD
St. Luke’s-Roosevelt Hospital Center, Columbia University College of Physicians and Surgeons, New York, NY

Carlos L. Alviar, MD
St. Luke’s-Roosevelt Hospital Center, Columbia University College of Physicians and Surgeons, New York, NY

Juan P. Cordova, MD
St. Luke’s-Roosevelt Hospital Center, Columbia University College of Physicians and Surgeons, New York, NY

Franz H. Messerli, MD, FACC, FACP
Director, Hypertension Program, Professor of Clinical Medicine, Columbia University College of Physicians and Surgeons; Division of Cardiology, St. Luke’s-Roosevelt Hospital Center, New York, NY

Address: Aleksandr Korniyenko, MD, Department of Medicine, St. Luke’s-Roosevelt Hospital Center, Columbia University College of Physicians & Surgeons, 1000 10th Avenue, New York, NY 10019; e-mail [email protected]

Article PDF
Article PDF
Related Articles

A 57-year-old black woman presented to the emergency department with severe, dull abdominal pain associated with nonbilious vomiting and nausea. She had diabetes mellitus and hypertension, for which she had been taking metformin (Glucophage) 500 mg twice a day and lisinopril (available as Prinivil and Zestril) 20 mg daily for the last 4 years.

Multiple admissions in the past 4 years

The patient started taking lisinopril 10 mg daily in 2005, and she presented to her medical provider 2 weeks later with abdominal discomfort. Colonoscopy was performed, which revealed a benign polyp. She continued taking her medications, including lisinopril.

She continued to occasionally have abdominal pain of variable severity, but it was tolerable until 6 months later, when she presented to the emergency department with severe recurrent abdominal pain.

In view of the clinical picture, her physicians decided to treat her for small bowel obstruction, and an exploratory laparotomy was performed. The surgeons noted that she had moderate ascites, adhesions on the omentum, and a thickened high loop of the small bowel that was unequivocally viable and hyperemic, with thickening of the mesentery. Ascitic fluid was evacuated, adhesions were lysed, and the abdomen was closed. She was discharged with the same medications, including lisinopril; the dose was subsequently increased for better control of her hypertension.

The woman was admitted three more times within the same year for the same symptoms and underwent multiple workups for pancreatitis, gastritis, small-bowel obstruction, and other common gastrointestinal diseases.

Present admission

On review of systems, she denied any dry cough, weight loss or gain, food allergies, new medications, or hematochezia.

On physical examination, she had hypoactive bowel sounds and diffuse tenderness with guarding around the epigastric area.

Laboratory tests did not reveal any abnormalities; in particular, her C1 esterase concentration was normal. Stool studies were negative for infectious diseases.

Plain radiography of the abdomen showed a nonobstructive bowel-gas pattern.

Figure 1. Abdominal computed tomography with intravenous and oral contrast shows oral contrast in the middle of the lumen, giving it a “target-sign” appearance (image A). On the last two images (C and D), the contrast did not reach the lumen, but thickening of the intestinal wall is visible (arrows). Image A shows impressive thickening of the duodenal wall (arrows). The submucosa can be differentiated from the serosa in all intestinal images because of the thickening of the mucosal wall with multiple dilated loops. Image B shows some ascites and thickening of the stomach wall (arrows).
Computed tomography (CT) of the abdomen and pelvis showed diffuse thickening of the duodenal wall, jejunum, and areas of the stomach. The duodenal wall was almost four times as thick as normal, with differentiation of the mucosa and submucosa due to the degree of edema (Figure 1). There was a trace of ascites around the liver and small intestine. Previous CT of the abdomen showed perihepatic fluid collection but no evidence of small-bowel obstruction or pancreatic pathology. However, that study did show multiple areas of wall-thickening with resolution at the jejunum and distal antrum-pylorus of the stomach.

She was diagnosed with gastrointestinal angioedema secondary to angiotensin-converting enzyme (ACE) inhibitor therapy. Her lisinopril was discontinued, and the symptoms resolved completely in 24 hours. On follow-up 8 weeks and 16 months later, her symptoms had not returned.

A RARE COMPLICATION OF ACE-INHIBITOR THERAPY

Angioedema occurs in 0.1% to 0.7% of patients taking ACE inhibitors, and it can affect about 1 of 2,500 patients during the first week of exposure.1–3 It usually manifests as swelling of the face, tongue, and lips, and in rare cases, the gastrointestinal wall. Thus, visceral angioedema is a rare complication of ACE-inhibitor therapy.

Because angioedema is less obvious when it involves abdominal organs, it presents a diagnostic challenge. It is placed lower in the differential diagnosis, as other, more common, and occasionally more high-risk medical conditions are generally considered first. Most of the time, the diagnosis is missed. Some physicians may not be aware of this problem, since only a few case reports have been published. Nevertheless, this potential complication needs to be considered when any patient receiving ACE inhibitors for treatment of hypertension, myocardial infarction, heart failure, or diabetic nephropathy presents with diffuse abdominal pain, diarrhea, or edema of the upper airways.4–8

If a high level of suspicion is applied along with good clinical judgment, then hospitalizations, unnecessary procedures, patient discomfort, and unnecessary health care costs can be prevented.

A MEDLINE SEARCH

To investigate the characteristics associated with this unusual presentation, including the time of symptom onset, the types of symptoms, and the diagnostic studies performed on the patients with visceral angioedema, we performed a MEDLINE search to identify case reports and case series published in English from 1980 to 2010 on the topic of abdominal or visceral angioedema. The search terms used were “visceral,” “intestinal angioedema,” “ACE-inhibitor side effects,” and the names of various ACE inhibitors.

Pertinent articles were identified, and clinical characteristics were collected, including demographics, onset of symptoms, the drug’s name, and others. In our summary below, data are presented as the mean and standard deviation for continuous variables and percentages for categorical variables.

 

 

SUMMARY OF REPORTED CASES

Our search revealed 27 reported cases of visceral angioedema associated with ACE inhibitors (a table summarizing our findings is available).9–34 The drug most often involved was lisinopril (11 cases), followed by enalapril (Vasotec) (8 cases).

Twenty-three (82%) of the cases were in women. The mean age of the patients was 49.5 ± 12.2 years (range 29–77 years); the mean age was 46.7 ± 11.7 years in women and 57 ± 13 years in men. Unfortunately, the race and ethnicity of the patients was documented in only some cases.

In 15 (54%) of the cases, the patient presented to a physician or emergency department within 72 hours (41.1 ± 17.4) of starting therapy, and in 8 cases the patient presented between 2 weeks and 18 months.

In 10 cases (including the case we are reporting here), the patients were kept on ACE inhibitors from 2 to 9 years after the initial presentation, as the diagnosis was missed.9,12,14,18,20,31,32 In 2 cases, the dose of the ACE inhibitor had been increased after the patient presented with the abdominal pain.

All of the patients were hospitalized for further diagnostic workup.

As for the presenting symptoms, all the patients had abdominal pain, 24 (86%) had emesis, 14 (50%) had diarrhea, and 20 (71%) had ascites. Laboratory results were mostly nonspecific. Twelve (44%) of the patients had leukocytosis. The C1 esterase inhibitor concentration was measured in 18 patients, and the results were normal in all of them.

Twenty-four (86%) of the patients underwent abdominal and pelvic CT or ultrasonography as part of the initial diagnostic evaluation, and intestinal wall-thickening was found in 21 (87.5%) of them.

Either surgery or gastrointestinal biopsy was performed in 16 (57%) of the patients; the surgical procedures included 2 cholecystectomies and 1 bone marrow biopsy. Only 1 case was diagnosed on the basis of clinical suspicion and abdominal radiographs alone.

The combination of intestinal and stomach angioedema was found in only 2 cases.

Two patients were kept on an ACE inhibitor in spite of symptoms and intestinal wall edema that showed a migratory pattern on imaging after chronic exposure.

The thickening involved the jejunum in 14 patients (50%), the ileum in 8 (29%), the duodenum in 5 (18%), the stomach in 2, and the sigmoid colon in 1.

In 12 cases (43%), visceral angioedema and its symptoms resolved within 48 hours of stopping the ACE inhibitor.

A DIAGNOSIS TO KEEP IN MIND

As we have seen, the diagnosis of visceral angioedema needs to be kept in mind when a patient—especially a middle-aged woman—taking an ACE inhibitor presents with abdominal pain, vomiting, diarrhea, leukocytosis, ascites, and wall-thickening of the small bowel on imaging studies.9,35,36

The diagnosis is hard to establish, and in the interim the patient may undergo invasive and unnecessary procedures, which can be avoided by a heightened awareness of this complication. In all of the reported cases, the patients required hospitalization because of the severity of symptoms and attempts to exclude other possible diseases.36

POSSIBLY DUE TO BRADYKININ

Several theories have been proposed to explain how visceral angioedema is induced by ACE inhibitors. The possible mechanisms that have been described include the following:

  • The accumulation of bradykinin and substance P secondary to the effect of the ACE inhibitor, which may lead to the inflammatory response, therefore increasing permeability of the vascular compartment
  • Deficiency of complement and the enzymes carboxypeptidase N and alpha-1 antitrypsin
  • An antibody-antigen reaction37
  • Hormones such as estrogen and progesterone (suggested by the greater number of women represented38)
  • Contrast media used for imaging39
  • Genetic predisposition
  • Inflammation due to acute-phase proteins
  • C1-inhibitor deficiency or dysfunction (however, the levels of C1/C4 and the C1-esterase inhibitor functional activity usually are normal2,10,40).

Many other theories are being explored.11,12,38,41–53

The most plausible mechanism is an increase in the levels of bradykinin and its metabolites.45 The absence of ACE can lead to breakdown of bradykinin to des-Arg bradykinin via the minor pathway, which can lead to more pronounced vasodilation and vascular permeability.54,55 During an acute attack of angioedema secondary to ACE inhibition, the bradykinin concentration can increase to more than 10 times the normal level.56

Moreover, C-reactive protein levels were higher (mean 4.42 mg/dL ± 0.15 mg/dL) in patients with ACE-inhibitor-induced angioedema than in those with other causes of angioedema (P < .0001).52 The patients taking ACE inhibitors without any previous angioedema had normal C-reactive protein levels (0.39 mg/dL ± 0.1 mg/dL).52

INCIDENCE RATES

In our review of the literature, all of the patients were taking an ACE inhibitor, and some were taking both an ACE inhibitor and an angiotensin-receptor blocker (ARB).

Initially, the incidence rate of angioedema was thought to be 0.1% to 0.2%, but recently the Omapatrilat Cardiovascular Treatment Assessment vs Enalapril (OCTAVE) trial had more than 12,000 patients on enalapril and reported the incidence of angioedema to be 0.68%,57 with a higher risk in women than in men (0.84% vs 0.54%)58 and a relative risk of 3.03 for blacks compared with whites.59

Even though ARBs seem to be safer, angioedema can recur in up to one-third of patients who switch from an ACE inhibitor to an ARB.60–63

Moreover, one study in the United States found that the frequency of hospital admission of patients with angioedema increased from 8,839 per year in 1998 to 11,925 in 2005, and the cost was estimated to be close to $123 million in 2005.64

Interestingly, when angioedema involved the face, it developed within the first week in 60% of cases,65 whereas when visceral angioedema developed, it did so within the first week in 59% of cases. Therefore, the timing of the onset is similar regardless of the body area involved.

Smokers who developed ACE-inhibitor-induced cough had a higher risk of ACE-inhibitor-induced angioedema in a retrospective cohort study by Morimoto,66 but no relationship to the area of involvement was made.

 

 

ON IMAGING, A THICKENED BOWEL WALL

Computed tomography can reveal bowel edema and ascites more reliably than plain radiography or barium studies. Edema thickens the bowel wall, with increased contrast enhancement that makes mesenteric vessels show up on the study. In some instances edema is so significant that edematous submucosa can be differentiated from the serosa due to impressive thickening of the mucosal wall.15,16 Oral contrast can be seen in the middle of the lumen, giving it a target-sign appearance. Edema of the small bowel and ascites can lead to fluid sequestration in the abdomen, resulting in a presentation with shock.67

Magnetic resonance imaging can be even more useful in identifying gastrointestinal angioedema, but it would not be cost-effective, and based on our study, CT and ultrasonography of the abdomen were diagnostic in most cases.

AVOIDING UNNECESSARY TESTING

Hemodynamic instability and abdominal pain usually trigger a surgical consult and a more extensive workup, but with a good clinical approach, unnecessary testing and invasive diagnostic procedures can be avoided under the right circumstances.

Numerous surgical procedures have been reported in patients presenting with visceral angioedema secondary to ACE inhibitors.67 Although a thorough history and physical examination can give us a clue in the diagnosis of drug-induced gastrointestinal angioedema, CT is extremely helpful, as it shows dilated loops, thickened mucosal folds, perihepatic fluid, ascites, mesenteric edema, and a “doughnut” or “stacked coin” appearance.17,68

So far, there have been only two reports of angioedema of the stomach (the case reported by Shahzad et al10 and the current report). Angioedema can affect any visceral organ, but we usually see involvement of the jejunum followed by the ileum and duodenum.40

FINDINGS ON ENDOSCOPY

Usually, endoscopic examination of the upper and lower gastrointestinal tract does not reveal any specific pathology, but endoscopy and biopsy can rule out other causes of abdominal pain, such as Crohn disease, ulcerative colitis, infection, malignancy, granuloma, and vasculitis. Also, hereditary or acquired C1-esterase deficiency and other autoimmune disorders should be considered in the workup.18,69 In the reported cases, endoscopy revealed petechial bleeding with generalized edema.19

Biopsy often demonstrates an expanded edematous submucosal layer with inflammatory cell infiltration and protrusion of the proper muscular layer into the submucosal layer.15 A proper muscular layer and an edematous submucosal layer can produce edema so severe as to obstruct the intestine.15

Ultrasonography or CT provides essential information as to location, structure, and size, and it rules out other diagnoses. Therefore, consideration should be given to noninvasive imaging studies and laboratory testing (C1-esterase inhibitor, complement, antinuclear antibody, complete metabolic panel, complete blood cell count) before resorting to endoscopy or exploratory laparotomy.20,70 In three case reports,29,30,32 abdominal ultrasonography did not show any thickening of the small-bowel wall. Several cases have been diagnosed with the help of endoscopy.

Symptoms usually resolve when the ACE inhibitor is stopped

There is no standard treatment for ACE-inhibitor-induced visceral angioedema. In most patients, stopping the drug, giving nothing by mouth, and giving intravenous fluids to prevent dehydration are sufficient. Symptoms usually resolve within 48 hours.

In several case reports, fresh-frozen plasma was used to increase the levels of kininase II, which can degrade high levels of bradykinin.51,71,72 However, no randomized controlled trial of fresh-frozen plasma for ACE-inhibitor-induced angioedema has been published.

Drugs for hereditary angioedema—eg, recombinant C1-INH, the kallikrein inhibitor ecallantide (Kalbitor), and the BKR-2-antagonist icatibant (Firazyr)73—have not been prospectively studied in gastrointestinal angioedema associated with ACE inhibitors. Icatibant has been shown to be effective in the treatment of hereditary angioedema and could be promising in treating angioedema secondary to ACE inhibitors.8 Rosenberg et al21 described a patient who was on prednisone when she developed intestinal angioedema, thus calling into question the efficacy of steroids in the treatment of visceral angioedema.

RAISING AWARENESS

Identifying the red flags (Table 1) can promote early recognition of ACE inhibitor-induced visceral angioedema. In previous reports, a median of 10 months elapsed between the onset of angioedema and withdrawal of the ACE inhibitor (range 1 day to 10 years).74 In our review of the literature, we found several patients with visceral angioedema who took ACE inhibitors for years before the cause of their symptoms was recognized.9,12,14,18,20,31,32

More than 40 million patients are currently taking ACE inhibitors or ARBs.9 Therefore, we suggest that patients with a known history of angioedema in response to these drugs should wear an identification bracelet to increase awareness and to prevent recurrence of angioedema.

A 57-year-old black woman presented to the emergency department with severe, dull abdominal pain associated with nonbilious vomiting and nausea. She had diabetes mellitus and hypertension, for which she had been taking metformin (Glucophage) 500 mg twice a day and lisinopril (available as Prinivil and Zestril) 20 mg daily for the last 4 years.

Multiple admissions in the past 4 years

The patient started taking lisinopril 10 mg daily in 2005, and she presented to her medical provider 2 weeks later with abdominal discomfort. Colonoscopy was performed, which revealed a benign polyp. She continued taking her medications, including lisinopril.

She continued to occasionally have abdominal pain of variable severity, but it was tolerable until 6 months later, when she presented to the emergency department with severe recurrent abdominal pain.

In view of the clinical picture, her physicians decided to treat her for small bowel obstruction, and an exploratory laparotomy was performed. The surgeons noted that she had moderate ascites, adhesions on the omentum, and a thickened high loop of the small bowel that was unequivocally viable and hyperemic, with thickening of the mesentery. Ascitic fluid was evacuated, adhesions were lysed, and the abdomen was closed. She was discharged with the same medications, including lisinopril; the dose was subsequently increased for better control of her hypertension.

The woman was admitted three more times within the same year for the same symptoms and underwent multiple workups for pancreatitis, gastritis, small-bowel obstruction, and other common gastrointestinal diseases.

Present admission

On review of systems, she denied any dry cough, weight loss or gain, food allergies, new medications, or hematochezia.

On physical examination, she had hypoactive bowel sounds and diffuse tenderness with guarding around the epigastric area.

Laboratory tests did not reveal any abnormalities; in particular, her C1 esterase concentration was normal. Stool studies were negative for infectious diseases.

Plain radiography of the abdomen showed a nonobstructive bowel-gas pattern.

Figure 1. Abdominal computed tomography with intravenous and oral contrast shows oral contrast in the middle of the lumen, giving it a “target-sign” appearance (image A). On the last two images (C and D), the contrast did not reach the lumen, but thickening of the intestinal wall is visible (arrows). Image A shows impressive thickening of the duodenal wall (arrows). The submucosa can be differentiated from the serosa in all intestinal images because of the thickening of the mucosal wall with multiple dilated loops. Image B shows some ascites and thickening of the stomach wall (arrows).
Computed tomography (CT) of the abdomen and pelvis showed diffuse thickening of the duodenal wall, jejunum, and areas of the stomach. The duodenal wall was almost four times as thick as normal, with differentiation of the mucosa and submucosa due to the degree of edema (Figure 1). There was a trace of ascites around the liver and small intestine. Previous CT of the abdomen showed perihepatic fluid collection but no evidence of small-bowel obstruction or pancreatic pathology. However, that study did show multiple areas of wall-thickening with resolution at the jejunum and distal antrum-pylorus of the stomach.

She was diagnosed with gastrointestinal angioedema secondary to angiotensin-converting enzyme (ACE) inhibitor therapy. Her lisinopril was discontinued, and the symptoms resolved completely in 24 hours. On follow-up 8 weeks and 16 months later, her symptoms had not returned.

A RARE COMPLICATION OF ACE-INHIBITOR THERAPY

Angioedema occurs in 0.1% to 0.7% of patients taking ACE inhibitors, and it can affect about 1 of 2,500 patients during the first week of exposure.1–3 It usually manifests as swelling of the face, tongue, and lips, and in rare cases, the gastrointestinal wall. Thus, visceral angioedema is a rare complication of ACE-inhibitor therapy.

Because angioedema is less obvious when it involves abdominal organs, it presents a diagnostic challenge. It is placed lower in the differential diagnosis, as other, more common, and occasionally more high-risk medical conditions are generally considered first. Most of the time, the diagnosis is missed. Some physicians may not be aware of this problem, since only a few case reports have been published. Nevertheless, this potential complication needs to be considered when any patient receiving ACE inhibitors for treatment of hypertension, myocardial infarction, heart failure, or diabetic nephropathy presents with diffuse abdominal pain, diarrhea, or edema of the upper airways.4–8

If a high level of suspicion is applied along with good clinical judgment, then hospitalizations, unnecessary procedures, patient discomfort, and unnecessary health care costs can be prevented.

A MEDLINE SEARCH

To investigate the characteristics associated with this unusual presentation, including the time of symptom onset, the types of symptoms, and the diagnostic studies performed on the patients with visceral angioedema, we performed a MEDLINE search to identify case reports and case series published in English from 1980 to 2010 on the topic of abdominal or visceral angioedema. The search terms used were “visceral,” “intestinal angioedema,” “ACE-inhibitor side effects,” and the names of various ACE inhibitors.

Pertinent articles were identified, and clinical characteristics were collected, including demographics, onset of symptoms, the drug’s name, and others. In our summary below, data are presented as the mean and standard deviation for continuous variables and percentages for categorical variables.

 

 

SUMMARY OF REPORTED CASES

Our search revealed 27 reported cases of visceral angioedema associated with ACE inhibitors (a table summarizing our findings is available).9–34 The drug most often involved was lisinopril (11 cases), followed by enalapril (Vasotec) (8 cases).

Twenty-three (82%) of the cases were in women. The mean age of the patients was 49.5 ± 12.2 years (range 29–77 years); the mean age was 46.7 ± 11.7 years in women and 57 ± 13 years in men. Unfortunately, the race and ethnicity of the patients was documented in only some cases.

In 15 (54%) of the cases, the patient presented to a physician or emergency department within 72 hours (41.1 ± 17.4) of starting therapy, and in 8 cases the patient presented between 2 weeks and 18 months.

In 10 cases (including the case we are reporting here), the patients were kept on ACE inhibitors from 2 to 9 years after the initial presentation, as the diagnosis was missed.9,12,14,18,20,31,32 In 2 cases, the dose of the ACE inhibitor had been increased after the patient presented with the abdominal pain.

All of the patients were hospitalized for further diagnostic workup.

As for the presenting symptoms, all the patients had abdominal pain, 24 (86%) had emesis, 14 (50%) had diarrhea, and 20 (71%) had ascites. Laboratory results were mostly nonspecific. Twelve (44%) of the patients had leukocytosis. The C1 esterase inhibitor concentration was measured in 18 patients, and the results were normal in all of them.

Twenty-four (86%) of the patients underwent abdominal and pelvic CT or ultrasonography as part of the initial diagnostic evaluation, and intestinal wall-thickening was found in 21 (87.5%) of them.

Either surgery or gastrointestinal biopsy was performed in 16 (57%) of the patients; the surgical procedures included 2 cholecystectomies and 1 bone marrow biopsy. Only 1 case was diagnosed on the basis of clinical suspicion and abdominal radiographs alone.

The combination of intestinal and stomach angioedema was found in only 2 cases.

Two patients were kept on an ACE inhibitor in spite of symptoms and intestinal wall edema that showed a migratory pattern on imaging after chronic exposure.

The thickening involved the jejunum in 14 patients (50%), the ileum in 8 (29%), the duodenum in 5 (18%), the stomach in 2, and the sigmoid colon in 1.

In 12 cases (43%), visceral angioedema and its symptoms resolved within 48 hours of stopping the ACE inhibitor.

A DIAGNOSIS TO KEEP IN MIND

As we have seen, the diagnosis of visceral angioedema needs to be kept in mind when a patient—especially a middle-aged woman—taking an ACE inhibitor presents with abdominal pain, vomiting, diarrhea, leukocytosis, ascites, and wall-thickening of the small bowel on imaging studies.9,35,36

The diagnosis is hard to establish, and in the interim the patient may undergo invasive and unnecessary procedures, which can be avoided by a heightened awareness of this complication. In all of the reported cases, the patients required hospitalization because of the severity of symptoms and attempts to exclude other possible diseases.36

POSSIBLY DUE TO BRADYKININ

Several theories have been proposed to explain how visceral angioedema is induced by ACE inhibitors. The possible mechanisms that have been described include the following:

  • The accumulation of bradykinin and substance P secondary to the effect of the ACE inhibitor, which may lead to the inflammatory response, therefore increasing permeability of the vascular compartment
  • Deficiency of complement and the enzymes carboxypeptidase N and alpha-1 antitrypsin
  • An antibody-antigen reaction37
  • Hormones such as estrogen and progesterone (suggested by the greater number of women represented38)
  • Contrast media used for imaging39
  • Genetic predisposition
  • Inflammation due to acute-phase proteins
  • C1-inhibitor deficiency or dysfunction (however, the levels of C1/C4 and the C1-esterase inhibitor functional activity usually are normal2,10,40).

Many other theories are being explored.11,12,38,41–53

The most plausible mechanism is an increase in the levels of bradykinin and its metabolites.45 The absence of ACE can lead to breakdown of bradykinin to des-Arg bradykinin via the minor pathway, which can lead to more pronounced vasodilation and vascular permeability.54,55 During an acute attack of angioedema secondary to ACE inhibition, the bradykinin concentration can increase to more than 10 times the normal level.56

Moreover, C-reactive protein levels were higher (mean 4.42 mg/dL ± 0.15 mg/dL) in patients with ACE-inhibitor-induced angioedema than in those with other causes of angioedema (P < .0001).52 The patients taking ACE inhibitors without any previous angioedema had normal C-reactive protein levels (0.39 mg/dL ± 0.1 mg/dL).52

INCIDENCE RATES

In our review of the literature, all of the patients were taking an ACE inhibitor, and some were taking both an ACE inhibitor and an angiotensin-receptor blocker (ARB).

Initially, the incidence rate of angioedema was thought to be 0.1% to 0.2%, but recently the Omapatrilat Cardiovascular Treatment Assessment vs Enalapril (OCTAVE) trial had more than 12,000 patients on enalapril and reported the incidence of angioedema to be 0.68%,57 with a higher risk in women than in men (0.84% vs 0.54%)58 and a relative risk of 3.03 for blacks compared with whites.59

Even though ARBs seem to be safer, angioedema can recur in up to one-third of patients who switch from an ACE inhibitor to an ARB.60–63

Moreover, one study in the United States found that the frequency of hospital admission of patients with angioedema increased from 8,839 per year in 1998 to 11,925 in 2005, and the cost was estimated to be close to $123 million in 2005.64

Interestingly, when angioedema involved the face, it developed within the first week in 60% of cases,65 whereas when visceral angioedema developed, it did so within the first week in 59% of cases. Therefore, the timing of the onset is similar regardless of the body area involved.

Smokers who developed ACE-inhibitor-induced cough had a higher risk of ACE-inhibitor-induced angioedema in a retrospective cohort study by Morimoto,66 but no relationship to the area of involvement was made.

 

 

ON IMAGING, A THICKENED BOWEL WALL

Computed tomography can reveal bowel edema and ascites more reliably than plain radiography or barium studies. Edema thickens the bowel wall, with increased contrast enhancement that makes mesenteric vessels show up on the study. In some instances edema is so significant that edematous submucosa can be differentiated from the serosa due to impressive thickening of the mucosal wall.15,16 Oral contrast can be seen in the middle of the lumen, giving it a target-sign appearance. Edema of the small bowel and ascites can lead to fluid sequestration in the abdomen, resulting in a presentation with shock.67

Magnetic resonance imaging can be even more useful in identifying gastrointestinal angioedema, but it would not be cost-effective, and based on our study, CT and ultrasonography of the abdomen were diagnostic in most cases.

AVOIDING UNNECESSARY TESTING

Hemodynamic instability and abdominal pain usually trigger a surgical consult and a more extensive workup, but with a good clinical approach, unnecessary testing and invasive diagnostic procedures can be avoided under the right circumstances.

Numerous surgical procedures have been reported in patients presenting with visceral angioedema secondary to ACE inhibitors.67 Although a thorough history and physical examination can give us a clue in the diagnosis of drug-induced gastrointestinal angioedema, CT is extremely helpful, as it shows dilated loops, thickened mucosal folds, perihepatic fluid, ascites, mesenteric edema, and a “doughnut” or “stacked coin” appearance.17,68

So far, there have been only two reports of angioedema of the stomach (the case reported by Shahzad et al10 and the current report). Angioedema can affect any visceral organ, but we usually see involvement of the jejunum followed by the ileum and duodenum.40

FINDINGS ON ENDOSCOPY

Usually, endoscopic examination of the upper and lower gastrointestinal tract does not reveal any specific pathology, but endoscopy and biopsy can rule out other causes of abdominal pain, such as Crohn disease, ulcerative colitis, infection, malignancy, granuloma, and vasculitis. Also, hereditary or acquired C1-esterase deficiency and other autoimmune disorders should be considered in the workup.18,69 In the reported cases, endoscopy revealed petechial bleeding with generalized edema.19

Biopsy often demonstrates an expanded edematous submucosal layer with inflammatory cell infiltration and protrusion of the proper muscular layer into the submucosal layer.15 A proper muscular layer and an edematous submucosal layer can produce edema so severe as to obstruct the intestine.15

Ultrasonography or CT provides essential information as to location, structure, and size, and it rules out other diagnoses. Therefore, consideration should be given to noninvasive imaging studies and laboratory testing (C1-esterase inhibitor, complement, antinuclear antibody, complete metabolic panel, complete blood cell count) before resorting to endoscopy or exploratory laparotomy.20,70 In three case reports,29,30,32 abdominal ultrasonography did not show any thickening of the small-bowel wall. Several cases have been diagnosed with the help of endoscopy.

Symptoms usually resolve when the ACE inhibitor is stopped

There is no standard treatment for ACE-inhibitor-induced visceral angioedema. In most patients, stopping the drug, giving nothing by mouth, and giving intravenous fluids to prevent dehydration are sufficient. Symptoms usually resolve within 48 hours.

In several case reports, fresh-frozen plasma was used to increase the levels of kininase II, which can degrade high levels of bradykinin.51,71,72 However, no randomized controlled trial of fresh-frozen plasma for ACE-inhibitor-induced angioedema has been published.

Drugs for hereditary angioedema—eg, recombinant C1-INH, the kallikrein inhibitor ecallantide (Kalbitor), and the BKR-2-antagonist icatibant (Firazyr)73—have not been prospectively studied in gastrointestinal angioedema associated with ACE inhibitors. Icatibant has been shown to be effective in the treatment of hereditary angioedema and could be promising in treating angioedema secondary to ACE inhibitors.8 Rosenberg et al21 described a patient who was on prednisone when she developed intestinal angioedema, thus calling into question the efficacy of steroids in the treatment of visceral angioedema.

RAISING AWARENESS

Identifying the red flags (Table 1) can promote early recognition of ACE inhibitor-induced visceral angioedema. In previous reports, a median of 10 months elapsed between the onset of angioedema and withdrawal of the ACE inhibitor (range 1 day to 10 years).74 In our review of the literature, we found several patients with visceral angioedema who took ACE inhibitors for years before the cause of their symptoms was recognized.9,12,14,18,20,31,32

More than 40 million patients are currently taking ACE inhibitors or ARBs.9 Therefore, we suggest that patients with a known history of angioedema in response to these drugs should wear an identification bracelet to increase awareness and to prevent recurrence of angioedema.

References
  1. Brown NJ, Snowden M, Griffin MR. Recurrent angiotensin-converting enzyme inhibitor–associated angioedema. JAMA 1997; 278:232233.
  2. Israili ZH, Hall WD. Cough and angioneurotic edema associated with angiotensin-converting enzyme inhibitor therapy. A review of the literature and pathophysiology. Ann Intern Med 1992; 117:234242.
  3. Messerli FH, Nussberger J. Vasopeptidase inhibition and angiooedema. Lancet 2000; 356:608609.
  4. Jessup M, Brozena S. Heart failure. N Engl J Med 2003; 348:20072018.
  5. Jessup M. The less familiar face of heart failure. J Am Coll Cardiol 2003; 41:224226.
  6. Chobanian AV. Clinical practice. Isolated systolic hypertension in the elderly. N Engl J Med 2007; 357:789796.
  7. Casas JP, Chua W, Loukogeorgakis S, et al. Effect of inhibitors of the renin-angiotensin system and other antihypertensive drugs on renal outcomes: systematic review and meta-analysis. Lancet 2005; 366:20262033.
  8. Weber MA, Messerli FH. Angiotensin-converting enzyme inhibitors and angioedema: estimating the risk. Hypertension 2008; 51:14651467.
  9. Oudit G, Girgrah N, Allard J. ACE inhibitor-induced angioedema of the intestine: Case report, incidence, pathophysiology, diagnosis and management. Can J Gastroenterol 2001; 15:827832.
  10. Shahzad G, Korsten MA, Blatt C, Motwani P. Angiotensin-converting enzyme (ACE) inhibitor-associated angioedema of the stomach and small intestine: a case report. Mt Sinai J Med 2006; 73:11231125.
  11. Chase MP, Fiarman GS, Scholz FJ, MacDermott RP. Angioedema of the small bowel due to an angiotensin-converting enzyme inhibitor. J Clin Gastroenterol 2000; 31:254257.
  12. Mullins RJ, Shanahan TM, Dobson RT. Visceral angioedema related to treatment with an ACE inhibitor. Med J Aust 1996; 165:319321.
  13. Schmidt TD, McGrath KM. Angiotensin-converting enzyme inhibitor angioedema of the intestine: a case report and review of the literature. Am J Med Sci 2002; 324:106108.
  14. Smoger SH, Sayed MA. Simultaneous mucosal and small bowel angioedema due to captopril. South Med J 1998; 91:10601063.
  15. Tojo A, Onozato ML, Fujita T. Repeated subileus due to angioedema during renin-angiotensin system blockade. Am J Med Sci 2006; 332:3638.
  16. De Backer AI, De Schepper AM, Vandevenne JE, Schoeters P, Michielsen P, Stevens WJ. CT of angioedema of the small bowel. AJR Am J Roentgenol 2001; 176:649652.
  17. Marmery H, Mirvis SE. Angiotensin-converting enzyme inhibitor-induced visceral angioedema. Clin Radiol 2006; 61:979982.
  18. Orr KK, Myers JR. Intermittent visceral edema induced by long-term enalapril administration. Ann Pharmacother 2004; 38:825827.
  19. Spahn TW, Grosse-Thie W, Mueller MK. Endoscopic visualization of angiotensin-converting enzyme inhibitor-induced small bowel angioedema as a cause of relapsing abdominal pain using double-balloon enteroscopy. Dig Dis Sci 2008; 53:12571260.
  20. Byrne TJ, Douglas DD, Landis ME, Heppell JP. Isolated visceral angioedema: an underdiagnosed complication of ACE inhibitors? Mayo Clin Proc 2000; 75:12011204.
  21. Rosenberg EI, Mishra G, Abdelmalek MF. Angiotensin-converting enzyme inhibitor-induced isolated visceral angioedema in a liver transplant recipient. Transplantation 2003; 75:730732.
  22. Salloum H, Locher C, Chenard A, et al. [Small bowel angioedema due to perindopril]. Gastroenterol Clin Biol 2005; 29:11801181.
  23. Arakawa M, Murata Y, Rikimaru Y, Sasaki Y. Drug-induced isolated visceral angioneurotic edema. Intern Med 2005; 44:975978.
  24. Abdelmalek MF, Douglas DD. Lisinopril-induced isolated visceral angioedema: review of ACE-inhibitor-induced small bowel angioedema. Dig Dis Sci 1997; 42:847850.
  25. Gregory KW, Davis RC. Images in clinical medicine. Angioedema of the intestine. N Engl J Med 1996; 334:1641.
  26. Farraye FA, Peppercorn MA, Steer ML, Joffe N, Rees M. Acute small-bowel mucosal edema following enalapril use. JAMA 1988; 259:3131.
  27. Jacobs RL, Hoberman LJ, Goldstein HM. Angioedema of the small bowel caused by an angiotensin-converting enzyme inhibitor. Am J Gastroenterol 1994; 89:127128.
  28. Herman L, Jocums SB, Coleman MD. A 29-year-old woman with crampy abdominal pain. Tenn Med 1999; 92:272273.
  29. Guy C, Cathébras P, Rousset H. Suspected angioedema of abdominal viscera. Ann Intern Med 1994; 121:900.
  30. Dupasquier E. [A rare clinical form of angioneurotic edema caused by enalapril: acute abdomen]. Arch Mal Coeur Vaiss 1994; 87:13711374.
  31. Jardine DL, Anderson JC, McClintock AD. Delayed diagnosis of recurrent visceral angio-oedema secondary to ACE inhibitor therapy. Aust N Z J Med 1999; 29:377378.
  32. Matsumura M, Haruki K, Kajinami K, Takada T. Angioedema likely related to angiotensin converting enzyme inhibitors. Intern Med 1993; 32:424426.
  33. Khan MU, Baig MA, Javed RA, et al. Benazepril induced isolated visceral angioedema: a rare and under diagnosed adverse effect of angiotensin converting enzyme inhibitors. Int J Cardiol 2007; 118:e68e69.
  34. Adhikari SP, Schneider JI. An unusual cause of abdominal pain and hypotension: angioedema of the bowel. J Emerg Med 2009; 36:2325.
  35. Gibbs CR, Lip GY, Beevers DG. Angioedema due to ACE inhibitors: increased risk in patients of African origin. Br J Clin Pharmacol 1999; 48:861865.
  36. Johnsen SP, Jacobsen J, Monster TB, Friis S, McLaughlin JK, Sørensen HT. Risk of first-time hospitalization for angioedema among users of ACE inhibitors and angiotensin receptor antagonists. Am J Med 2005; 118:14281329.
  37. Bi CK, Soltani K, Sloan JB, Weber RR, Elliott WJ, Murphy MB. Tissue-specific autoantibodies induced by captopril. Clin Res 1987; 35:922A.
  38. Bork K, Dewald G. Hereditary angioedema type III, angioedema associated with angiotensin II receptor antagonists, and female sex. Am J Med 2004; 116:644645.
  39. Witten DM, Hirsch FD, Hartman GW. Acute reactions to urographic contrast medium: incidence, clinical characteristics and relationship to history of hypersensitivity states. Am J Roentgenol Radium Ther Nucl Med 1973; 119:832840.
  40. Eck SL, Morse JH, Janssen DA, Emerson SG, Markovitz DM. Angioedema presenting as chronic gastrointestinal symptoms. Am J Gastroenterol 1993; 88:436439.
  41. Coleman JW, Yeung JH, Roberts DH, Breckenridge AM, Park BK. Drug-specific antibodies in patients receiving captopril. Br J Clin Pharmacol 1986; 22:161165.
  42. Kallenberg CG. Autoantibodies during captopril treatment. Arthritis Rheum 1985; 28:597598.
  43. Inman WH, Rawson NS, Wilton LV, Pearce GL, Speirs CJ. Postmarketing surveillance of enalapril. I: Results of prescription-event monitoring. BMJ 1988; 297:826829.
  44. Lefebvre J, Murphey LJ, Hartert TV, Jiao Shan R, Simmons WH, Brown NJ. Dipeptidyl peptidase IV activity in patients with ACE-inhibitor-associated angioedema. Hypertension 2002; 39:460464.
  45. Molinaro G, Cugno M, Perez M, et al. Angiotensin-converting enzyme inhibitor-associated angioedema is characterized by a slower degradation of des-arginine(9)-bradykinin. J Pharmacol Exp Ther 2002; 303:232237.
  46. Adam A, Cugno M, Molinaro G, Perez M, Lepage Y, Agostoni A. Aminopeptidase P in individuals with a history of angiooedema on ACE inhibitors. Lancet 2002; 359:20882089.
  47. Binkley KE, Davis A. Clinical, biochemical, and genetic characterization of a novel estrogen-dependent inherited form of angioedema. J Allergy Clin Immunol 2000; 106:546550.
  48. Yeung JH, Coleman JW, Park BK. Drug-protein conjugates—IX. Immunogenicity of captopril-protein conjugates. Biochem Pharmacol 1985; 34:40054012.
  49. Abbosh J, Anderson JA, Levine AB, Kupin WL. Angiotensin converting enzyme inhibitor-induced angioedema more prevalent in transplant patients. Ann Allergy Asthma Immunol 1999; 82:473476.
  50. Pichler WJ, Lehner R, Späth PJ. Recurrent angioedema associated with hypogonadism or anti-androgen therapy. Ann Allergy 1989; 63:301305.
  51. Bass G, Honan D. Octaplas is not equivalent to fresh frozen plasma in the treatment of acute angioedema. Eur J Anaesthesiol 2007; 24:10621063.
  52. Bas M, Hoffmann TK, Bier H, Kojda G. Increased C-reactive protein in ACE-inhibitor-induced angioedema. Br J Clin Pharmacol 2005; 59:233238.
  53. Herman AG. Differences in structure of angiotensin-converting enzyme inhibitors might predict differences in action. Am J Cardiol 1992; 70:102C108C.
  54. Cunnion KM, Lee JC, Frank MM. Capsule production and growth phase influence binding of complement to Staphylococcus aureus. Infect Immunol 2001; 69:67966803.
  55. Cunnion KM, Wagner E, Frank MM. Complement and kinins. In:Parlow TG, Stites DP, Imboden JB, editors. Medical Immunology. 10th ed. New York, NY: Lange Medical Books; 2001:186188.
  56. Pellacani A, Brunner HR, Nussberger J. Plasma kinins increase after angiotensin-converting enzyme inhibition in human subjects. Clin Sci (Lond) 1994; 87:567574.
  57. Bristol-Myers Squibb Pharmaceutical Research Institute. FDA Advisory Committee Briefing Book for OMAPATRILAT Tablets NDA 21-188. www.fda.gov/ohrms/dockets/ac/02/briefing/3877B2_01_BristolMeyersSquibb.pdf. Accessed 2/4/2011.
  58. Kostis JB, Kim HJ, Rusnak J, et al. Incidence and characteristics of angioedema associated with enalapril. Arch Intern Med 2005; 165:16371642.
  59. Mahoney EJ, Devaiah AK. Angioedema and angiotensin-converting enzyme inhibitors: are demographics a risk? Otolaryngol Head Neck Surg 2008; 139:105108.
  60. Warner KK, Visconti JA, Tschampel MM. Angiotensin II receptor blockers in patients with ACE inhibitor-induced angioedema. Ann Pharmacother 2000; 34:526528.
  61. Kyrmizakis DE, Papadakis CE, Liolios AD, et al. Angiotensin-converting enzyme inhibitors and angiotensin II receptor antagonists. Arch Otolaryngol Head Neck Surg 2004; 130:14161419.
  62. MacLean JA, Hannaway PJ. Angioedema and AT1 receptor blockers: proceed with caution. Arch Intern Med 2003; 163:14881489,
  63. Abdi R, Dong VM, Lee CJ, Ntoso KA. Angiotensin II receptor blocker-associated angioedema: on the heels of ACE inhibitor angioedema. Pharmacotherapy 2002; 22:11731175.
  64. Lin RY, Shah SN. Increasing hospitalizations due to angioedema in the United States. Ann Allergy Asthma Immunol 2008; 101:185192.
  65. Slater EE, Merrill DD, Guess HA, et al. Clinical profile of angioedema associated with angiotensin converting-enzyme inhibition. JAMA 1988; 260:967970.
  66. Morimoto T, Gandhi TK, Fiskio JM, et al. An evaluation of risk factors for adverse drug events associated with angiotensin-converting enzyme inhibitors. J Eval Clin Pract 2004; 10:499509.
  67. Cohen N, Sharon A, Golik A, Zaidenstein R, Modai D. Hereditary angioneurotic edema with severe hypovolemic shock. J Clin Gastroenterol 1993; 16:237239.
  68. Ciaccia D, Brazer SR, Baker ME. Acquired C1 esterase inhibitor deficiency causing intestinal angioedema: CT appearance. AJR Am J Roentgenol 1993; 161:12151216.
  69. Malcolm A, Prather CM. Intestinal angioedema mimicking Crohn’s disease. Med J Aust 1999; 171:418420.
  70. Schmidt TD, McGrath KM. Angiotensin-converting enzyme inhibitor angioedema of the intestine: a case report and review of the literature. Am J Med Sci 2002; 324:106108.
  71. Karim MY, Masood A. Fresh-frozen plasma as a treatment for life-threatening ACE-inhibitor angioedema. J Allergy Clin Immunol 2002; 109:370371.
  72. Warrier MR, Copilevitz CA, Dykewicz MS, Slavin RG. Fresh frozen plasma in the treatment of resistant angiotensin-converting enzyme inhibitor angioedema. Ann Allergy Asthma Immunol 2004; 92:573575.
  73. Bas M, Adams V, Suvorava T, Niehues T, Hoffmann TK, Kojda G. Nonallergic angioedema: role of bradykinin. Allergy 2007; 62:842856.
  74. Agostoni A, Cicardi M, Cugno M, Zingale LC, Gioffré D, Nussberger J. Angioedema due to angiotensin-converting enzyme inhibitors. Immunopharmacology 1999; 44:2125.
References
  1. Brown NJ, Snowden M, Griffin MR. Recurrent angiotensin-converting enzyme inhibitor–associated angioedema. JAMA 1997; 278:232233.
  2. Israili ZH, Hall WD. Cough and angioneurotic edema associated with angiotensin-converting enzyme inhibitor therapy. A review of the literature and pathophysiology. Ann Intern Med 1992; 117:234242.
  3. Messerli FH, Nussberger J. Vasopeptidase inhibition and angiooedema. Lancet 2000; 356:608609.
  4. Jessup M, Brozena S. Heart failure. N Engl J Med 2003; 348:20072018.
  5. Jessup M. The less familiar face of heart failure. J Am Coll Cardiol 2003; 41:224226.
  6. Chobanian AV. Clinical practice. Isolated systolic hypertension in the elderly. N Engl J Med 2007; 357:789796.
  7. Casas JP, Chua W, Loukogeorgakis S, et al. Effect of inhibitors of the renin-angiotensin system and other antihypertensive drugs on renal outcomes: systematic review and meta-analysis. Lancet 2005; 366:20262033.
  8. Weber MA, Messerli FH. Angiotensin-converting enzyme inhibitors and angioedema: estimating the risk. Hypertension 2008; 51:14651467.
  9. Oudit G, Girgrah N, Allard J. ACE inhibitor-induced angioedema of the intestine: Case report, incidence, pathophysiology, diagnosis and management. Can J Gastroenterol 2001; 15:827832.
  10. Shahzad G, Korsten MA, Blatt C, Motwani P. Angiotensin-converting enzyme (ACE) inhibitor-associated angioedema of the stomach and small intestine: a case report. Mt Sinai J Med 2006; 73:11231125.
  11. Chase MP, Fiarman GS, Scholz FJ, MacDermott RP. Angioedema of the small bowel due to an angiotensin-converting enzyme inhibitor. J Clin Gastroenterol 2000; 31:254257.
  12. Mullins RJ, Shanahan TM, Dobson RT. Visceral angioedema related to treatment with an ACE inhibitor. Med J Aust 1996; 165:319321.
  13. Schmidt TD, McGrath KM. Angiotensin-converting enzyme inhibitor angioedema of the intestine: a case report and review of the literature. Am J Med Sci 2002; 324:106108.
  14. Smoger SH, Sayed MA. Simultaneous mucosal and small bowel angioedema due to captopril. South Med J 1998; 91:10601063.
  15. Tojo A, Onozato ML, Fujita T. Repeated subileus due to angioedema during renin-angiotensin system blockade. Am J Med Sci 2006; 332:3638.
  16. De Backer AI, De Schepper AM, Vandevenne JE, Schoeters P, Michielsen P, Stevens WJ. CT of angioedema of the small bowel. AJR Am J Roentgenol 2001; 176:649652.
  17. Marmery H, Mirvis SE. Angiotensin-converting enzyme inhibitor-induced visceral angioedema. Clin Radiol 2006; 61:979982.
  18. Orr KK, Myers JR. Intermittent visceral edema induced by long-term enalapril administration. Ann Pharmacother 2004; 38:825827.
  19. Spahn TW, Grosse-Thie W, Mueller MK. Endoscopic visualization of angiotensin-converting enzyme inhibitor-induced small bowel angioedema as a cause of relapsing abdominal pain using double-balloon enteroscopy. Dig Dis Sci 2008; 53:12571260.
  20. Byrne TJ, Douglas DD, Landis ME, Heppell JP. Isolated visceral angioedema: an underdiagnosed complication of ACE inhibitors? Mayo Clin Proc 2000; 75:12011204.
  21. Rosenberg EI, Mishra G, Abdelmalek MF. Angiotensin-converting enzyme inhibitor-induced isolated visceral angioedema in a liver transplant recipient. Transplantation 2003; 75:730732.
  22. Salloum H, Locher C, Chenard A, et al. [Small bowel angioedema due to perindopril]. Gastroenterol Clin Biol 2005; 29:11801181.
  23. Arakawa M, Murata Y, Rikimaru Y, Sasaki Y. Drug-induced isolated visceral angioneurotic edema. Intern Med 2005; 44:975978.
  24. Abdelmalek MF, Douglas DD. Lisinopril-induced isolated visceral angioedema: review of ACE-inhibitor-induced small bowel angioedema. Dig Dis Sci 1997; 42:847850.
  25. Gregory KW, Davis RC. Images in clinical medicine. Angioedema of the intestine. N Engl J Med 1996; 334:1641.
  26. Farraye FA, Peppercorn MA, Steer ML, Joffe N, Rees M. Acute small-bowel mucosal edema following enalapril use. JAMA 1988; 259:3131.
  27. Jacobs RL, Hoberman LJ, Goldstein HM. Angioedema of the small bowel caused by an angiotensin-converting enzyme inhibitor. Am J Gastroenterol 1994; 89:127128.
  28. Herman L, Jocums SB, Coleman MD. A 29-year-old woman with crampy abdominal pain. Tenn Med 1999; 92:272273.
  29. Guy C, Cathébras P, Rousset H. Suspected angioedema of abdominal viscera. Ann Intern Med 1994; 121:900.
  30. Dupasquier E. [A rare clinical form of angioneurotic edema caused by enalapril: acute abdomen]. Arch Mal Coeur Vaiss 1994; 87:13711374.
  31. Jardine DL, Anderson JC, McClintock AD. Delayed diagnosis of recurrent visceral angio-oedema secondary to ACE inhibitor therapy. Aust N Z J Med 1999; 29:377378.
  32. Matsumura M, Haruki K, Kajinami K, Takada T. Angioedema likely related to angiotensin converting enzyme inhibitors. Intern Med 1993; 32:424426.
  33. Khan MU, Baig MA, Javed RA, et al. Benazepril induced isolated visceral angioedema: a rare and under diagnosed adverse effect of angiotensin converting enzyme inhibitors. Int J Cardiol 2007; 118:e68e69.
  34. Adhikari SP, Schneider JI. An unusual cause of abdominal pain and hypotension: angioedema of the bowel. J Emerg Med 2009; 36:2325.
  35. Gibbs CR, Lip GY, Beevers DG. Angioedema due to ACE inhibitors: increased risk in patients of African origin. Br J Clin Pharmacol 1999; 48:861865.
  36. Johnsen SP, Jacobsen J, Monster TB, Friis S, McLaughlin JK, Sørensen HT. Risk of first-time hospitalization for angioedema among users of ACE inhibitors and angiotensin receptor antagonists. Am J Med 2005; 118:14281329.
  37. Bi CK, Soltani K, Sloan JB, Weber RR, Elliott WJ, Murphy MB. Tissue-specific autoantibodies induced by captopril. Clin Res 1987; 35:922A.
  38. Bork K, Dewald G. Hereditary angioedema type III, angioedema associated with angiotensin II receptor antagonists, and female sex. Am J Med 2004; 116:644645.
  39. Witten DM, Hirsch FD, Hartman GW. Acute reactions to urographic contrast medium: incidence, clinical characteristics and relationship to history of hypersensitivity states. Am J Roentgenol Radium Ther Nucl Med 1973; 119:832840.
  40. Eck SL, Morse JH, Janssen DA, Emerson SG, Markovitz DM. Angioedema presenting as chronic gastrointestinal symptoms. Am J Gastroenterol 1993; 88:436439.
  41. Coleman JW, Yeung JH, Roberts DH, Breckenridge AM, Park BK. Drug-specific antibodies in patients receiving captopril. Br J Clin Pharmacol 1986; 22:161165.
  42. Kallenberg CG. Autoantibodies during captopril treatment. Arthritis Rheum 1985; 28:597598.
  43. Inman WH, Rawson NS, Wilton LV, Pearce GL, Speirs CJ. Postmarketing surveillance of enalapril. I: Results of prescription-event monitoring. BMJ 1988; 297:826829.
  44. Lefebvre J, Murphey LJ, Hartert TV, Jiao Shan R, Simmons WH, Brown NJ. Dipeptidyl peptidase IV activity in patients with ACE-inhibitor-associated angioedema. Hypertension 2002; 39:460464.
  45. Molinaro G, Cugno M, Perez M, et al. Angiotensin-converting enzyme inhibitor-associated angioedema is characterized by a slower degradation of des-arginine(9)-bradykinin. J Pharmacol Exp Ther 2002; 303:232237.
  46. Adam A, Cugno M, Molinaro G, Perez M, Lepage Y, Agostoni A. Aminopeptidase P in individuals with a history of angiooedema on ACE inhibitors. Lancet 2002; 359:20882089.
  47. Binkley KE, Davis A. Clinical, biochemical, and genetic characterization of a novel estrogen-dependent inherited form of angioedema. J Allergy Clin Immunol 2000; 106:546550.
  48. Yeung JH, Coleman JW, Park BK. Drug-protein conjugates—IX. Immunogenicity of captopril-protein conjugates. Biochem Pharmacol 1985; 34:40054012.
  49. Abbosh J, Anderson JA, Levine AB, Kupin WL. Angiotensin converting enzyme inhibitor-induced angioedema more prevalent in transplant patients. Ann Allergy Asthma Immunol 1999; 82:473476.
  50. Pichler WJ, Lehner R, Späth PJ. Recurrent angioedema associated with hypogonadism or anti-androgen therapy. Ann Allergy 1989; 63:301305.
  51. Bass G, Honan D. Octaplas is not equivalent to fresh frozen plasma in the treatment of acute angioedema. Eur J Anaesthesiol 2007; 24:10621063.
  52. Bas M, Hoffmann TK, Bier H, Kojda G. Increased C-reactive protein in ACE-inhibitor-induced angioedema. Br J Clin Pharmacol 2005; 59:233238.
  53. Herman AG. Differences in structure of angiotensin-converting enzyme inhibitors might predict differences in action. Am J Cardiol 1992; 70:102C108C.
  54. Cunnion KM, Lee JC, Frank MM. Capsule production and growth phase influence binding of complement to Staphylococcus aureus. Infect Immunol 2001; 69:67966803.
  55. Cunnion KM, Wagner E, Frank MM. Complement and kinins. In:Parlow TG, Stites DP, Imboden JB, editors. Medical Immunology. 10th ed. New York, NY: Lange Medical Books; 2001:186188.
  56. Pellacani A, Brunner HR, Nussberger J. Plasma kinins increase after angiotensin-converting enzyme inhibition in human subjects. Clin Sci (Lond) 1994; 87:567574.
  57. Bristol-Myers Squibb Pharmaceutical Research Institute. FDA Advisory Committee Briefing Book for OMAPATRILAT Tablets NDA 21-188. www.fda.gov/ohrms/dockets/ac/02/briefing/3877B2_01_BristolMeyersSquibb.pdf. Accessed 2/4/2011.
  58. Kostis JB, Kim HJ, Rusnak J, et al. Incidence and characteristics of angioedema associated with enalapril. Arch Intern Med 2005; 165:16371642.
  59. Mahoney EJ, Devaiah AK. Angioedema and angiotensin-converting enzyme inhibitors: are demographics a risk? Otolaryngol Head Neck Surg 2008; 139:105108.
  60. Warner KK, Visconti JA, Tschampel MM. Angiotensin II receptor blockers in patients with ACE inhibitor-induced angioedema. Ann Pharmacother 2000; 34:526528.
  61. Kyrmizakis DE, Papadakis CE, Liolios AD, et al. Angiotensin-converting enzyme inhibitors and angiotensin II receptor antagonists. Arch Otolaryngol Head Neck Surg 2004; 130:14161419.
  62. MacLean JA, Hannaway PJ. Angioedema and AT1 receptor blockers: proceed with caution. Arch Intern Med 2003; 163:14881489,
  63. Abdi R, Dong VM, Lee CJ, Ntoso KA. Angiotensin II receptor blocker-associated angioedema: on the heels of ACE inhibitor angioedema. Pharmacotherapy 2002; 22:11731175.
  64. Lin RY, Shah SN. Increasing hospitalizations due to angioedema in the United States. Ann Allergy Asthma Immunol 2008; 101:185192.
  65. Slater EE, Merrill DD, Guess HA, et al. Clinical profile of angioedema associated with angiotensin converting-enzyme inhibition. JAMA 1988; 260:967970.
  66. Morimoto T, Gandhi TK, Fiskio JM, et al. An evaluation of risk factors for adverse drug events associated with angiotensin-converting enzyme inhibitors. J Eval Clin Pract 2004; 10:499509.
  67. Cohen N, Sharon A, Golik A, Zaidenstein R, Modai D. Hereditary angioneurotic edema with severe hypovolemic shock. J Clin Gastroenterol 1993; 16:237239.
  68. Ciaccia D, Brazer SR, Baker ME. Acquired C1 esterase inhibitor deficiency causing intestinal angioedema: CT appearance. AJR Am J Roentgenol 1993; 161:12151216.
  69. Malcolm A, Prather CM. Intestinal angioedema mimicking Crohn’s disease. Med J Aust 1999; 171:418420.
  70. Schmidt TD, McGrath KM. Angiotensin-converting enzyme inhibitor angioedema of the intestine: a case report and review of the literature. Am J Med Sci 2002; 324:106108.
  71. Karim MY, Masood A. Fresh-frozen plasma as a treatment for life-threatening ACE-inhibitor angioedema. J Allergy Clin Immunol 2002; 109:370371.
  72. Warrier MR, Copilevitz CA, Dykewicz MS, Slavin RG. Fresh frozen plasma in the treatment of resistant angiotensin-converting enzyme inhibitor angioedema. Ann Allergy Asthma Immunol 2004; 92:573575.
  73. Bas M, Adams V, Suvorava T, Niehues T, Hoffmann TK, Kojda G. Nonallergic angioedema: role of bradykinin. Allergy 2007; 62:842856.
  74. Agostoni A, Cicardi M, Cugno M, Zingale LC, Gioffré D, Nussberger J. Angioedema due to angiotensin-converting enzyme inhibitors. Immunopharmacology 1999; 44:2125.
Issue
Cleveland Clinic Journal of Medicine - 78(5)
Issue
Cleveland Clinic Journal of Medicine - 78(5)
Page Number
297-304
Page Number
297-304
Publications
Publications
Topics
Article Type
Display Headline
Visceral angioedema due to angiotensin-converting enzyme inhibitor therapy
Display Headline
Visceral angioedema due to angiotensin-converting enzyme inhibitor therapy
Sections
Inside the Article

KEY POINTS

  • Visceral angioedema due to ACE-inhibitor therapy can easily be diagnosed by clinical suspicion and abdominal computed tomography (CT).
  • Many physicians are not aware of this condition and so may subject patients to unnecessary invasive procedures, including surgery and endoscopy.
  • If a middle-aged woman taking an ACE inhibitor presents with abdominal pain and emesis, the differential diagnosis should include visceral angioedema, and CT should be strongly considered.
Disallow All Ads
Alternative CME
Article PDF Media

In reply: Coadministration of clopidogrel and proton pump inhibitors

Article Type
Changed
Thu, 10/19/2017 - 15:36
Display Headline
In reply: Coadministration of clopidogrel and proton pump inhibitors

In Reply: I thank Dr. Keller for his interest in my review on the side effects and drug interactions of proton pump inhibitors (PPIs).1 In particular, the concern about the potentially increased risk of a cardiovascular event in patients taking a PPI while on clopidogrel is a matter of active research. Since the prevention of death, myocardial infarction, or stroke is the desired outcome in patients receiving antiplatelet therapy, any reduction in the antiplatelet effect of clopidogrel could put patients at increased risk. Because of the enormous number of patients on both PPIs and clopidogrel, investigators are studying the effect of PPIs on clopidogrel to determine the true significance in day-to-day practice. We should expect that the data will continue to evolve in the coming years as more research is done on this important interaction.

The FDA Web site that Dr. Keller brings up2 was posted a few months after the submission of my manuscript. But even with the FDA’s cautionary words, it is important to realize that the risk that purportedly exists with the interaction of omeprazole and clopidogrel and the suggestion for the alternative use of pantoprazole are both based on pharmacokinetic, pharmacodynamic, and epidemiologic studies, not on clinical outcome data.

As much as we would like to rely on such studies, pharmacokinetic and pharmacodynamic studies do not address clinical outcomes, and observational studies cannot account for every confounder, because patients in these studies are not randomly assigned to the intervention, which is the rationale behind the necessity for a prospective trial. The Clopidogrel and the Optimization of Gastrointestinal Events (COGENT) study,3 a prospective randomized controlled trial with 3,761 analyzed patients, found no differences in adjudicated cardiovascular outcomes between groups who received a clopidogrel plus omeprazole vs clopidogrel alone.3 Although the COGENT study ended prematurely because of bankruptcy of the funding source, these outcomes represent the only randomized prospective data that can be found to date on PubMed. With such large numbers of patients in each group (1,876 and 1,885, respectively) and no differences in outcomes, it stands to reason that only a study with massive sample sizes would be able to detect a statistically significant difference. Differences between clopidogrel-treated patients taking and not taking omeprazole are likely be found in a well-designed prospective trial; however, it would be virtually impossible to find differences among PPIs.

To make matters even less convincing that therapy should be altered, the Working Group on High On-treatment Platelet Reactivity stated in their recent consensus paper that there are “limited data to support that alteration of therapy based on platelet function measurements actually improves outcomes.” 4 Additionally, a recent multisociety Expert Consensus Document discussing the concomitant use of PPIs and thienopyridine drugs to reduce gastrointestinal complications further supports this argument.5 Therefore, it is difficult to justify a marked increase in cost of the PPI selected (pantoprazole costs nearly seven times more per dose than omeprazole, according to one Web site6) for a benefit that is supported only by theoretical and observational data, not by outcome data.

As Dr. Keller also mentions, Aggrenox can be used for secondary stroke prophylaxis, but a discussion about a therapeutic exchange between clopidogrel and other antiplatelet agents was beyond the scope of my review. A recently published joint guideline of the American Heart Association and the American Stroke Association guideline should be consulted for further information.7

Other gastroprotective therapies are available. However, misoprostol (as mentioned) is associated with significant gastrointestinal side effects and must be taken four times a day. H2-receptor antagonists are not considered to be as effective as PPIs.8,9

References
  1. Madanick RD. Proton pump inhibitor side effects and drug interactions: much ado about nothing? Cleve Clin J Med 2011; 78:39–49.
  2. US Food and Drug Administration. FDA reminder to avoid concomitant use of Plavix (clopidogrel) and omeprazole. www.fda.gov/Drugs/DrugSafety/ucm231161.htm. Accessed March 23, 2011.
  3. Bhatt DL, Cryer BL, Contant CF, et al. Clopidogrel with or without omeprazole in coronary artery disease. N Engl J Med 2010; 363:1909–1917.
  4. Bonello L, Tantry US, Marcucci R, et al. Consensus and future directions on the definition of high on-treatment platelet reactivity to adenosine diphosphate. J Am Coll Cardiol 2010; 56:919–33.
  5. Abraham NS, Hlatky MA, Antman EM, et al. ACCF/ACG/AHA 2010 expert consensus document on the concomitant use of proton pump inhibitors and thienopyridines:a focused update of the ACCF/ACG/AHA 2008 expert consensus document on reducing the gastrointestinal risks of antiplatelet therapy and NSAID use. Am J Gastroenterol 2010; 105:2533–2549.
  6. HealthWarehouse. www.healthwarehouse.com. Accessed March 23, 2011.
  7. Furie KL, Kasner SE, Adams RJ, et al. Guidelines for the prevention of stroke in patients with stroke or transient ischemic attack: a guideline for healthcare professionals from the American Heart Association/American Stroke Association. Stroke 2011; 42:227–276.
  8. Bhatt DL, Scheiman J, Abraham NS, et al. ACCF/ACG/AHA 2008 expert consensus document on reducing the gastrointestinal risks of antiplatelet therapy and NSAID use: a report of the American College of Cardiology Foundation task force on clinical expert consensus documents.Circulation 2008; 118:1894–1909.
  9. Lanza FL, Chan FK, Quigley EM, et al. Guidelines forprevention of NSAID-related ulcer complications. Am JGastroenterol 2009; 104:728–738.
Article PDF
Author and Disclosure Information

Ryan D. Madanick, MD
University of North Carolina School of Medicine, Chapel Hill

Issue
Cleveland Clinic Journal of Medicine - 78(5)
Publications
Topics
Page Number
284-285
Sections
Author and Disclosure Information

Ryan D. Madanick, MD
University of North Carolina School of Medicine, Chapel Hill

Author and Disclosure Information

Ryan D. Madanick, MD
University of North Carolina School of Medicine, Chapel Hill

Article PDF
Article PDF
Related Articles

In Reply: I thank Dr. Keller for his interest in my review on the side effects and drug interactions of proton pump inhibitors (PPIs).1 In particular, the concern about the potentially increased risk of a cardiovascular event in patients taking a PPI while on clopidogrel is a matter of active research. Since the prevention of death, myocardial infarction, or stroke is the desired outcome in patients receiving antiplatelet therapy, any reduction in the antiplatelet effect of clopidogrel could put patients at increased risk. Because of the enormous number of patients on both PPIs and clopidogrel, investigators are studying the effect of PPIs on clopidogrel to determine the true significance in day-to-day practice. We should expect that the data will continue to evolve in the coming years as more research is done on this important interaction.

The FDA Web site that Dr. Keller brings up2 was posted a few months after the submission of my manuscript. But even with the FDA’s cautionary words, it is important to realize that the risk that purportedly exists with the interaction of omeprazole and clopidogrel and the suggestion for the alternative use of pantoprazole are both based on pharmacokinetic, pharmacodynamic, and epidemiologic studies, not on clinical outcome data.

As much as we would like to rely on such studies, pharmacokinetic and pharmacodynamic studies do not address clinical outcomes, and observational studies cannot account for every confounder, because patients in these studies are not randomly assigned to the intervention, which is the rationale behind the necessity for a prospective trial. The Clopidogrel and the Optimization of Gastrointestinal Events (COGENT) study,3 a prospective randomized controlled trial with 3,761 analyzed patients, found no differences in adjudicated cardiovascular outcomes between groups who received a clopidogrel plus omeprazole vs clopidogrel alone.3 Although the COGENT study ended prematurely because of bankruptcy of the funding source, these outcomes represent the only randomized prospective data that can be found to date on PubMed. With such large numbers of patients in each group (1,876 and 1,885, respectively) and no differences in outcomes, it stands to reason that only a study with massive sample sizes would be able to detect a statistically significant difference. Differences between clopidogrel-treated patients taking and not taking omeprazole are likely be found in a well-designed prospective trial; however, it would be virtually impossible to find differences among PPIs.

To make matters even less convincing that therapy should be altered, the Working Group on High On-treatment Platelet Reactivity stated in their recent consensus paper that there are “limited data to support that alteration of therapy based on platelet function measurements actually improves outcomes.” 4 Additionally, a recent multisociety Expert Consensus Document discussing the concomitant use of PPIs and thienopyridine drugs to reduce gastrointestinal complications further supports this argument.5 Therefore, it is difficult to justify a marked increase in cost of the PPI selected (pantoprazole costs nearly seven times more per dose than omeprazole, according to one Web site6) for a benefit that is supported only by theoretical and observational data, not by outcome data.

As Dr. Keller also mentions, Aggrenox can be used for secondary stroke prophylaxis, but a discussion about a therapeutic exchange between clopidogrel and other antiplatelet agents was beyond the scope of my review. A recently published joint guideline of the American Heart Association and the American Stroke Association guideline should be consulted for further information.7

Other gastroprotective therapies are available. However, misoprostol (as mentioned) is associated with significant gastrointestinal side effects and must be taken four times a day. H2-receptor antagonists are not considered to be as effective as PPIs.8,9

In Reply: I thank Dr. Keller for his interest in my review on the side effects and drug interactions of proton pump inhibitors (PPIs).1 In particular, the concern about the potentially increased risk of a cardiovascular event in patients taking a PPI while on clopidogrel is a matter of active research. Since the prevention of death, myocardial infarction, or stroke is the desired outcome in patients receiving antiplatelet therapy, any reduction in the antiplatelet effect of clopidogrel could put patients at increased risk. Because of the enormous number of patients on both PPIs and clopidogrel, investigators are studying the effect of PPIs on clopidogrel to determine the true significance in day-to-day practice. We should expect that the data will continue to evolve in the coming years as more research is done on this important interaction.

The FDA Web site that Dr. Keller brings up2 was posted a few months after the submission of my manuscript. But even with the FDA’s cautionary words, it is important to realize that the risk that purportedly exists with the interaction of omeprazole and clopidogrel and the suggestion for the alternative use of pantoprazole are both based on pharmacokinetic, pharmacodynamic, and epidemiologic studies, not on clinical outcome data.

As much as we would like to rely on such studies, pharmacokinetic and pharmacodynamic studies do not address clinical outcomes, and observational studies cannot account for every confounder, because patients in these studies are not randomly assigned to the intervention, which is the rationale behind the necessity for a prospective trial. The Clopidogrel and the Optimization of Gastrointestinal Events (COGENT) study,3 a prospective randomized controlled trial with 3,761 analyzed patients, found no differences in adjudicated cardiovascular outcomes between groups who received a clopidogrel plus omeprazole vs clopidogrel alone.3 Although the COGENT study ended prematurely because of bankruptcy of the funding source, these outcomes represent the only randomized prospective data that can be found to date on PubMed. With such large numbers of patients in each group (1,876 and 1,885, respectively) and no differences in outcomes, it stands to reason that only a study with massive sample sizes would be able to detect a statistically significant difference. Differences between clopidogrel-treated patients taking and not taking omeprazole are likely be found in a well-designed prospective trial; however, it would be virtually impossible to find differences among PPIs.

To make matters even less convincing that therapy should be altered, the Working Group on High On-treatment Platelet Reactivity stated in their recent consensus paper that there are “limited data to support that alteration of therapy based on platelet function measurements actually improves outcomes.” 4 Additionally, a recent multisociety Expert Consensus Document discussing the concomitant use of PPIs and thienopyridine drugs to reduce gastrointestinal complications further supports this argument.5 Therefore, it is difficult to justify a marked increase in cost of the PPI selected (pantoprazole costs nearly seven times more per dose than omeprazole, according to one Web site6) for a benefit that is supported only by theoretical and observational data, not by outcome data.

As Dr. Keller also mentions, Aggrenox can be used for secondary stroke prophylaxis, but a discussion about a therapeutic exchange between clopidogrel and other antiplatelet agents was beyond the scope of my review. A recently published joint guideline of the American Heart Association and the American Stroke Association guideline should be consulted for further information.7

Other gastroprotective therapies are available. However, misoprostol (as mentioned) is associated with significant gastrointestinal side effects and must be taken four times a day. H2-receptor antagonists are not considered to be as effective as PPIs.8,9

References
  1. Madanick RD. Proton pump inhibitor side effects and drug interactions: much ado about nothing? Cleve Clin J Med 2011; 78:39–49.
  2. US Food and Drug Administration. FDA reminder to avoid concomitant use of Plavix (clopidogrel) and omeprazole. www.fda.gov/Drugs/DrugSafety/ucm231161.htm. Accessed March 23, 2011.
  3. Bhatt DL, Cryer BL, Contant CF, et al. Clopidogrel with or without omeprazole in coronary artery disease. N Engl J Med 2010; 363:1909–1917.
  4. Bonello L, Tantry US, Marcucci R, et al. Consensus and future directions on the definition of high on-treatment platelet reactivity to adenosine diphosphate. J Am Coll Cardiol 2010; 56:919–33.
  5. Abraham NS, Hlatky MA, Antman EM, et al. ACCF/ACG/AHA 2010 expert consensus document on the concomitant use of proton pump inhibitors and thienopyridines:a focused update of the ACCF/ACG/AHA 2008 expert consensus document on reducing the gastrointestinal risks of antiplatelet therapy and NSAID use. Am J Gastroenterol 2010; 105:2533–2549.
  6. HealthWarehouse. www.healthwarehouse.com. Accessed March 23, 2011.
  7. Furie KL, Kasner SE, Adams RJ, et al. Guidelines for the prevention of stroke in patients with stroke or transient ischemic attack: a guideline for healthcare professionals from the American Heart Association/American Stroke Association. Stroke 2011; 42:227–276.
  8. Bhatt DL, Scheiman J, Abraham NS, et al. ACCF/ACG/AHA 2008 expert consensus document on reducing the gastrointestinal risks of antiplatelet therapy and NSAID use: a report of the American College of Cardiology Foundation task force on clinical expert consensus documents.Circulation 2008; 118:1894–1909.
  9. Lanza FL, Chan FK, Quigley EM, et al. Guidelines forprevention of NSAID-related ulcer complications. Am JGastroenterol 2009; 104:728–738.
References
  1. Madanick RD. Proton pump inhibitor side effects and drug interactions: much ado about nothing? Cleve Clin J Med 2011; 78:39–49.
  2. US Food and Drug Administration. FDA reminder to avoid concomitant use of Plavix (clopidogrel) and omeprazole. www.fda.gov/Drugs/DrugSafety/ucm231161.htm. Accessed March 23, 2011.
  3. Bhatt DL, Cryer BL, Contant CF, et al. Clopidogrel with or without omeprazole in coronary artery disease. N Engl J Med 2010; 363:1909–1917.
  4. Bonello L, Tantry US, Marcucci R, et al. Consensus and future directions on the definition of high on-treatment platelet reactivity to adenosine diphosphate. J Am Coll Cardiol 2010; 56:919–33.
  5. Abraham NS, Hlatky MA, Antman EM, et al. ACCF/ACG/AHA 2010 expert consensus document on the concomitant use of proton pump inhibitors and thienopyridines:a focused update of the ACCF/ACG/AHA 2008 expert consensus document on reducing the gastrointestinal risks of antiplatelet therapy and NSAID use. Am J Gastroenterol 2010; 105:2533–2549.
  6. HealthWarehouse. www.healthwarehouse.com. Accessed March 23, 2011.
  7. Furie KL, Kasner SE, Adams RJ, et al. Guidelines for the prevention of stroke in patients with stroke or transient ischemic attack: a guideline for healthcare professionals from the American Heart Association/American Stroke Association. Stroke 2011; 42:227–276.
  8. Bhatt DL, Scheiman J, Abraham NS, et al. ACCF/ACG/AHA 2008 expert consensus document on reducing the gastrointestinal risks of antiplatelet therapy and NSAID use: a report of the American College of Cardiology Foundation task force on clinical expert consensus documents.Circulation 2008; 118:1894–1909.
  9. Lanza FL, Chan FK, Quigley EM, et al. Guidelines forprevention of NSAID-related ulcer complications. Am JGastroenterol 2009; 104:728–738.
Issue
Cleveland Clinic Journal of Medicine - 78(5)
Issue
Cleveland Clinic Journal of Medicine - 78(5)
Page Number
284-285
Page Number
284-285
Publications
Publications
Topics
Article Type
Display Headline
In reply: Coadministration of clopidogrel and proton pump inhibitors
Display Headline
In reply: Coadministration of clopidogrel and proton pump inhibitors
Sections
Disallow All Ads
Alternative CME
Article PDF Media

Coadministration of clopidogrel and proton pump inhibitors

Article Type
Changed
Thu, 10/19/2017 - 15:34
Display Headline
Coadministration of clopidogrel and proton pump inhibitors

To the Editor: Thank you for the excellent review on proton pump inhibitors (PPIs) in the January 2011 issue.1 I would like to make the following comments about Dr. Madanick’s suggested algorithm (see Figure 2 in the article) for deciding whether to use a PPI in patients requiring clopidogrel:

A posting dated October 27, 2010, on the Web site of the US Food and Drug Administration (FDA) states the following: “With regard to the proton pump inhibitor (PPI) drug class, this recommendation [against the concomitant use of Plavix (clopidogrel) and omeprazole (Prilosec)] applies only to omeprazole and not to all PPIs. Not all PPIs have the same inhibitory effect on the enzyme [CYP2C19] that is crucial for conversion of Plavix into its active form. Pantoprazole (Protonix) may be an alternative PPI for consideration. It is a weak inhibitor of CYP2C19 and has less effect on the pharmacological activity of Plavix than omeprazole.”2 Thus, when it is deemed necessary to coadminister clopidogrel with a PPI, pantoprazole appears to be the preferred PPI.

If the patient is taking clopidogrel for stroke prophylaxis, one can consider switching to Aggrenox (aspirin plus extended-release dipyridamole), which has no warnings regarding coadministration with PPIs.

Patients taking aspirin plus clopidogrel may benefit by the addition of misoprostol (Cytotec), which is indicated for reducing the risk of aspirin-induced gastric ulcers in patients at high risk of complications from gastric ulcer.

References
  1. Madanick RD. Proton pump inhibitor side effects and drug interactions: much ado about nothing? Cleve Clin J Med 2011; 78:3949.
  2. US Food and Drug Administration. FDA reminder to avoid concomitant use of Plavix (clopidogrel) and omeprazole. www.fda.gov/Drugs/DrugSafety/ucm231161.htm. Accessed March 23, 2011.
Article PDF
Author and Disclosure Information

David L. Keller, MD
Providence Medical Group, Torrance, CA

Issue
Cleveland Clinic Journal of Medicine - 78(5)
Publications
Topics
Page Number
284-285
Sections
Author and Disclosure Information

David L. Keller, MD
Providence Medical Group, Torrance, CA

Author and Disclosure Information

David L. Keller, MD
Providence Medical Group, Torrance, CA

Article PDF
Article PDF
Related Articles

To the Editor: Thank you for the excellent review on proton pump inhibitors (PPIs) in the January 2011 issue.1 I would like to make the following comments about Dr. Madanick’s suggested algorithm (see Figure 2 in the article) for deciding whether to use a PPI in patients requiring clopidogrel:

A posting dated October 27, 2010, on the Web site of the US Food and Drug Administration (FDA) states the following: “With regard to the proton pump inhibitor (PPI) drug class, this recommendation [against the concomitant use of Plavix (clopidogrel) and omeprazole (Prilosec)] applies only to omeprazole and not to all PPIs. Not all PPIs have the same inhibitory effect on the enzyme [CYP2C19] that is crucial for conversion of Plavix into its active form. Pantoprazole (Protonix) may be an alternative PPI for consideration. It is a weak inhibitor of CYP2C19 and has less effect on the pharmacological activity of Plavix than omeprazole.”2 Thus, when it is deemed necessary to coadminister clopidogrel with a PPI, pantoprazole appears to be the preferred PPI.

If the patient is taking clopidogrel for stroke prophylaxis, one can consider switching to Aggrenox (aspirin plus extended-release dipyridamole), which has no warnings regarding coadministration with PPIs.

Patients taking aspirin plus clopidogrel may benefit by the addition of misoprostol (Cytotec), which is indicated for reducing the risk of aspirin-induced gastric ulcers in patients at high risk of complications from gastric ulcer.

To the Editor: Thank you for the excellent review on proton pump inhibitors (PPIs) in the January 2011 issue.1 I would like to make the following comments about Dr. Madanick’s suggested algorithm (see Figure 2 in the article) for deciding whether to use a PPI in patients requiring clopidogrel:

A posting dated October 27, 2010, on the Web site of the US Food and Drug Administration (FDA) states the following: “With regard to the proton pump inhibitor (PPI) drug class, this recommendation [against the concomitant use of Plavix (clopidogrel) and omeprazole (Prilosec)] applies only to omeprazole and not to all PPIs. Not all PPIs have the same inhibitory effect on the enzyme [CYP2C19] that is crucial for conversion of Plavix into its active form. Pantoprazole (Protonix) may be an alternative PPI for consideration. It is a weak inhibitor of CYP2C19 and has less effect on the pharmacological activity of Plavix than omeprazole.”2 Thus, when it is deemed necessary to coadminister clopidogrel with a PPI, pantoprazole appears to be the preferred PPI.

If the patient is taking clopidogrel for stroke prophylaxis, one can consider switching to Aggrenox (aspirin plus extended-release dipyridamole), which has no warnings regarding coadministration with PPIs.

Patients taking aspirin plus clopidogrel may benefit by the addition of misoprostol (Cytotec), which is indicated for reducing the risk of aspirin-induced gastric ulcers in patients at high risk of complications from gastric ulcer.

References
  1. Madanick RD. Proton pump inhibitor side effects and drug interactions: much ado about nothing? Cleve Clin J Med 2011; 78:3949.
  2. US Food and Drug Administration. FDA reminder to avoid concomitant use of Plavix (clopidogrel) and omeprazole. www.fda.gov/Drugs/DrugSafety/ucm231161.htm. Accessed March 23, 2011.
References
  1. Madanick RD. Proton pump inhibitor side effects and drug interactions: much ado about nothing? Cleve Clin J Med 2011; 78:3949.
  2. US Food and Drug Administration. FDA reminder to avoid concomitant use of Plavix (clopidogrel) and omeprazole. www.fda.gov/Drugs/DrugSafety/ucm231161.htm. Accessed March 23, 2011.
Issue
Cleveland Clinic Journal of Medicine - 78(5)
Issue
Cleveland Clinic Journal of Medicine - 78(5)
Page Number
284-285
Page Number
284-285
Publications
Publications
Topics
Article Type
Display Headline
Coadministration of clopidogrel and proton pump inhibitors
Display Headline
Coadministration of clopidogrel and proton pump inhibitors
Sections
Disallow All Ads
Alternative CME
Article PDF Media

Fear and Loathing on the Interview Trail

Article Type
Changed
Tue, 12/13/2016 - 12:08
Display Headline
Fear and Loathing on the Interview Trail

As we strive to provide the most advanced care possible to our patients with vascular disease, there is one area of our practice that stands out as being as archaic as cellophane wrapping of aneurysms: the process of finding a job.

Instead of an enlightening quest to find the perfect practice, many graduating vascular fellows have found it to be a tiring and laborious process. The excitement around the opportunity to finally do what we have trained so long and hard to do has been eclipsed in frustration by a disorganized and antiquated interview process.

The business side of medicine has very much been on display in the popular media with the institution of new healthcare reform legislation. No financially viable business would fly a client halfway across the country to find out if they're a 'good guy' prior to discussing a potential deal. This should not be the routine in medicine, either.

I implore vascular surgeons to utilize technology to modernize the process of expanding their practice. Replace the 'good old boy' and device rep networks with posts on the Society for Vascular Surgery (SVS) Job bank. Loose the recruiters and write a concise description of your current practice and what you have to offer. Replace the mandatory meet and greet first interview with a video conference via SKYPE. This will obviate the need to clear your schedule for the day and shelve the awkward interview in the operating theater.

A colleague of mine has equated interviewing for a vascular surgery job with trying to buy a used car without knowing the price. Even if it requires a non-disclosure agreement, opening the books to a prospective partner early in the interview process is vital. In addition to demonstrating integrity and building good will, it validates your case mix and volume. Without this knowledge, an applicant cannot make an informed decision to join a practice.

Most importantly, be professional. If a candidate is not what you're looking for, be straightforward and tactful. Communicate; don't string someone along in case another prospect falls through.

For those of you who are getting ready to or are still looking for your first job- do your homework. Converse with your faculty members, get in touch with the local device reps, and talk to former fellows about your job prospects. On the interview don't hesitate to pull the ancillary staff aside for their opinion of the group. One of my co-fellows did this and learned that the surgeon had not booked a case in the operating room for six months. The hospital administrators confessed that they were interviewing to replace the surgeon, not hire on a partner.

There are a multitude of resources available to arm yourself when it comes to contract negotiations. Read "The Physician's Comprehensive Guide to Negotiating" (SEAK, Inc.) by Steven Babitsky and James J. Mangraviti, Jr. I obtained a copy through interlibrary loan and read it in two days between cases. It is well worth the small time investment.
Call your institution's physician contract liaison to review a sample contract and gain access to the MGMA physician salary survey. Remember these data includes responses from vascular surgeons in both private practice and academics and may not reflect your true market value. Talk to the fellows who graduated before you to get an idea of what range of offers are out there.

Attend regional and national vascular surgery conferences to network and gain leads. The SVS young surgeon's forum at the annual meeting is a good introduction into the job search process. The most comprehensive review to date is the Mote Vascular Symposium put on by Dr. Russell H. Samson's group out of Sarasota, FL. Beg, borrow, or steal the weekend off to attend this meeting.

Our goal as graduating fellows is to not only provide exceptional care to the patients with vascular disease in the communities we join, but to modernize the practice of our partners. Whether this means an aggressive endovascular approach to aneurysmal disease or overseeing the redesign of the practice's website, by virtue of training in the information age we are well prepared.

Every page of this newspaper is geared toward making us better vascular surgeons. So please, let's start by bringing the interview process in to the 21st century.

Christopher Everett, M.D., is a Vascular Surgery Fellow, year two, at the Greenville University Hospital Medical Center, Greenville, SC.

Author and Disclosure Information

Publications
Sections
Author and Disclosure Information

Author and Disclosure Information

As we strive to provide the most advanced care possible to our patients with vascular disease, there is one area of our practice that stands out as being as archaic as cellophane wrapping of aneurysms: the process of finding a job.

Instead of an enlightening quest to find the perfect practice, many graduating vascular fellows have found it to be a tiring and laborious process. The excitement around the opportunity to finally do what we have trained so long and hard to do has been eclipsed in frustration by a disorganized and antiquated interview process.

The business side of medicine has very much been on display in the popular media with the institution of new healthcare reform legislation. No financially viable business would fly a client halfway across the country to find out if they're a 'good guy' prior to discussing a potential deal. This should not be the routine in medicine, either.

I implore vascular surgeons to utilize technology to modernize the process of expanding their practice. Replace the 'good old boy' and device rep networks with posts on the Society for Vascular Surgery (SVS) Job bank. Loose the recruiters and write a concise description of your current practice and what you have to offer. Replace the mandatory meet and greet first interview with a video conference via SKYPE. This will obviate the need to clear your schedule for the day and shelve the awkward interview in the operating theater.

A colleague of mine has equated interviewing for a vascular surgery job with trying to buy a used car without knowing the price. Even if it requires a non-disclosure agreement, opening the books to a prospective partner early in the interview process is vital. In addition to demonstrating integrity and building good will, it validates your case mix and volume. Without this knowledge, an applicant cannot make an informed decision to join a practice.

Most importantly, be professional. If a candidate is not what you're looking for, be straightforward and tactful. Communicate; don't string someone along in case another prospect falls through.

For those of you who are getting ready to or are still looking for your first job- do your homework. Converse with your faculty members, get in touch with the local device reps, and talk to former fellows about your job prospects. On the interview don't hesitate to pull the ancillary staff aside for their opinion of the group. One of my co-fellows did this and learned that the surgeon had not booked a case in the operating room for six months. The hospital administrators confessed that they were interviewing to replace the surgeon, not hire on a partner.

There are a multitude of resources available to arm yourself when it comes to contract negotiations. Read "The Physician's Comprehensive Guide to Negotiating" (SEAK, Inc.) by Steven Babitsky and James J. Mangraviti, Jr. I obtained a copy through interlibrary loan and read it in two days between cases. It is well worth the small time investment.
Call your institution's physician contract liaison to review a sample contract and gain access to the MGMA physician salary survey. Remember these data includes responses from vascular surgeons in both private practice and academics and may not reflect your true market value. Talk to the fellows who graduated before you to get an idea of what range of offers are out there.

Attend regional and national vascular surgery conferences to network and gain leads. The SVS young surgeon's forum at the annual meeting is a good introduction into the job search process. The most comprehensive review to date is the Mote Vascular Symposium put on by Dr. Russell H. Samson's group out of Sarasota, FL. Beg, borrow, or steal the weekend off to attend this meeting.

Our goal as graduating fellows is to not only provide exceptional care to the patients with vascular disease in the communities we join, but to modernize the practice of our partners. Whether this means an aggressive endovascular approach to aneurysmal disease or overseeing the redesign of the practice's website, by virtue of training in the information age we are well prepared.

Every page of this newspaper is geared toward making us better vascular surgeons. So please, let's start by bringing the interview process in to the 21st century.

Christopher Everett, M.D., is a Vascular Surgery Fellow, year two, at the Greenville University Hospital Medical Center, Greenville, SC.

As we strive to provide the most advanced care possible to our patients with vascular disease, there is one area of our practice that stands out as being as archaic as cellophane wrapping of aneurysms: the process of finding a job.

Instead of an enlightening quest to find the perfect practice, many graduating vascular fellows have found it to be a tiring and laborious process. The excitement around the opportunity to finally do what we have trained so long and hard to do has been eclipsed in frustration by a disorganized and antiquated interview process.

The business side of medicine has very much been on display in the popular media with the institution of new healthcare reform legislation. No financially viable business would fly a client halfway across the country to find out if they're a 'good guy' prior to discussing a potential deal. This should not be the routine in medicine, either.

I implore vascular surgeons to utilize technology to modernize the process of expanding their practice. Replace the 'good old boy' and device rep networks with posts on the Society for Vascular Surgery (SVS) Job bank. Loose the recruiters and write a concise description of your current practice and what you have to offer. Replace the mandatory meet and greet first interview with a video conference via SKYPE. This will obviate the need to clear your schedule for the day and shelve the awkward interview in the operating theater.

A colleague of mine has equated interviewing for a vascular surgery job with trying to buy a used car without knowing the price. Even if it requires a non-disclosure agreement, opening the books to a prospective partner early in the interview process is vital. In addition to demonstrating integrity and building good will, it validates your case mix and volume. Without this knowledge, an applicant cannot make an informed decision to join a practice.

Most importantly, be professional. If a candidate is not what you're looking for, be straightforward and tactful. Communicate; don't string someone along in case another prospect falls through.

For those of you who are getting ready to or are still looking for your first job- do your homework. Converse with your faculty members, get in touch with the local device reps, and talk to former fellows about your job prospects. On the interview don't hesitate to pull the ancillary staff aside for their opinion of the group. One of my co-fellows did this and learned that the surgeon had not booked a case in the operating room for six months. The hospital administrators confessed that they were interviewing to replace the surgeon, not hire on a partner.

There are a multitude of resources available to arm yourself when it comes to contract negotiations. Read "The Physician's Comprehensive Guide to Negotiating" (SEAK, Inc.) by Steven Babitsky and James J. Mangraviti, Jr. I obtained a copy through interlibrary loan and read it in two days between cases. It is well worth the small time investment.
Call your institution's physician contract liaison to review a sample contract and gain access to the MGMA physician salary survey. Remember these data includes responses from vascular surgeons in both private practice and academics and may not reflect your true market value. Talk to the fellows who graduated before you to get an idea of what range of offers are out there.

Attend regional and national vascular surgery conferences to network and gain leads. The SVS young surgeon's forum at the annual meeting is a good introduction into the job search process. The most comprehensive review to date is the Mote Vascular Symposium put on by Dr. Russell H. Samson's group out of Sarasota, FL. Beg, borrow, or steal the weekend off to attend this meeting.

Our goal as graduating fellows is to not only provide exceptional care to the patients with vascular disease in the communities we join, but to modernize the practice of our partners. Whether this means an aggressive endovascular approach to aneurysmal disease or overseeing the redesign of the practice's website, by virtue of training in the information age we are well prepared.

Every page of this newspaper is geared toward making us better vascular surgeons. So please, let's start by bringing the interview process in to the 21st century.

Christopher Everett, M.D., is a Vascular Surgery Fellow, year two, at the Greenville University Hospital Medical Center, Greenville, SC.

Publications
Publications
Article Type
Display Headline
Fear and Loathing on the Interview Trail
Display Headline
Fear and Loathing on the Interview Trail
Sections
Article Source

PURLs Copyright

Inside the Article

About The Residents' Forum

Article Type
Changed
Tue, 12/13/2016 - 12:08
Display Headline
About The Residents' Forum

With this issue, we begin a new column which we have chosen to call Residents' Forum. We hope that Vascular Residents, Fellows, and others interested in entering this exciting field will consider submitting an essay on topics important to them related to vascular surgery and their careers.


-George Andros, M.D., Medical Editor

Author and Disclosure Information

Publications
Sections
Author and Disclosure Information

Author and Disclosure Information

With this issue, we begin a new column which we have chosen to call Residents' Forum. We hope that Vascular Residents, Fellows, and others interested in entering this exciting field will consider submitting an essay on topics important to them related to vascular surgery and their careers.


-George Andros, M.D., Medical Editor

With this issue, we begin a new column which we have chosen to call Residents' Forum. We hope that Vascular Residents, Fellows, and others interested in entering this exciting field will consider submitting an essay on topics important to them related to vascular surgery and their careers.


-George Andros, M.D., Medical Editor

Publications
Publications
Article Type
Display Headline
About The Residents' Forum
Display Headline
About The Residents' Forum
Sections
Article Source

PURLs Copyright

Inside the Article

Presentation, Not Treatment, May Determine Cost in CLI

Article Type
Changed
Tue, 12/13/2016 - 12:08
Display Headline
Presentation, Not Treatment, May Determine Cost in CLI

LAKE BUENA VISTA, FLA. - Endovascular treatment for critical limb ischemia offered no significant cost savings over open repair in a recent analysis of outcomes in 137 patients.

Of the 148 patients included in the retrospective review, 42% were treated with an endovascular procedure, 47% with an open procedure, and the remaining with a hybrid of the two. The mean costs were $49,802 for an endovascular hospitalization and $45,832 for an open repair hospitalization; these amounts were not significantly different, Dr. Nicholas Gargiulo III reported at the annual meeting of the Society for Clinical Vascular Surgery.

The mean lengths of stay were also similar, at 9.3 days and 10.4 days in the endovascular and open repair groups, respectively, said Dr. Gargiulo of Montefiore Medical Center, New York.

The percentage of patients discharged to a skilled nursing facility was 35% vs. 44% of endovascular and open repair patients, respectively. Although endovascular repair was associated with slightly increased likelihood of recovering enough function to be released to home upon hospital discharge, this difference also did not reach statistical significance.

Rates of readmission within 90 days were similar at 12% and 13% for the endovascular repair and open repair patients, respectively.

For hybrid repair patients, the hospitalization costs ($27,922) and length of stay (9.8 days) were lower, compared with the other groups. However, the readmission rate was much higher, at 50%, and the percentage discharged to home initially was lower at 28%.

The study included all of those patients who presented with critical limb ischemia at Montefiore Medical Center from January 1, 2007, through December 2007, for whom complete data were available. The patients, who had a mean age of 67 years and Rutherford Class 4 or 5 disease, underwent initial diagnostic evaluation with conventional arteriography, and the treatment approach was based on the anatomic TransAtlantic InterSociety Consensus II classification and adequate runoff.

A variety of endovascular interventions and open procedures were used. The perioperative mortality rate was 2.7%, and amputation-free survival was 94.6% at 1 year.

Most of the patients had hypertension and diabetes; a large variety of other comorbidities were seen as well.

About two-thirds of the cohort presented with rest pain, and the remainder presented with gangrene or ulceration. The hospitalization costs were higher in those who presented with gangrene and ulceration, Dr. Gargiulo said. Over the past year, more patients have been presenting with gangrene than with rest pain, and this is a concern, he added.

“Interestingly, the only thing that was different is that those with rest pain cost less than the patients with gangrene and ulceration … gangrene and ulceration increase the length of stay, increase readmission, and of course increase supplies and nursing services, resulting in an overall increase in mean cost,” Dr. Gargiulo said.

Possible cost-cutting measures include educational programs, new alliances with podiatry colleagues, prevention, and new trials, he added.

“In conclusion, endovascular and open procedures were equally cost effective in this diverse ethnic population of patients with critical limb ischemia, and patients with gangrene and ulceration have increased health care costs. It appears it's not the type of procedure which incurs cost, but the clinical presentation,” he said.

Dr. Gargiulo had no disclosures.

Author and Disclosure Information

Publications
Topics
Author and Disclosure Information

Author and Disclosure Information

LAKE BUENA VISTA, FLA. - Endovascular treatment for critical limb ischemia offered no significant cost savings over open repair in a recent analysis of outcomes in 137 patients.

Of the 148 patients included in the retrospective review, 42% were treated with an endovascular procedure, 47% with an open procedure, and the remaining with a hybrid of the two. The mean costs were $49,802 for an endovascular hospitalization and $45,832 for an open repair hospitalization; these amounts were not significantly different, Dr. Nicholas Gargiulo III reported at the annual meeting of the Society for Clinical Vascular Surgery.

The mean lengths of stay were also similar, at 9.3 days and 10.4 days in the endovascular and open repair groups, respectively, said Dr. Gargiulo of Montefiore Medical Center, New York.

The percentage of patients discharged to a skilled nursing facility was 35% vs. 44% of endovascular and open repair patients, respectively. Although endovascular repair was associated with slightly increased likelihood of recovering enough function to be released to home upon hospital discharge, this difference also did not reach statistical significance.

Rates of readmission within 90 days were similar at 12% and 13% for the endovascular repair and open repair patients, respectively.

For hybrid repair patients, the hospitalization costs ($27,922) and length of stay (9.8 days) were lower, compared with the other groups. However, the readmission rate was much higher, at 50%, and the percentage discharged to home initially was lower at 28%.

The study included all of those patients who presented with critical limb ischemia at Montefiore Medical Center from January 1, 2007, through December 2007, for whom complete data were available. The patients, who had a mean age of 67 years and Rutherford Class 4 or 5 disease, underwent initial diagnostic evaluation with conventional arteriography, and the treatment approach was based on the anatomic TransAtlantic InterSociety Consensus II classification and adequate runoff.

A variety of endovascular interventions and open procedures were used. The perioperative mortality rate was 2.7%, and amputation-free survival was 94.6% at 1 year.

Most of the patients had hypertension and diabetes; a large variety of other comorbidities were seen as well.

About two-thirds of the cohort presented with rest pain, and the remainder presented with gangrene or ulceration. The hospitalization costs were higher in those who presented with gangrene and ulceration, Dr. Gargiulo said. Over the past year, more patients have been presenting with gangrene than with rest pain, and this is a concern, he added.

“Interestingly, the only thing that was different is that those with rest pain cost less than the patients with gangrene and ulceration … gangrene and ulceration increase the length of stay, increase readmission, and of course increase supplies and nursing services, resulting in an overall increase in mean cost,” Dr. Gargiulo said.

Possible cost-cutting measures include educational programs, new alliances with podiatry colleagues, prevention, and new trials, he added.

“In conclusion, endovascular and open procedures were equally cost effective in this diverse ethnic population of patients with critical limb ischemia, and patients with gangrene and ulceration have increased health care costs. It appears it's not the type of procedure which incurs cost, but the clinical presentation,” he said.

Dr. Gargiulo had no disclosures.

LAKE BUENA VISTA, FLA. - Endovascular treatment for critical limb ischemia offered no significant cost savings over open repair in a recent analysis of outcomes in 137 patients.

Of the 148 patients included in the retrospective review, 42% were treated with an endovascular procedure, 47% with an open procedure, and the remaining with a hybrid of the two. The mean costs were $49,802 for an endovascular hospitalization and $45,832 for an open repair hospitalization; these amounts were not significantly different, Dr. Nicholas Gargiulo III reported at the annual meeting of the Society for Clinical Vascular Surgery.

The mean lengths of stay were also similar, at 9.3 days and 10.4 days in the endovascular and open repair groups, respectively, said Dr. Gargiulo of Montefiore Medical Center, New York.

The percentage of patients discharged to a skilled nursing facility was 35% vs. 44% of endovascular and open repair patients, respectively. Although endovascular repair was associated with slightly increased likelihood of recovering enough function to be released to home upon hospital discharge, this difference also did not reach statistical significance.

Rates of readmission within 90 days were similar at 12% and 13% for the endovascular repair and open repair patients, respectively.

For hybrid repair patients, the hospitalization costs ($27,922) and length of stay (9.8 days) were lower, compared with the other groups. However, the readmission rate was much higher, at 50%, and the percentage discharged to home initially was lower at 28%.

The study included all of those patients who presented with critical limb ischemia at Montefiore Medical Center from January 1, 2007, through December 2007, for whom complete data were available. The patients, who had a mean age of 67 years and Rutherford Class 4 or 5 disease, underwent initial diagnostic evaluation with conventional arteriography, and the treatment approach was based on the anatomic TransAtlantic InterSociety Consensus II classification and adequate runoff.

A variety of endovascular interventions and open procedures were used. The perioperative mortality rate was 2.7%, and amputation-free survival was 94.6% at 1 year.

Most of the patients had hypertension and diabetes; a large variety of other comorbidities were seen as well.

About two-thirds of the cohort presented with rest pain, and the remainder presented with gangrene or ulceration. The hospitalization costs were higher in those who presented with gangrene and ulceration, Dr. Gargiulo said. Over the past year, more patients have been presenting with gangrene than with rest pain, and this is a concern, he added.

“Interestingly, the only thing that was different is that those with rest pain cost less than the patients with gangrene and ulceration … gangrene and ulceration increase the length of stay, increase readmission, and of course increase supplies and nursing services, resulting in an overall increase in mean cost,” Dr. Gargiulo said.

Possible cost-cutting measures include educational programs, new alliances with podiatry colleagues, prevention, and new trials, he added.

“In conclusion, endovascular and open procedures were equally cost effective in this diverse ethnic population of patients with critical limb ischemia, and patients with gangrene and ulceration have increased health care costs. It appears it's not the type of procedure which incurs cost, but the clinical presentation,” he said.

Dr. Gargiulo had no disclosures.

Publications
Publications
Topics
Article Type
Display Headline
Presentation, Not Treatment, May Determine Cost in CLI
Display Headline
Presentation, Not Treatment, May Determine Cost in CLI
Article Source

PURLs Copyright

Inside the Article